Breast and Cosmetic
The correct response is Option C. The key element to avoid postoperative deformities in liposuction is avoiding the zones of adherence. These are zones where the superficial fascial system sends elements through the deep compartment attaching to the investing fascia of the underlying musculature. These zones of adherence accentuate localized fat deposits between them. The lateral gluteal depression is just superior to the outer lateral thigh and attention to that zone is important when performing liposuction along the outer thigh area. The other zones are not related to the outer thigh, except the inferolateral iliotibial tract, which is distal to the area of concern.
2013 A 21-year-old female cheerleader is scheduled to undergo suction-assisted lipectomy for contouring of the outer thigh. She is physically fit and has lost 10 lb (4.5 kg) over the past 6 months. Current weight is 145 lb (65.8 kg). Preoperative examination showed adiposity along the outer thigh area. To decrease the risk of postoperative deformities in this patient, particular attention should be paid to which of the following zones of adherence? A ) Gluteal crease B ) Iliotibial tract C ) Lateral gluteal depression D ) Mid medial thigh E ) Posterior thigh
The correct response is Option C. Fracturing the nose with medial and lateral osteotomies is necessary to mobilize the bony nasal pyramid and correct the collapsed left nasal bone by out-fracturing it. A left spreader graft is also necessary to keep the left internal nasal valve open and prevent the left nasal bone from collapsing and recurrence of the deformity. A spreader graft is not necessary on the right because the right nasal passage is widely patent. Placing bilateral spreader grafts would give the nasal dorsum a wide appearance and is not required. Since the rhinoplasty was performed through a closed technique, the columella is not destabilized, which can happen during the open rhinoplasty technique. A columellar strut is not necessary. The patient has left internal nasal valve collapse, not external nasal valve collapse. An alar batten graft is not indicated in this situation.
2013 A 21-year-old man comes to the office because of difficulty breathing through the left nostril after he was struck in the nose during a soccer game 1 year ago. He had a nosebleed at the time but did not seek medical treatment. Physical examination shows a depressed left nasal sidewall and a buckle in the nasal septum. He has increased difficulty breathing through the left naris when the right naris is occluded, although the nostril appears open. The right nasal passage is widely patent. Closed rhinoplasty with septoplasty is planned. In addition to submucous resection of septal cartilage, which of the following is the most appropriate technique for correction of the nasal airway obstruction? A ) Lateral osteotomies with a right spreader graft B ) Left lateral osteotomy with a columellar strut C ) Left lateral osteotomy with a left spreader graft D ) Medial osteotomies with bilateral spreader grafts E ) Right medial osteotomy with a left alar batten graft
The correct response is Option E. The first suture for a broad, bulbous tip is the transdomal suture. This suture will narrow the dome and narrow the convexity of the lateral crus with mild increased tip projection. The interdomal suture is used mainly if there is asymmetry in domal height or to reduce the interdomal width. The columellar septal suture is used to establish tip strength and integrity, which might have been lost with a transfixion incision. Lateral crural mattress sutures are used to create lateral crural concavity. Medial crural sutures or medial crural septal sutures are used to increase or decrease tip projection.
2013 A 21-year-old woman comes to the office for consultation regarding rhinoplasty. She says she is dissatisfied with the tip of her nose because it is "too big and wide." Which of the following is the most effective suture technique to achieve a more refined triangular tip in this patient? A ) Columellar septal B ) Interdomal C ) Lateral crural mattress D ) Medial crural E ) Transdomal
The correct response is Option D. Physical examination of a tuberous breast would show herniation of the nipple-areola complex. A constricted inframammary fold, rather than an effaced inframammary fold, is often associated with tuberous breast deformity. Macromastia and/or grade III ptosis of the nipple-areola complex are not standard components of tuberous breast deformity. Absence of the sternal head of the pectoralis muscle is a characteristic feature of Poland syndrome.
2013 A 23-year-old woman comes to the office for consultation regarding surgical correction of a tuberous breast deformity. On physical examination, which of the following characteristics is most likely in this patient? A ) Absence of the sternal head of the pectoralis muscle B ) Effacement of the inframammary fold C ) Grade III ptosis of the nipple-areola complex D ) Herniation of breast tissue into the nipple-areola complex E ) Macromastia
The correct response is Option D. The chromophore for the pulsed-dye laser at a wavelength of 585 nm is oxyhemoglobin. Thus, this laser is best suited to treat vascular lesions. Rhytides, acne scars, and dyschromias can also be managed by ablative resurfacing techniques, such as a carbon dioxide laser, which is absorbed by water. Er:YAG has a wavelength of 2940 nm and is absorbed by water. This laser causes less collateral thermal necrosis than a carbon dioxide laser. Acne scarring is best managed with infrared lasers at wavelengths of 1064 to 1540 nm. These include the Nd:YAG, diode, and erbium lasers. Tattoos are best managed with a Q-switched ruby laser at a wavelength of 694 nm, which is absorbed by melanin and carbon pigments.
2013 A 24-year-old woman comes to the office because of a capillary malformation of the right cheek. Which of the following lasers is the most appropriate treatment in this patient? A ) Carbon dioxide laser (10,200 nm) B ) Er:YAG (2940 nm) C ) Nd:YAG (1064 nm) D ) Pulsed-dye (585 nm) E ) Q-switched ruby (694 nm)
The correct response is Option A. The most important factor in determining the maximum acceptable prosthesis size in this patient is breast base width. Grade of nipple-areola ptosis, areola diameter, maximum manufactured prosthesis volume, and pectoralis height may all impact overall appearance of the breast but do not have an impact on breast prosthesis size choice.
2013 A 24-year-old woman with bilateral micromastia comes for consultation regarding augmentation mammaplasty. The patient says she would like her breasts to be "as big as possible." On examination, which of the following is the most important factor in determining the maximum acceptable prosthesis size for this patient? A ) Breast base width B ) Diameter of the areola C ) Grade of nipple-areola ptosis D ) Maximum manufactured prosthesis volume E ) Pectoralis muscle height-to-prosthesis height ratio
The correct response is Option D. When performing a submucous resection for airway obstruction, leaving an intact L-strut is recommended for nasal support. When an L-strut fracture occurs, it should be repaired to avoid middle-third nasal collapse. The strut tends to rotate posteriorly, creating a saddle-nose deformity. Spreader grafts secured with sutures will act like a batten graft and secure the L-strut in place. A columella graft is used to support the structure and position of the lower third of the nose. A crural turnover graft is used to support weakened or deformed lower lateral cartilages. A dorsal onlay graft is used for dorsal augmentation and would not adequately support the fracture. A spring graft spans between both upper lateral cartilages and is used to widen the middle vault.
2013 A 25-year-old man undergoes a submucous resection of the septum for airway obstruction. While the surgeon is scoring the remaining L-strut, the cartilage fractures along the dorsal strut. Reconstruction with which of the following grafts is the most appropriate next step in management? A ) Columella B ) Crural turnover C ) Dorsal onlay D ) Spreader E ) Spring
The correct response is Option E. Concern regarding an association between silicone breast prostheses and connective tissue disease was raised in the 1980s and early 1990s, eventually leading to the US Food and Drug Administration (FDA) moratorium of the use of silicone breast prostheses in augmentation mammaplasty. Since then, multiple cohort studies and case control studies in Europe and North America have failed to determine a causative association between silicone breast prostheses and any traditional or atypical connective tissue diseases.
2013 A 25-year-old woman is considering augmentation mammaplasty with silicone prostheses. The patient asks about the associated risks of developing connective tissue disease. Which of the following risk assessments is most accurate in this patient? A ) Increased risk of extracapsular leak only B ) Increased risk of intra- and extracapsular leak C ) Increased risk only if the silicone migrates to the lymph node D ) Increased risk only in the pre-1990 prostheses E ) No increased risk
The correct response is Option B. A woman's lifetime risk of developing breast and/or ovarian cancer is greatly increased if she inherits an altered BRCA1 or BRCA2 gene. Women with an inherited alteration in one of these genes have an increased risk of developing these cancers at a young age (before menopause) and often have multiple close family members with the disease. These women may also have an increased chance of developing colon cancer. Men with an altered BRCA1 or BRCA2 gene also have an increased risk of breast cancer (primarily if the alteration is in BRCA2) and possibly prostate cancer. Alterations in the BRCA2 gene have also been associated with an increased risk of lymphoma, melanoma, and cancers of the pancreas, gallbladder, bile duct, and stomach in some men and women. According to estimates of lifetime risk, approximately 13.2% (132 of 1000 individuals) of women in the general population will develop breast cancer, compared with estimates of 36 to 85% (360 to 850 of 1000) of women with an altered BRCA1 or BRCA2 gene. In other words, women with an altered BRCA1 or BRCA2 gene are 3 to 7 times more likely to develop breast cancer than women without alterations in those genes. Lifetime risk estimates of ovarian cancer for women in the general population indicate that 1.7% (17 of 1000) will get ovarian cancer, compared with 16 to 60% (160 to 600 of 1000) of women with altered BRCA1 or BRCA2 genes. No data are available from long-term studies of the general population comparing the cancer risk in women who have a BRCA1 or BRCA2 alteration with women who do not have an alteration in these genes. Therefore, these figures are estimated ranges that may change as more research data are added.
2013 A 26-year-old woman with a strong family history of breast cancer undergoes genetic testing. She is found to have a deleterious mutation of the BRCA1 gene. Which of the following best describes her lifetime risk for the development of breast cancer when compared with women without this mutation? A ) Two times greater B ) Six times greater C ) Ten times greater D ) Fifteen times greater E ) Twenty times greater
The correct response is Option D. Body dysmorphic disorder (BDD) is a preoccupation with an imagined defect in one's appearance, or, if a slight physical anomaly is present, the person's concern is marked excessive. According to the diagnostic criteria in the DSM-IV, the preoccupation should last for at least one hour per day, and have clinically significant impairment in social or occupational functioning, as in this clinical case. Approximately 5% of patients seeking aesthetic surgery have BDD. The most common preoccupation in BDD is with the nose. Between 20 and 33% of patients seeking rhinoplasty have at least some features of BDD. Previous reports suggest that rhinoplasty in these patients is associated with marked dissatisfaction and an increase in BDD symptoms, not an improvement. Interestingly, the commonly used mnemonic of SIMON to identify a BDD patient - "single, immature male, overly narcissistic" was disproven by the research of Picavet, et al., who found no relationship between sex or marital status and BDD. BDD patients do best with psychiatric help and are likely to have worsened quality of life if surgery is performed.
2013 A 28-year-old man who is an aspiring actor comes to the office for consultation regarding rhinoplasty. He says he feels that his nose is preventing him from being a successful actor. Examination shows a 1-mm dorsal hump and a 0.5-mm supratip depression. No abnormalities of nasal width and tip shape are noted, and nasal symmetry is acceptable. Examination of the internal airway is within the normal ranges. Which of the following is the most appropriate management? A ) External rhinoplasty with rasping of the nasal hump, osteotomy, and infracture B ) Injection of hyaluronic acid gel fillers C ) Internal rhinoplasty with hump reduction D ) Referral to psychiatric consultation E ) Tip rhinoplasty only
The correct response is Option C. When harvesting septal cartilage as a graft, a minimum of 10 mm of a dorsal-caudal L-shaped strut should remain to prevent collapse. While some authors advocate a more conservative approach, leaving 15 mm, others are more aggressive, leaving as little as 8 mm. The generally accepted rule of thumb, however, is 10 mm.
2013 A 29-year-old woman comes for evaluation because she is dissatisfied with the appearance of her nose. Physical examination shows internal nasal valve collapse. Rhinoplasty with spreader grafts and the use of septal cartilage is planned. Which of the following best represents the minimum amount of dorsal-caudal strut that must be retained to prevent collapse? A ) 2 mm B ) 5 mm C ) 10 mm D ) 15 mm E ) 20 mm
The correct response is Option D. Soft-tissue fillers are minimally invasive and offer an attractive alternative to revision rhinoplasty. A variety of fillers are available, including hyaluronic acid derivatives, calcium hydroxyapatite gel, and silicone. Because of the risk of adverse reactions, silicone injectables should be avoided. Hyaluronic acid-derived and calcium hydroxyapatite fillers are better tolerated but still may occasionally cause infection, necrosis, or thinning of the soft-tissue envelope. To minimize the risk of these complications, fillers should be placed in the sub-superficial musculoaponeurotic system plane just above the plane of the periosteum. This will lessen the chance of visibility and palpability. Also, use should be restricted to the nasal dorsum and nasal sidewalls. The nasal tip and alae should be avoided because necrosis is at a much higher risk. The use of soft-tissue fillers in the nose should be approached with caution.
2013 A 29-year-old woman comes to the office 1 year postoperatively after rhinoplasty with slight irregularities and asymmetry of the nasal bridge and tip. Physical examination shows mild depression of the nasal dorsum and asymmetric alar domes. Injection of a calcium hydroxyapatite gel is planned. Which of the following combinations of injection depth and anatomical location is most appropriate in this patient to minimize complications? A ) Subcutaneous area into the nasal alae B ) Subcutaneous area into the nasal alar domes C ) Subperiosteal area into the nasal sidewall D ) Supraperichondrial area into the nasal dorsum E ) Supraperichondrial area into the nasal tip
The correct response is Option B. A known risk of liposuction is fat embolism syndrome (FES), and clinical examination remains the gold standard for diagnosis. The three classic symptoms of FES are respiratory distress, decreased cerebral function, and petechial rash. This generally occurs within 48 hours postoperatively. With manual disruption of both fatty tissue and blood vessels that occurs with liposuction, microparticulate fat showers the lung, brain, kidney, and skin with emboli, leading to mechanical obstruction and/or a biochemical inflammatory reaction. Overall mortality from FES after liposuction is approximately 10 to 15%. Lidocaine at high concentrations can cause serious central nervous system disturbances, including anxiety, agitation, psychosis, seizures and/or coma, and cardiovascular toxicity, including arrhythmias and hypotension. Symptoms of toxicity include light-headedness, euphoria, digital paresthesia, restlessness, and drowsiness. Symptoms of objective toxicity include nausea, vomiting, tremors, blurred vision, tinnitus, confusion, excitement, psychosis, and muscular fasciculations. Seizures and cardiorespiratory depression occur typically when blood levels reach 8 to 12 mcg/mL. Above this level, subjects may become comatose, with respiratory arrest and cardiac asystole. A rash is not associated with lidocaine toxicity. The safe dose of lidocaine has been established as 35 mg/kg during liposuction, and some authors suggest using 55 mg/kg may be safe. Anaphylaxis from a drug allergy also does not cause a rash but can cause respiratory distress from laryngeal edema. Pulmonary edema is characterized by respiratory distress that does not respond to oxygen therapy. This can occur because of fluid overload or from physiologic issues related to extubation. Pulmonary embolism must be considered with respiratory distress but is not associated with rash and can occur at any time postoperatively.
2013 A 30-year-old woman is brought to the emergency department 1 day after undergoing outpatient liposuction because of difficulty breathing and confusion. A tumescent technique with lidocaine was used during the procedure, in which 4000 mL of tumescent fluid was infiltrated and 4000 mL of lipoaspirate was removed. Physical examination shows respiratory distress and petechial rash. Which of the following is the most likely cause of this patient's condition? A ) Drug allergy B ) Fat embolism C ) Lidocaine toxicity D ) Pulmonary edema E ) Pulmonary embolism
The correct response is Option D. In addition to a thorough medical history, careful analysis of the patient's goals and evaluation of the anatomy is the basis for correctly selecting the optimum choice for aesthetic restoration of the abdomen after child bearing. This patient's skin redundancy above and below the navel requires that panniculectomy incorporate downward tension for the portions superior to and inferior to the umbilicus. Miniabdominoplasty fails to include supraumbilical skin tensioning. Liposuction alone may actually worsen the degree of skin laxity by deflating fat compartments or disrupting musculocutaneous suspensory fasciae. It is unlikely that prosthetic material will be necessary to repair a diastasis when plication (folding adjacent fascial halves with sutures) suffices unless a concurrent hernia of significant proportions exists. Traditional abdominoplasty incorporates mobilization of the abdominal panniculus, addresses laxity above and below the navel, and may incorporate more recent techniques such as progressive tension sutures to ensure an optimum aesthetic result while minimizing the need for revision. Fleur-de-Lis abdominoplasty involves a vertical incision and is not indicated. Cryolipolysis is not effective in this level of skin laxity.
2013 A 32-year-old woman, gravida 3, para 3, comes to the office for consultation regarding a slimmer abdominal contour. She says the most weight she lost after pregnancy was 30 lb (13.6 kg). Current BMI is 23 kg/m2. Examination shows mild diastasis recti and vertical skin redundancy above and below the umbilicus. No hernias or scars are noted. Which of the following is the most appropriate treatment? A ) Suction-assisted lipectomy B ) Repair of the diastasis with prosthetic mesh and cryolipolysis C ) Miniabdominoplasty and suction-assisted lipectomy of the epigastrium D ) Abdominoplasty with rectus plication E ) Fleur-de-Lis abdominoplasty
The correct response is Option B. The most common medial innervation of the nipple-areola complex is mainly 57% from the anterior cutaneous branches of the third and fourth intercostal nerves. The third intercostal nerve accounts for 21.4%. They always reach the areolar edge between 8 and 11 o'clock on the left and 1 and 4 o'clock on the right. The nerve innervation to the nipple-areola complex is important in planning different incisions around the areola in both reduction mammaplasty and mastopexy.
2013 A 33-year-old woman comes to the office for consultation because she is dissatisfied with the "sagging" appearance of her breasts. Examination shows grade II ptosis and loss of fullness in the upper pole. A vertical mastopexy is planned. The most common medial innervation to the nipple-areola complex is the anterior cutaneous branches of which of the following intercostal nerves? A ) Second and third B ) Third and fourth C ) Fourth and fifth D ) Fifth and sixth E ) Sixth and seventh
The correct response is Option A. Phyllodes are large benign tumors that occur primarily in the perimenopausal patient. Previously, they were referred to as cystosarcoma phyllodes, a term coined in 1838 because the tumors are fleshy and have a gross leaf-like intracanalicular growth pattern. However, this is a misnomer because these tumors do not behave like sarcomas and are rarely malignant. The histologic characteristics that separate fibroadenomas from phyllodes tumors are not well defined and have been somewhat controversial. Nevertheless, phyllodes tumors typically are large fibroadenomas that histologically have more stromal cellularity than that seen in the typical fibroadenoma. The classification of benign versus malignant phyllodes tumors is not sharply delineated, and the term borderline lesion may be more appropriate. Borderline lesions have more mitoses per high-power field and moderate nuclear pleomorphism. They have a tendency to recur after local excision but do not demonstrate true malignant behavior. When metastases of a phyllodes tumor have been reported, there have been obvious sarcomatous elements such as liposarcoma or rhabdomyosarcoma in the lesion. The surgical treatment of phyllodes tumors has recently been redefined. In the past, simple or radical mastectomies were recommended for the treatment of large phyllodes tumors. Currently, most surgeons perform more conservative surgery. Several clinical studies have recommended the excision of tumors with 1-cm clear margins or mastectomy if breast conservation is impossible.
2013 A 33-year-old woman comes to the office with a 6-cm rapidly growing tumor of the left breast. She wears a size 36C brassiere. The tumor has a bluish hue and skin veins are dilated. A phyllodes tumor is diagnosed, and surgical excision is planned. Which of the following is the most appropriate surgical procedure to treat this patient? A ) Excision with 1-cm margin B ) Excision with 2-cm margin C ) Excisional biopsy D ) Modified radical mastectomy E ) Radical mastectomy
The correct response is Option A. Knowledge of the innervation of the external ear is critical to the understanding of its embryologic development, as well as in the delivery of adequate local anesthesia for minor surgical procedures. Sensation to the external ear is derived from several cranial and extracranial nerve branches. The great auricular (C2 to C3) and lesser occipital (C2) are cranial nerves which innervate the posterior aspect of the auricle and lobule. While the distribution is variable, in most cases the lesser occipital supplies the superior ear and mastoid region while the great auricular nerve supplies the inferior ear and a portion of the preauricular area. The anterior surface of the ear, including the helix, scapha, and concha, is supplied by the auriculotemporal nerve (V3 trigeminal) and is most likely to be injured in a microvascular decompression for the treatment of trigeminal neuralgia. Branches of the vagus (X) and glossopharyngeal (IX) nerve innervate the external auditory meatus. The innervation to the external ear follows its embryologic branchial arch origins with the great auricular nerve innervating first branchial arch structures and the auriculotemporal nerve innervating second branchial arch structures. An auriculotemporal nerve block provides anesthesia to the helix and tragus and is approached by injecting 2 to 4 mL of anesthesia superiorly and anteriorly to the tragus. The great auricular nerves and lesser occipital nerves are blocked by injecting 2 to 4 mL of anesthetic to the posterior sulcus from the inferior aspect of the earlobe. This will provide anesthesia to the earlobe and lateral helix.
2013 A 35-year-old woman is evaluated because of numbness of the upper helical rim of the left ear 30 days after she underwent neurosurgical decompression to treat facial pain. Which of the following nerves was most likely injured? A ) Auriculotemporal B ) Glossopharyngeal C ) Great auricular D ) Lesser occipital E ) Vagus
The correct response is Option A. Minimizing complications after rhinoplasty is a priority for every surgeon performing the procedure. Perioperative steroid administration has been shown to decrease postoperative edema and ecchymosis in a number of prospective randomized trials. In an effort to further elucidate the significance of the data and develop an evidence-based algorithm for steroid administration, a meta-analysis of the existing literature was performed. All articles were reviewed for relevant data, which were extracted, pooled, and compared. Seven prospective randomized trials investigating perioperative steroid use in rhinoplasty have been conducted and reported. Four of these studies had the same method of patient edema and ecchymosis assessment, and their data were compared. Based on results from the four relevant studies, perioperative steroid use significantly reduces postoperative edema and ecchymosis of the upper and lower eyelids at 1 day and 7 days postoperatively (P < .0001). Preoperative steroid administration decreases postoperative upper and lower eyelid edema at 1 day preoperatively, when compared with postoperative administration (P < .05). Extended dosing is superior to one-time dosing (P < .05). Perioperative steroid use decreases postoperative edema and ecchymosis associated with rhinoplasty. Preoperative administration is superior to postoperative, and extended dosing is superior to singular. Based on these results, evidence-based guidelines for perioperative steroid administration can be given.
2013 A 35-year-old woman is scheduled to undergo functional septorhinoplasty for nasal airway obstruction. In this patient, perioperative administration of corticosteroids is most likely to have which of the following effects on edema and ecchymosis? Edema Ecchymosis A ) Decreased decreased B ) Decreased no change C ) Increased increased D ) No change decreased E ) No change no change
The correct response is Option D. As surgical approaches to breast cancer treatment have evolved, nipple-sparing mastectomy (NSM) has emerged as an alternative to other approaches. It was initially used for prophylactic mastectomies, and patients reported increased satisfaction and body image with nipple-areola complex (NAC) preservation. The role of NSM has been expanded to therapeutic mastectomy, and with that there has been increased research in the oncologic safety of this approach. Studies have evaluated therapeutic NSM in the context of invasive ductal carcinoma, invasive lobular carcinoma, and ductal carcinoma in situ. The type of cancer does not appear to be associated with the oncologic safety of NSM. Several studies have demonstrated an inverse association between NAC involvement and distance of the tumor from the nipple. While these studies have varied in their distance cutoffs, nipple involvement is reported to be over 50% when the tumor-nipple distance is less than 2 cm, as noted in one study. There is a direct correlation between tumor size and NAC involvement—the same study cited data that when the tumor was greater than 4 cm, the likelihood of nipple involvement was greater than 50%. One published screening algorithm for plastic surgery includes tumor size less than 3 cm, and tumor location greater than 2 cm from the nipple as criteria for NSM candidacy. A periareolar scar, if large, may compromise the blood supply to the NAC. Acceptable incisions for NSM, however, include a periareolar incision of 25 to 50%.
2013 A 41-year-old woman comes to the office because of an invasive ductal carcinoma of the left breast. On mammography, the tumor is 3 cm from the nipple and measures 4 cm. A left-sided lateral periareolar scar extending from the 12 o'clock to the 3 o'clock position from a previous biopsy is noted. The patient wishes to undergo a nipple-sparing mastectomy. Which of the following findings places the patient at greatest oncologic risk, including risk for de novo or recurrent cancer or inadequate surgical margins, with this procedure? A ) Distance of tumor to nipple B ) Patient age C ) Presence of the periareolar scar D ) Size of tumor E ) Type of tumor
The correct response is Option B. Anagen effluvium occurs after an insult to the hair follicle that impairs its mitotic or metabolic activity. This hair loss is commonly associated with chemotherapy. The characteristic finding in anagen effluvium is the tapered fracture of the hair shafts. The hair shaft narrows as a result of damage to the matrix. Eventually, the shaft fractures at the site of narrowing and causes the loss of hair. Hair regrowth occurs after the cessation of chemotherapy. Androgenetic or androgenic alopecia is caused by the action of androgens. Dihydrotestosterone (DHT) is partially to blame, as it is in men. Androgenic alopecia can be caused by a variety of factors tied to the actions of hormones, including some ovarian cysts, taking high-androgen-index birth control pills, pregnancy, and menopause. Telogen effluvium is attributable to stress on the body, such as childbirth, malnutrition, severe infection, major surgery, or extreme mental stress. Many of the 90% or so of hairs in the growing (anagen) or transitional (catagen) phases can shift all at once into the resting (telogen) phase. A few weeks to several months after the stressful event, a shedding phenomenon called telogen effluvium begins. It is possible to lose handfuls of hair at a time. This phenomenon is usually self-limited, and hair growth returns. Alopecia areata is an inflammatory condition thought to result from the immune system attacking the hair follicles at the root. Treatment may include steroids or minoxidil. Hair loss can be temporary or permanent. Traction alopecia is caused by localized trauma from tight hairstyles, braids, cornrows, etc. If recognized early enough, the hair will grow back.
2013 A 41-year-old woman comes to the office for consultation regarding breast reconstruction after mastectomy. She is also embarrassed by her sudden loss of hair as a result of chemotherapy with paclitaxel. All of her hair has fallen out, and she wears a wig. Which of the following is the most likely diagnosis for this patient's hair loss? A ) Alopecia areata B ) Anagen effluvium C ) Androgenetic alopecia D ) Telogen effluvium E ) Traction alopecia
The correct response is Option C. Fat grafting has been shown to successfully address acquired contour deformities in breast reconstruction. Changing the prosthesis to a saline prosthesis of the same size and dimension will not address the problem of the step-off between the prosthesis and the chest wall in this slender patient (BMI 20 kg/m2) which is due to decreased soft-tissue coverage in the superior pole. Likewise, increasing the size and dimensions of the silicone prosthesis would not correct this contour deformity and would also lose the symmetry with the contralateral breast. This patient, without excess lower abdominal skin and subcutaneous fat, is not a candidate for a deep inferior epigastric perforator flap. Placement of acellular dermal matrix as an inferolateral sling is typically performed in the initial stage of tissue expansion and reconstruction. It can help define the inframammary fold, which this patient does not need.
2013 A 42-year-old woman comes to the office because she is dissatisfied with an obvious step-off between the chest wall and the superior pole of the breast 6 months after she underwent immediate expander reconstruction of the left breast. No further adjuvant therapy was indicated. After full expansion, the tissue expander was removed, and a permanent smooth, round silicone prosthesis was placed. BMI today is 28 kg/m2. Examination shows a well-defined inframammary fold. The volume is matched to the contralateral breast. Which of the following is the most appropriate surgical procedure for correction of this patient's deformity? A ) Exchange of the silicone prosthesis with a silicone prosthesis of a larger volume and dimension B ) Exchange of the silicone prosthesis with a saline prosthesis of the same volume and dimension C ) Fat grafting to the superior pole and chest wall D ) Placement of acellular dermal matrix as an inferolateral sling E ) Removal of the prosthesis and reconstruction with a deep inferior epigastric perforator flap
The correct response is Option E. There is a significant and growing body of evidence showing that normal body temperature during surgery specifically reduces the likelihood of surgical site infections and reduces the risk of bleeding. For this patient with a long operative time and exposure of a significant amount of body surface area, she is at increased risk for hypothermia and surgical site infection. Fat necrosis is attributable to devascularization of fatty tissue. Seroma formation is attributable to inadequate drainage of the surgical wounds. The likelihood of deep venous thrombosis and pulmonary embolus is reduced by the use of subcutaneous heparin, low-molecular-weight heparin, and/or sequential compression devices.
2013 A 43-year-old woman is scheduled to undergo mastopexy and circumferential lower body lift in an accredited hospital setting for body contouring. She lost 150 lbs (68 kg) after undergoing bariatric surgery 3 years ago. The planned duration of the procedure is 6 hours. In the preoperative holding room, she is dressed in an air-heated gown. Maintenance of normothermia during the perioperative period is most likely to decrease the risk of which of the following complications in this patient? A ) Deep venous thrombosis B ) Fat necrosis C ) Pulmonary embolism D ) Seroma formation E ) Surgical site infection
The correct response is Option E. The patient described will have a 10 × 16-cm, full-thickness defect of the abdominal wall in the setting of previous radiation. A full-thickness defect this large in the setting of past radiation makes primary closure alone and primary closure with interpositional mesh a poor choice. Bilateral component separation will not be possible because the rectus muscles will be resected with the specimen. A pedicled anterolateral thigh flap will offer well-vascularized, nonradiated tissue that will include soft-tissue coverage as well as fascia for abdominal wall reconstruction. A free latissimus muscle flap, while providing well-vascularized tissue, would involve the increased risks of microsurgery and not provide fascia for closure. Therefore, Option E is most appropriate.
2013 A 43-year-old woman with a history of ovarian cancer is evaluated because of recurrence in the incision and bladder dome. History includes resection, chemotherapy, and radiation therapy. The gynecologic surgeon plans a wide resection of skin, fascia, bilateral lower rectus muscles, and bladder dome resulting in a 10 × 16-cm defect in the central lower abdomen. Which of the following is the most appropriate procedure to achieve abdominal closure? A ) Bilateral component separation B ) Primary closure only C ) Primary closure with interpositional mesh D ) Use of a free latissimus muscle flap E ) Use of a pedicled anterolateral thigh flap
The correct response is Option E. The key to the scenario described is that the plastic surgeon is out of the office and the man is not his patient. However, as soon as a prescription is written, he is now a patient, and the plastic surgeon is obligated to document this encounter. A prescription is a legal document and as such is subject to local, state, and federal laws. Each state has its own individual laws, many of which are very restrictive, demanding taking a history and conducting a physical examination before prescribing as well as maintaining written records of all treatments and prescriptions. Federal law limits its prescription writing laws to controlled substances. These laws require that the prescriber have a bona fide patient-physician relationship with any person for whom he or she prescribes controlled substances. This relationship includes maintenance of a written medical record. Violating these standards can jeopardize a license. Contacting his treating physician is unnecessary. Sending the man to the emergency department or ordering an x-ray study is unnecessary. HIPAA stands for the Health Insurance Portability and Accountability Act. This rule set national standards for the protection of individually identifiable health information. Having the form signed would not be the most appropriate next step.
2013 A 45-year-old man approaches his neighbor, a plastic surgeon, at a cocktail party. He was seen in the emergency department 1 day ago with a broken toe and was given a prescription for ibuprofen. He says his toe is still aching and asks for a prescription for acetaminophen with codeine, which he has taken before. The plastic surgeon asks the man about his medical history, and a physical examination shows "buddy" taped toes and moderate ecchymosis and swelling. The plastic surgeon writes the prescription for acetaminophen with codeine. Which of the following is the most necessary next step for the plastic surgeon? A ) Contact his treating physician B ) Have the man sign a Health Insurance Portability and Accountability Act (HIPAA) consent form C ) Order a repeat x-ray study D ) Send the man to the emergency department E ) Write a medical note of the encounter
The correct response is Option B. When performing brachioplasty, injury to the medial antebrachial cutaneous nerve can occur. This nerve runs along to the medial epicondyle and adjacent to the basilic vein. Maintaining superficial dissection at the ulnar aspect of the elbow and preserving the basilic vein is paramount in minimizing injury to the medial antebrachial cutaneous nerve. Although theoretically possible, injury to the ulnar and median nerves has not been reported with brachioplasty. The axillary nerve and posterior interosseous nerve should not be in the field of dissection when performing a brachioplasty.
2013 A 45-year-old woman comes to the office because of persistent dysesthesias of the medial forearm 2 months after she underwent brachioplasty for brachial ptosis. Examination shows a well-healed scar on the medial aspect of the arm extending to the proximal elbow. Which of the following nerves was most likely injured during the procedure? A ) Axillary B ) Medial antebrachial cutaneous C ) Median D ) Posterior interosseous E ) Ulnar
The correct response is Option D. Almost 60% of the blood flow to the breast is from the internal mammary artery. The second and third anterior perforating branches are most dominant. In a superomedial pedicle both the second and third descending branches are captured in the pedicle. In a pure medial pedicle, it is usually the third. The other branches do not constitute any significant contribution to the pedicle blood supply in a superomedial or medial reduction.
2013 A 45-year-old woman comes to the office for consultation regarding reduction mammaplasty because of pain of the neck and upper back. She currently wears a size 42 DDD brassiere and would like to be a C cup. A vertical reduction mammaplasty with a superomedial pedicle is planned. Which of the following is the dominant blood supply for this pedicle? A ) Ascending branch from the fifth intercostal space B ) Ascending branch from the sixth intercostal space C ) Descending branch from the first intercostal space D ) Descending branch from the second intercostal space E ) Descending branch from the fourth intercostal space
The correct response is Option E. Since the U.S. Food and Drug Administration approval and introduction of botulinum toxin type A in the 1990s, injectable cosmetic treatments have exponentially outnumbered cosmetic surgical treatments provided by plastic surgeons, and the prevalence of treatment continues to increase. Dysport is a product developed by Medicis (Scottsdale, AZ), which also has botulinum toxin as an active ingredient to reduce glabellar furrows. Dysport is contraindicated for individuals who have an allergy to cow's milk protein.
2013 A 45-year-old woman comes to the office for consultation regarding smoothing of furrows between the eyebrows. Which of the following findings is a contraindication to abobotulinum toxin type A (Dysport) treatment in this patient? A ) Diabetes mellitus B ) Ehlers-Danlos syndrome C ) Gluten intolerance D ) Lupus E ) Milk allergy
The correct response is Option B. Erythema following laser resurfacing is an anticipated consequence of therapy. Posttreatment erythema is more severe and of longer duration with carbon dioxide laser resurfacing when compared to the fractionated carbon dioxide or Er:YAG laser. Postoperative topical application of ascorbic acid has been shown to decrease the duration as well as the severity of erythema. Topical therapy with ascorbic acid should be applied following reepithelialization. Antibiotics or antivirals have not been shown to decrease erythema. Hydroquinone is a skin bleaching agent that does not treat erythema. Topical corticosteroids postoperatively may delay reepithelialization and have not been associated with a decrease in erythema.
2013 A 46-year-old woman comes to the office for evaluation of persistent erythema 2 weeks after she underwent full-face carbon dioxide laser resurfacing. Which of the following topical treatments is most appropriate to decrease this patient's postoperative erythema? A ) Amoxicillin B ) Ascorbic acid C ) Hydroquinone D ) Prednisone E ) Valacyclovir
The correct response is Option C. This patient suffers from pseudogynecomastia, also known as lipomastia. Reduction mammaplasty with repositioning the nipple at or above the inframammary fold, reduction of the size of the areola, removal of excess skin, and removal of excess fatty tissue will most appropriately correct this patient's deformity. However, many insurance companies recognize this code as a gender-specific code for women. This patient's concerns are primarily with appearance. Breast reduction should be performed as a cosmetic procedure. No good controlled studies show the benefit of radiofrequency in the treatment of gynecomastia or pseudogynecomastia. The safety and efficacy of phosphatidylcholine injections have yet to be established. Liposuction will not address the skin problem or correct the enlarged areolae. Mastectomy for gynecomastia is a surgical procedure for the removal of painful, periareolar glandular tissue usually in pubescent males and is sometimes covered by insurance.
2013 A 48-year-old man comes to the office because he is dissatisfied with the appearance of his "enlarged" breasts. He underwent gastric bypass surgery 2 years ago followed by a 110-lb (50-kg) weight loss. Current weight is 185 lb (84 kg), which has been stable for 6 months. Physical examination shows nipples with an enlarged areola inferior to the inframammary fold, excess skin with loss of skin elasticity, and a moderate amount of fatty tissue. No palpable or painful masses are noted. Which of the following is the most appropriate surgical procedure for correction of this patient's deformity? A ) Injection of phosphatidylcholine B ) Nonsurgical radiofrequency fat ablation C ) Reduction mammaplasty D ) Skin-sparing mastectomy E ) Ultrasound-assisted liposuction
The correct response is Option C. Silicone prostheses are radiopaque on mammography. Therefore, when placed in the subglandular position, a small percentage of breast tissue is obscured on mammography. Breast prostheses made completely of or in part with silicone have not been shown to cause a delay in detection of breast cancer. Women with breast prostheses are not more likely to develop breast cancer. Women with breast prostheses who have developed breast cancer are not diagnosed at a more advanced stage and do not have a worse prognosis or survival when compared with women without prostheses. Silicone prostheses are less likely to show superior pole rippling when compared with saline prostheses. If a saline prosthesis ruptures, the saline tends to become absorbed by the body, resulting in an obvious decrease in breast size after a few days. When silicone prostheses rupture, the silicone may remain intracapsular. These ruptures may change the breast shape slightly but usually do not change the size and are often subclinical.
2013 A 49-year-old woman is scheduled to undergo subglandular augmentation mammaplasty with silicone prostheses. During the preoperative discussion, the patient asks about postoperative complications with silicone versus saline prostheses. Which of the following is a disadvantage of using silicone in this patient? A ) Their rupture results in an obvious decrease in breast size B ) They are more likely to result in invasive breast cancer C ) They can obscure breast tissue on mammagraphy D ) They may show more rippling
The correct response is Option E. Partial or total nipple necrosis may be one of the most devastating complications of reduction mammaplasty. The incidence of compromise of the nipple-areola complex is typically less than 5% after breast reduction. Increased BMI is a risk factor for and increased risk of both nipple necrosis and wound healing complications. Reduction mammaplasty is frequently performed on an outpatient basis. Patients are seen within a week of surgery for wound checks, but this may be too late to address a nipple with vascular compromise. If nipple ischemia is noted at the time of surgery during inset, stitches should be released and vascularity reevaluated. An objective assessment of blood flow can be aided with the use of fluorescein injection and a Woods lamp, or with newer screening modalities that are currently being evaluated for this purpose. While inset may be reattempted, the patient will likely require conversion to a free nipple graft. The nipple should be grafted to well-vascularized, deepithelialized dermis and not to ischemic fat that may be part of the compromised pedicle. If nipple ischemia is detected in the early postoperative period, and there is no hematoma or issue with external compression, the nipple should be released from its inset position. This will relieve tension on the pedicle. If the nipple does not improve, the patient is taken back to the operating room for free nipple grafting. If nipple ischemia is not identified in the early postoperative period, the patient should be treated with conservative wound care until healing is complete. Nipple reconstruction can then be undertaken in a delayed manner.
2013 A 5-ft 1-in (155-cm), 185-lb (84-kg), 45-year-old woman comes to the office for follow-up 1 week after she underwent reduction mammaplasty with a medial pedicle in the outpatient facility. BMI is 35 kg/m2. On examination, the right nipple-areola complex is dusky and cool. Nipple ischemia is suspected. Which of the following is the most appropriate next step in management? A ) Debridement of the necrotic nipple with primary closure B ) One-stage debridement of the necrotic nipple and reconstruction of the nipple-areola complex C ) One-stage exploration and conversion to a free nipple graft D ) Release of all insetting sutures E ) Observation only
The correct response is Option D. Dry eye syndrome (dysfunctional tear syndrome) is a problem of tear deficiency and eye discomfort that may result in damage to the cornea. Combining upper and lower eyelid blepharoplasty relative to staged upper and lower eyelid surgery presents a greater risk for dry eye syndrome due to orbicularis oculi dysfunction after surgery. Women on hormone replacement therapy also have a higher risk for developing dry eye syndrome after blepharoplasty. Eye lubrication before and after surgery needs to be strongly considered. Patients with prior laser vision correction should wait at least 6 months before pursuing blepharoplasty because of the effects on corneal sensation and tear production. In this case, the patient's Lasik procedure 1 year prior would not impair her surgical outcome with upper and lower eyelid blepharoplasty. Chemosis is characterized by conjunctival swelling and irritation after blepharoplasty surgery, requiring lubrication after surgery. Some surgeons advocate treatment of this condition with steroid drops. Eyelid malposition is more likely in patients with a negative vector, defined as those with a maxilla that does not project beyond the orbital rim. It is also common in patients who have poor eyelid tone, diagnosed by snap test or evident as ectropion. Major adverse postoperative events such as bleeding and loss of vision are rare. Bleeding is more risky with uncontrolled hypertension. Smoking history, diabetes, and hypothyroidism are not directly associated with specific complications of blepharoplasty.
2013 A 50-year-old woman comes to the office for consultation because of upper eyelid dermatochalasis and lower eyelid bags. History includes hypothyroidism and type 2 diabetes mellitus. She underwent laser eye (Lasik) surgery 1 year ago. Medications include estrogen and thyroid hormone replacements as well as metformin. Upper and lower eyelid blepharoplasty is planned. This patient is at greatest risk for which of the following postoperative complications? A ) Bleeding B ) Blindness C ) Chemosis D ) Dry eye syndrome E ) Eyelid malposition
The correct response is Option E. The zygomaticofacial nerve provides sensory innervation to the lateral fat pad of the lower eyelid. Branches of the trigeminal nerve provide sensation to the face. The infraorbital nerve, the second branch of the trigeminal nerve, supplies innervation to the lower eyelid, cheek, and upper lip. The lateral palpebral branch of the lacrimal nerve, a branch of the infraorbital nerve, supplies sensory innervation to the superior lateral portion of the upper eyelid. The infratrochlear nerve provides sensory innervation to the medial aspect of the upper and lower eyelid. The lacrimal nerve provides sensation to the upper eyelid through the first branch of the trigeminal nerve. The facial nerve is a motor nerve to the face and is not responsible for sensation in the face.
2013 A 50-year-old woman undergoes upper and lower eyelid blepharoplasty with local anesthesia and intravenous sedation. The procedure begins with no patient discomfort; however, the patient reports marked pain once removal of the lower lateral fat pad is initiated. Which of the following nerves is the source of pain in this patient? A ) Facial nerve B ) Infraorbital nerve C ) Infratrochlear nerve D ) Lacrimal nerve E ) Zygomaticofacial nerve
The correct response is Option C. Anaplastic large T-cell lymphoma (ALCL) is a rare (1 per million) non-Hodgkin lymphoma that has been reported in women with and without breast prostheses. However, increasing case reports suggest an association with breast prostheses, although direct causation has not been established. ALCL associated with breast prostheses has malignant cells infiltrating the periprosthetic capsule or in the periprosthetic fluid collection. It is associated with both silicone- and saline-filled prostheses and seen in patients who have had prostheses for augmentation mammaplasty as well as breast reconstruction. Although the cytology is the same between ALCL associated with and without breast prostheses, ALCL that develops around prostheses tend to have an indolent clinical course and favorable prognosis when compared with systemic ALCL.
2013 A 53-year-old woman comes to the office because of unilateral swelling of the breast 5 years after undergoing subglandular augmentation mammaplasty. A diagnosis of anaplastic large T-cell lymphoma (ALCL) is established. Which of the following is most likely to represent the progression of this patient's disease when compared with a patient who has ALCL but no breast prostheses? A ) A more aggressive clinical course and a poorer prognosis B ) A more aggressive clinical course but a more favorable prognosis C ) A more indolent clinical course and a more favorable prognosis D ) A more indolent clinical course but a poorer prognosis E ) The same clinical course and prognosis
The correct response is Option E. A Ryan flap involves advancement of the lower thoracic skin and subcutaneous tissue in postmastectomy breast reconstruction. It is helpful for modest supplementation of prethoracic skin coverage and creation of a well-defined inframammary fold. The benefits of this maneuver include a good skin color match, ease of performance, and a scar that is confined to the inframammary fold area. A Ryan flap is not a global component of any of the breast reconstruction codes. The advancement flap procedure is reported separately. The code selected is based upon the surface area of the flap: 14001 (advancement flap, 10 to 30 cm2) or 14301 (advancement flap, 30 to 60 cm2).
2013 A 53-year-old woman comes to the office for symmetry revision of a previous breast reconstruction that requires a Ryan flap. The area of advancement is 15 × 4 cm. Which of the following is the most appropriate Current Procedural Terminology (CPT) coding for this procedure? A ) 14001 (Advancement flap, 10 to 30 cm2) B ) 15734 (Muscle, myocutaneous or fasciocutaneous flap; trunk) and 14301 (advancement flap, 30 to 60 cm2) C ) 19380 (Revision of reconstructed breast) D ) 19380 (Revision of reconstructed breast) and 14001 (Advancement flap, 10 to 30 cm2) E ) 19380 (Revision of reconstructed breast) and 14301 (Advancement flap, 30 to 60 cm2)
The correct response is Option D. Hyperpigmentation is the most common adverse effect of laser resurfacing. It occurs in 36% of patients and is most common in people with Fitzpatrick skin Types III to VI. Treatment consists of hydroquinone and tretinoin. Sun exposure should be avoided. Rates of hyperpigmentation can be reduced in those pretreated with retinoic acid and bleaching agents. While hyperpigmentation can be permanent, with proper treatment it usually resolves within a few months. Acne can occur post-laser treatment. It is especially common in patients with a prior history and should be treated with standard acne therapies. Infection risk from bacteria is minimized with prophylactic antibiotics and good topical care. Viral herpes simplex outbreaks can occur in those with and without a history. Antiviral prophylaxis is now used in all patients undergoing laser resurfacing. Yeast infections are also a possible infectious complication. These respond well to systemic antifungals. Scarring can occur with improper technique that causes excessive thermal damage (i.e., too many passes and excessive energy fluencies). Areas that develop scarring can be treated with topical and intralesional corticosteroids, silicone sheeting, and pulsed-dye laser. Erythema is not considered a complication and is a normal part of the healing process. It can last 1 to 4 months depending on the type of laser used.
2013 A 54-year-old woman comes to the office because of severe facial rhytides and photodamage. Examination shows Fitzpatrick skin type III. Ablative laser resurfacing is planned. Which of the following is the most likely complication of laser resurfacing in this patient? A ) Acneiform eruption B ) Bacterial infection C ) Erythema D ) Hyperpigmentation E ) Scarring
The correct response is Option C. The layers of the upper eyelid, in order from superficial to deep, are skin, orbicularis oculi muscle, retro-orbicularis oculi fat, orbital septum, orbital fat (central and nasal or medial in the upper eyelid), levator palpebrae superioris muscle and aponeurosis, Müller muscle, and the conjunctiva. In a Fasanella-Servat procedure for the correction of eyelid ptosis, the conjunctiva and Müller muscle are grasped between clamps just above the border of the tarsal plate. The tissue in the clamp is then excised and closed, thus resecting the Müller muscle and conjunctiva. At the base of the wound after the resection is the levator muscle. The capsulopalpebral fascia and Lockwood ligament are part of the lower eyelid and orbital contents. The capsulopalpebral fascia inserts on the inferior border of the tarsus. It makes up the anterior superior portion of the lower eyelid retractors distal to the Lockwood ligament. The Lockwood ligament is a fascial thickening that supports the globe. It surrounds the inferior rectus and inferior oblique muscles and fuses with the capsulopalpebral fascia. It is analogous to the Whitnall ligament in the upper eyelid.
2013 A 56-year-old woman undergoes resection of the conjunctiva and Müller muscle for treatment of blepharoptosis. Which of the following structures is encountered immediately anterior to the Müller muscle? A ) Capsulopalpebral fascia B ) Central fat pad C ) Levator palpebrae superioris muscle D ) Lockwood ligament E ) Retro-orbicularis oculi fat
The correct response is Option B. A patient who undergoes blepharoplasty and ptosis repair in the context of a compensated brow ptosis is likely to experience worsened brow ptosis after the procedure. Evaluation of the blepharoplasty patient requires careful examination of the entire upper third of the face. Patients may have, in addition to excess upper eyelid skin, an eyelid ptosis. In addition to identifying the ptosis, it is important to recognize compensated brow ptosis. A compensated eyelid ptosis occurs when the patient uses the frontalis muscles to raise the eyebrows, which results in a functional improvement in visual fields. This is most easily identified by having the patient close her eyes, and evaluate the automatic raising of the eyebrow on eyelid opening. In this case, the change in the position of the brow on downward gaze and on frontal gaze indicates a compensated brow ptosis. After ptosis repair and blepharoplasty, brow ptosis can become more manifest as the need for compensation decreases.
2013 A 57-year-old woman comes to the office because she is dissatisfied with the appearance of her eyes. She says they appear "small" and "tired." Physical examination shows dermatochalasis of the upper eyelids, 2 mm of eyelid ptosis, deep transverse rhytides of the forehead, and fine periorbital rhytides. She elevates her eyebrows 3 mm when she opens her eyelids. A skin-only blepharoplasty with formal eyelid ptosis repair is planned. After the procedure, which of the following clinical findings is most likely in this patient? A ) Blepharospasm B ) Brow ptosis C ) Decreased pretarsal show D ) Decreased volumetric convexity E ) Eyelid retraction
The correct response is Option D. Traditional brachioplasty comprises a T-shaped scar along the length of the arm and the axilla. This patient has excess skin on her arm, which dictates the necessity for an excisional procedure. Further liposuction of any modality will exaggerate her presentation further, including Vaser liposuction, a form of ultrasound-assisted liposuction, and laser-assisted liposuction, both of which are reported to tighten skin through heating its undersurface. There is no evidence to support Thermage as an effective treatment for lax skin. Lipobrachioplasty is a technique which safely combines liposuction with excisional surgery, performing liposuction on the proposed area to be excised. In a thinned arm, potential risks of neuropathy and seroma that accompany liposuction are unnecessary in a patient with lax skin that requires excision to improve contour. Limited incision brachioplasty with excision limited to an ellipse in the axilla, with or without liposuction, provides very limited results with regard to improving overall arm contour, particularly with moderate-to-severe skin redundancy.
2013 A 60-year-old woman comes to the office for evaluation of a poor outcome after liposuction of the arms. She says she is dissatisfied with the marked laxity of the skin of her arms. A photograph is shown. History includes gastric bypass surgery followed by a 100-lb (45.3-kg) weight loss. BMI is 28 kg/m2. Which of the following procedures is most appropriate to improve contour of the arm in this patient? A ) Laser-assisted liposuction B ) Lipobrachioplasty C ) Radiofrequency treatment (Thermage) D ) T-incision brachioplasty E ) Ultrasound-assisted liposuction
The correct response is Option E. Motor nerve dysfunction in the first few hours after surgery is common. This muscle weakness is attributable to the lingering effects of local anesthetic. For surgeons who inject the right and left sides of the face at different steps during the procedure, facial muscle asymmetry will be expected. Motor nerve dysfunction that is present days later is usually due to traction, cautery sutures, or transection. This patient underwent fat grafting to the mid face, and therefore, swelling is expected. Many surgeons routinely give intraoperative corticosteroids to decrease postoperative swelling. This patient does not need corticosteroids for her muscle weakness. Removal of the sutures and reexploration are not indicated.
2013 A 60-year-old woman is evaluated in the recovery room during the first hour after rhytidectomy, plication of the platysma, and malar fat grafting. On examination, the patient's mouth appears crooked when she speaks. Moderate diffuse swelling of the mid face and weakness of the right lower lip are noted. Which of the following is the most appropriate next step in management? A ) Administration of methylprednisolone B ) Consultation with a neurologist C ) Reexploration D ) Removal of the tension sutures E ) Observation only
The correct response is Option B. In patients wishing to avoid platysmaplasty or surgical neck lift, platysmal bands can be softened with neuromodulators. The platysma is a very superficial muscle, and injecting neuromodulators too deeply may affect the strap muscles, causing dysphagia, or the cricothyroid muscle, causing voice changes. While injection into the pars facialis just below the mandibular margin is safe, the cervicomental junction is considered a danger zone because of the potential effect on deeper muscles involved in swallowing. Injection inferior to the thyroid cartilage and centrally would not effectively treat the banding but a superficial injection into the fat is unlikely to cause any muscular disturbance. Injection along the inferior border of the mandible at the angle may affect facial nerve function but would not cause dysphagia or voice change. Injection at the medial margin of the sternocleidomastoid is unlikely in the treatment of platysmal banding. In a patient who presents with dysphagia or vocal changes after treatment of platysmal banding with neuromodulator, a reversible orally active anticholinesterase agent like pyridostigmine may be useful to counteract some of the effects until the agent wears off. Care must be taken to monitor for adverse side effects of anticholinesterase treatment, such as nausea, vomiting, diarrhea, and increased salivation.
2013 A 65-year-old woman comes to the office because of dysphagia and voice changes 3 days after undergoing injection of 50 units botulinum toxin type A because of platysmal banding. This patient's condition is most likely caused by injection of botulinum toxin type A to which of the following anatomical locations? A ) Central fat pad below the thyroid cartilage B ) Cervicomental junction C ) Inferior border of the mandible at the angle D ) Pars facialis below the mandibular margin
The correct response is Option B. This patient has a spinal accessory nerve injury related to the recent rhytidectomy. The spinal accessory nerve (XI) may potentially be injured as it passes through the posterior triangle of the neck. Iatrogenic injury is the most common cause of spinal accessory nerve dysfunction. Patients with injury to the spinal accessory nerve present with shoulder pain and trapezius muscle palsy that subsequently results in drooping of the shoulder girdle inferior and laterally along with scapular winging. The diagnosis is confirmed and the level of injury assessed with the use of electromyography and nerve conduction studies. Loss of spinal accessory motor nerve function due to neurapraxia should be managed conservatively, while the remaining patients with no sign of clinical or electrical recovery by 3 months should undergo evaluation for surgical exploration with neurolysis, repair, or grafting. Patients often have pain secondary to loss of the ability to suspend the shoulder girdle appropriately. Physical therapy with strengthening of the remaining scapular stabilizers, prevention of trapezius stretch/lengthening, and maintaining full range of motion of the shoulder girdle are important to good function after the nerve recovers. Shoulder splinting is of no benefit. The great auricular nerve is not involved in the patient's pathology, so nerve blocks would not be helpful. Similarly, MRI of the shoulder joint does not image the injured area, and does not help in patient management or surgical planning. Upper extremity angiography is of no benefit in the diagnosis or surgical planning of this disorder.
2013 A 65-year-old woman comes to your office because of pain and weakness in the left shoulder 3 months after undergoing cervicofacial rhytidectomy with a different practice. The pain began immediately after the procedure. She has no history of cervical spine disease, neuropathy, diabetes mellitus, rheumatoid diseases, or other trauma. Physical examination shows atrophy of the left trapezius muscle and left shoulder droop. She has full passive range of shoulder motion but limited active abduction. On attempts at active arm abduction, scapular winging is noted. Which of the following is the most appropriate next step? A ) Angiography of the upper extremity B ) Electromyography and nerve conduction study C ) MRI of the glenohumeral joint D ) Nerve blocking of the great auricular nerve E ) Shoulder splinting
The correct response is Option A. The World Health Organization defines successful breast-feeding as 6 months of exclusive breast-feeding. While several papers have examined the ability to breast-feed after reduction mammaplasty, the definition of successful breast-feeding varies greatly. In general, there exists consensus that neither pedicle type nor quantity of resected glandular tissue affected successful breast-feeding. Successful breast-feeding requires that the mammary lobules remain connected to the ducts and nipple after surgery; the free nipple technique precludes breast-feeding. The amount of gland that has to be preserved during reduction mammaplasty for maintaining the ability to breast-feed has not yet been determined. Many studies have shown that regardless of the type of full-thickness pedicle (inferior, superior, or medial) that is used for reduction mammaplasty, breast-feeding rates are equivalent to those for women who have not had reduction mammaplasty.
2013 A healthy 24-year-old nulliparous woman comes to the office for consultation regarding reduction mammaplasty because of back pain and macromastia. BMI is 26 kg/m2. She wears a 36 DDD brassiere. Which of the following is most likely to adversely affect the patient's ability to breast-feed after reduction mammaplasty? A ) Pedicle thickness B ) Pedicle type C ) Preoperative BMI D ) Preoperative breast cup size E ) Volume of resected tissue
The correct response is Option C. This is a case of inadvertent lidocaine toxicity with subsequent seizure and cardiac arrest. Furthermore, lipid emulsion has been used with apparent success early in the spectrum of local anesthetic systemic toxicity to preempt cardiac arrest. The role of lipid emulsion has expanded to treatment of cardiac toxicity due to other lipophilic drugs. Dantrolene is a treatment for malignant hypothermia. Dimercaprol is a chelating agent used for the treatment of heavy metal toxicities. N-acetylcysteine is used as a mucolytic and also in cases of acetaminophen overdose. Naloxone is used to treat narcotic overdose. There are no data to suggest that any of these medications are otherwise helpful in lidocaine toxicity.
2013 A previously healthy 38-year-old woman has onset of a brief tonic-clonic seizure 30 minutes after a lidocaine-based tumescent anesthesia is administered during large-volume liposuction of the abdomen, hips, and thighs. After 3 minutes, she has onset of asystole, and cardiopulmonary resuscitation is initiated. She is unresponsive to the standard ACLS resuscitation protocols for asystole. Administration of which of the following is the most appropriate next step? A ) Dantrolene B ) Dimercaprol C ) Lipid emulsion D ) N-acetylcysteine E ) Naloxone
The correct response is Option C. Placement of Mustardé sutures is the most common maneuver performed for a routine otoplasty and consists of horizontal mattress sutures placed between the scapha cartilage and the conchal cartilage in order to recreate the antihelical fold. Conchal setback sutures are also commonly used during an otoplasty and fixate the conchal cartilage to the mastoid fascia. Scapha cartilage would not be fixated to the mastoid fascia or else a significant pinning deformity would result. Occasionally, a wedge of excess conchal cartilage is excised to lessen the degree of conchal prominence in cases of excess. In this situation, conchal cartilage is sutured to conchal cartilage to close the resultant defect so that no contour irregularity is produced from the resection. This would not, however, have any effect on producing an antihelical fold. There is no indication for scapha cartilage to be fixated to scapha cartilage for routine otoplasty.
2013 An 8-year-old boy is scheduled to undergo otoplasty to correct prominence of the ears. Recreation of the antihelical fold, conchal setback, and lobule setback are planned. Mustardé sutures will be used to recreate the antihelical fold. The most appropriate location for placement of the Mustardé sutures in this patient is between which of the following structures? A ) Conchal cartilage and conchal cartilage B ) Conchal cartilage and mastoid fascia C ) Scapha cartilage and conchal cartilage D ) Scapha cartilage and mastoid fascia E ) Scapha cartilage and scapha cartilage
The correct response is Option A. This patient is likely suffering from a pulmonary embolism and requires emergent workup and treatment with anticoagulation. Although she was treated perioperatively with subcutaneous heparin, deep venous thrombosis (DVT) can still develop as well as a resultant pulmonary embolism (PE). In one survey of board-certified plastic surgeons, PE was found to be the leading cause of death following liposuction. In aesthetic plastic surgery patients, abdominoplasty patients appear to be at a higher risk of venous thromboembolism (VTE) when compared to those undergoing other commonly performed elective procedures. Rates range from 1 in 1000 to 1 in 300 for standard abdominoplasty, and when combined with other procedures, the risk of VTE and death from PE increases significantly. Any possible symptoms that are suspicious for PE, even several weeks postoperatively, must be taken very seriously and treated urgently with immediate presentation to the closest emergency department for aggressive and urgent medical workup as well as imaging and laboratory studies to evaluate for DVT and/or PE. Venous thrombus formation occurs secondary to a triad of factors describes by Virchow: venous stasis, vascular injury, and hypercoagulability. During abdominoplasty, the combination of general anesthesia, supine positioning, and immobilization promotes venous stasis. In addition, decreased venous return prevents clearance of activated clotting factors,further leading to thrombus formation. The highest risk period for fatal postoperative PE occurs 3-7 days after surgery, with approximately 10% of symptomatic PE being fatal within 1 hour of first symptoms. The risk of symptomatic venous thromboembolism is highest within 2 weeks of surgery and remains elevated for 2 to 3 months. Two thirds of patients with a DVT may appear clinically silent making the need for thromboprophylaxis extremely important. Current strategies to reduce risk include proper patient positioning, early ambulation postoperatively, flexion of the patient's knees 5 degrees, supplemented with mechanical (e.g., pneumatic compression boots) and/or pharmacological prophylaxis in most patients. The most common practice for pharmacological prophylaxis is to begin therapy preoperatively and continue treatment until 5 to 10 days after surgery. Some studies have even suggested a full month of postoperative treatment for those patients at a higher risk.
2013 An otherwise healthy 34-year-old woman calls the office because of nonspecific left-sided chest pain and mild shortness of breath 2 weeks after she underwent abdominoplasty with plication of a rectus diastasis and liposuction of the flanks. The procedure was performed during general anesthesia in 3 hours and 15 minutes. She was admitted overnight, and heparin was administered subcutaneously during her hospital stay, including one dose preoperatively. She reports no postoperative complications and feels extremely well otherwise. She says she is "finally getting back to herself physically" and does not want to worry her husband or children. She just wanted to make sure she had not "pulled a muscle or injured anything." Which of the following is the most appropriate action by the plastic surgeon? A ) Evaluation at the emergency department B ) Reassurance C ) Reduction of physical activity D ) Use of incentive spirometer at home to assist in improving pulmonary toilet E ) Use of over-the-counter anti-inflammatory drugs
The correct response is Option C. Component separation for closure of large abdominal wall defects was first described by Ramirez in 1990. The purpose of the surgery is to achieve abdominal wall closure with well-vascularized, innervated muscle flaps. The muscles of the abdominal wall are innervated by the intercostal nerves from T7-L4. The plane below the external oblique and above the internal oblique is an avascular plane that will allow for medial advancement of the rectus muscle flaps while protecting the intercostal nerves that run under the internal oblique.
2013 To maintain innervated muscle flaps during abdominal component separation, which of the following is the most appropriate plane of dissection? A ) Below subcutaneous fat pad, above fascia B ) Below fascia, above external oblique C ) Below external oblique, above internal oblique D ) Below internal oblique, above transversalis E ) Below transversalis, above peritoneum
The correct response is Option E. The external ear provides a versatile cartilage source for reconstructive procedures, especially for augmentative rhinoplasty. Short- and long-term morbidity associated with ear cartilage harvest using concha, tragus, and scapha as donor sites were evaluated. The relevant morbidity factors in the early postoperative period were hematoma formation (6.7%) and sensory impairment (3.3%). In long-term follow-up, sensory impairment was the most frequent condition objectively assessed and subjectively complained of (12.9%). Overall, sensory impairment was confined to concha as the donor site. Anthropometric measurements showed a mean difference in the length of the affected ear compared with the contralateral ear of 1.8 mm, a width difference of 2.5 mm, a difference in tragus/lateral canthus distance of 1.4 mm, and a difference in protrusion angle of 2.4 degrees. Aesthetically relevant complications were rare and their occurrence restricted to single cases. Long-term cases of necrosis, infection, and perichondritis were exceedingly rare. Cartilage graft harvest from the auricle can be considered as a relatively safe procedure with a favorable aesthetic outcome. Cauliflower ear may result from long-standing loss of blood supply to the ear cartilage and formation of neocartilage from disrupted perichondrium. This complication is almost never seen from harvest but from auricular trauma.
2013 Which of the following is the most common long-term complication of auricular cartilage harvesting? A ) Cauliflower ear B ) Hematoma C ) Necrosis D ) Perichondritis E ) Sensory impairment
The correct response is Option B. The septum, the caudal border of the upper lateral cartilage, the pyriform aperture, and the anterior border of the inferior turbinate define the internal nasal valve. It is the narrowest portion of the nasal airway and accounts for approximately 50% of nasal airway resistance. The entrance to the nostril is not an area of resistance in particular; however, the inner nostril can contribute to resistance particularly in the secondary rhinoplasty patient or a patient with weak lower lateral cartilages. This area is called the external nasal valve and is bounded by the caudal edge of the lateral crus of the lower lateral cartilage, the soft-tissue alae, the membranous septum, and the nostril sill. The majority of airflow in the nose is through the middle meatus. It exits through the choanae posteriorly into the nasopharynx. The choanae can be a source of resistance in the case of congenital choanal atresia where this region is blocked by bone or soft tissue. This would typically present shortly after birth. The keystone area is the junction of the bony and cartilaginous septum with the bony dorsum. It is a structural landmark and does not describe a region of airflow.
2013 Which of the following regions accounts for the most marked contribution to total nasal airflow resistance? A ) Choanae B ) Internal nasal valve C ) Keystone area D ) Middle meatus E ) Nasal alae
The correct response is Option D. The results of fat grafting are dependent upon the surgeon's experience and technique. Strategies to increase the take of the fat graft and decrease fat necrosis include atraumatic harvest techniques. Placement of the graft in multiple small aliquots increases the availability for vascularity and creating a lattice-like framework when depositing the grafts avoids large-volume deposits. Placement of the grafts in a few large-volume deposits is avoided to minimize fat necrosis and creation of fatty oil cysts. Fat grafting has been refuted for soft tissues that have been affected by radiation.
2013 Which of the following techniques minimizes fat necrosis and increases the viability of fat grafting? A ) Avoidance of pressure on grafted areas postoperatively B ) Avoiding placement of the grafts in irradiated tissue C ) Centrifuging the fat grafts D ) Placing the graft in multiple small volumes in a lattice-like framework E ) Using ultrasound-assisted liposuction to harvest grafts
The correct response is Option C. Patients often come to plastic surgeons with subcutaneous masses. An understanding of the differential diagnosis is helpful to counsel patients. Accessory breast tissue (polymastia) along the milk line is common. This breast tissue is responsive to hormonal influence, and patients will describe cyclical pain and swelling coinciding with menses and with milk letdown. Surgical excision can be performed. Polythelia is the presence of accessory nipples. Lipomas, epidermal inclusion cysts, and sarcomas can occur as subcutaneous masses, but they do not change in character based on hormonal influence.
2014 A 21-year-old woman comes to the office because of a lump 2 cm below the inframammary fold. She says she has "always had it," but it grew larger during pregnancy and has not decreased. The mass was painful during breast-feeding, and it is occasionally tender. Which of the following is the most likely diagnosis? A ) Epidermal inclusion cyst B ) Lipoma C ) Polymastia D ) Polythelia E ) Sarcoma
The correct response is Option D. Because of their small size, micrografts and minigrafts appear to have a lower metabolic requirement to thrive. They tend to grow in areas of fibrosis and burn scars and over skin grafts and flaps, including split-thickness skin grafts. The rate of survival and ultimate hair growth under these circumstances appears to be approximately 85%, compared with approximately 95% on unscarred, healthy tissue. More recently, follicular unit grafts have been used in hair transplantation.
2014 A 27-year-old man comes to the office for hair restoration to correct alopecia of the scalp. He sustained a burn injury to the scalp when he was a child and underwent split-thickness skin grafting to treat the burn. Micrograft hair transplantion to restore the hairline is planned. Which of the following is the most likely percentage of micrograft survival and ultimate hair growth in this patient? A ) 10% B ) 25% C ) 50% D ) 85% E ) 95%
The correct response is Option C. The breast receives its arterial blood supply from multiple sources, and this fact is used to design multiple pedicles for the nipple-areola complex that can work reliably for both mastopexy and reduction mammaplasty procedures. The superior pedicle receives its arterial blood supply primarily from the internal mammary branch from the second interspace. It is usually about 1 to 2 cm below the surface of the skin just medial to the breast meridian as it approaches the areola and may be localized with a handheld Doppler device during preoperative planning. The inferior pedicle and central pedicle designs are primarily supplied by branches of the internal mammary system from the fourth interspace. Additionally, there is some accessory input from the intercostal branches at the level of the inframammary fold with the inferior pedicle design. These secondary vessels are typically interrupted in a central pedicle operation. The medial pedicle design receives its arterial input mainly from the third superficial branch of the internal mammary artery. This vessel may be damaged by previous augmentation mammaplasty. The lateral pedicle design receives its arterial supply from superficial branches of the lateral thoracic artery.
2014 A 28-year-old woman is scheduled to undergo vertical mastopexy. She has no history of previous breast surgery. A superior pedicle technique is planned. Which of the following is the dominant blood supply for this pedicle? A ) Deep branches of the internal mammary artery from the fourth interspace B ) Deep branches of the internal mammary artery from the fifth interspace C ) Superficial branches of the internal mammary artery from the second interspace D ) Superficial branches of the internal mammary artery from the fourth interspace E ) Superficial branches of the lateral thoracic artery
The correct response is Option B. The combination of late-onset swelling without signs of periprosthetic infection (fever, cellulitis), no history of trauma, and a negative ultrasonography suggests late-onset seroma, as can occur with a double capsule phenomenon. Late seromas occur as a complication in about 1% of reported breast implant series. This issue seems to be more common in the setting of textured implants, particularly those implants manufactured with an aggressive texturing process. At surgery, a capsule layer is seen lining the pocket, which often contains a substantial volume of serosangineous seroma fluid and a textured implant coated in a tight second capsule at the center of the pocket. Double capsule has been reported in both the subglandular and submuscular positions. A giant fibroadenoma of the breast would have a dominant mass, distortion of the breast shape, and would be visible on ultrasonography. Abscess would be likely to occur with fever, chills, and cellulitis of the breast. Hematoma of this size would be likely to have a history of trauma, breast pain, and external bruising. Although anaplastic large cell lymphoma is a possibility in the differential of late-onset seromas, it is a rare disorder. Seroma fluid, obtained either by ultrasound-guided aspiration or at the time of open surgery, should be sent for cytologic examination and immunohistochemistry to rule out this rare possibility.
2014 A 30-year-old woman comes to the office because of a 3-week history of unilateral swelling of the left breast. She underwent subglandular placement of textured silicone breast implants 4 years ago. She has had no trauma, fevers, or chills. A 1-week course of an oral antibiotic prescribed by her family physician has failed to resolve the swelling. On physical examination, the left breast is 300 to 400 mL larger than the right breast. No other abnormalities are noted. Ultrasonography report shows seroma and results are negative for hematoma or mass. Which of the following is the most likely diagnosis in this patient? A ) Anaplastic large cell lymphoma B ) Double capsule phenomenon C ) Giant fibroadenoma of the breast D ) Hematoma due to capsule tear E ) Periprosthetic abscess
The correct response is Option B. It has long been realized that combination augmentation mammaplasty operations are more difficult and have a higher revision rate than either operation alone. A recent review of 177 primary augmentation mammaplasty cases found that, of the factors listed, preexisting breast ptosis and simultaneous mastopexy were both linked to a higher rate of reoperation when possible contributing factors were statistically analyzed. Furthermore, increasing grades of breast ptosis were linked with increasingly higher reoperation rates. Although incision site for augmentation mammaplasty has been markedly linked to the rates of capsular contracture, inframammary incisions have been shown in at least two studies to date to have the lowest rate of capsule formation, with periareolar and transaxillary incisions showing 5 to 10 times higher rates of capsule-related complications.
2014 A 30-year-old woman comes to the office for augmentation mammaplasty and mastopexy after a 50-lb (23-kg) weight loss. She wears a size 38B brassiere. Physical examination shows grade II ptosis and a sternal notch to nipple distance of 26 cm bilaterally. Simultaneous augmentation mammaplasty with short-T mastopexy using smooth saline-filled breast implants that will be implanted in a dual-plane configuration through an inframammary incision is planned. Which of the following factors puts this patient at highest risk for reoperation? A ) Inframammary implant insertion route B ) Presence of breast ptosis C ) Use of drains D ) Use of saline implants E ) Use of smooth-walled implants
The correct response is Option A. The cartilaginous structures of Asian noses are substantially different from those of Caucasian noses in terms of the shape, size, thickness, and relationship to other structures. The lengths of the upper and lower lateral cartilage of Asian noses are similar to those of Caucasian noses. However, the heights of the upper and lower lateral cartilage of Asian noses are shorter than those of Caucasian noses. Therefore, rhinoplasty with cephalic resection of the lower lateral cartilage in Asian noses should be approached with caution to prevent overresection. The overlapping length of upper lateral cartilage and nasal bone is similar in both Asians and Caucasians.
2014 A 32-year-old Korean man comes to the office for evaluation of a wide nose with decreased projection. He desires rhinoplasty for an improved aesthetic appearance. Compared with Caucasian nasal anatomy, which of the following is most likely in this patient? A ) Height of the lower lateral cartilage is shorter B ) Height of the upper lateral cartilage is longer C ) Length of the septal cartilage is longer D ) Length of the upper lateral cartilage is shorter E ) Overlapping length of the upper lateral cartilage and the nasal bone is shorter
The correct response is Option B. Fat embolization syndrome (FES) is clinically characterized by a triad of symptoms occurring within 24 to 72 hours following surgery or trauma: 1) alterations in mental status, 2) respiratory dysfunction, including hypoxemia or tachypnea, and 3) a petechial rash involving the anterior trunk, axillary, or head and neck regions. Common predisposing conditions include traumatic long bone and pelvic fractures, orthopedic procedures, liposuction, and soft-tissue injuries. Non-traumatic etiologies include pancreatitis, diabetes mellitus, osteomyelitis, and alcoholic liver disease. Proposed mechanisms for fat embolism include 1) mechanical disruption—release of fat droplets from disrupted bone marrow or adipose tissue forced into torn venules in areas of trauma, or 2) biochemical—release of free fatty acids as chylomicrons induced by systemic changes from trauma or sepsis. Diagnosis of FES is primarily clinical. Major diagnostic criteria include respiratory distress, cerebral dysfunction, and petechial rash. Minor criteria include tachycardia, tachypnea, fever, hypoxemia, thrombocytopenia, and hypocalcemia. Treatment for FES is primarily supportive. Respiratory support should focus on maintaining adequate tissue oxygenation and arterial saturation. Mechanical ventilation may be necessary. Hemodynamic and fluid resuscitation may be required. Use of corticosteroids remains controversial. The triad of symptoms including the petechiae concentrated in the upper truncal region would be an unusual occurrence for an allergic medication reaction. Lidocaine toxicity occurs initially with perioral numbness, vertigo, and visual In-Service Examination disturbances, and progresses to muscle twitching, unconsciousness, seizures, and finally to cardiorespiratory failure. Lidocaine toxicity would be earlier in onset and is not associated with a petechial rash. Although thrombotic thrombocytopenic purpura (TTP) may initially occur with petechiae, TTP is more commonly associated with headache, confusion, and digestive symptoms such as diarrhea, nausea, and abdominal pain. Transient ischemic attack may occur with temporary loss of vision, hemiparesis, confusion, or paresthesia.
2014 A 32-year-old man is brought to the office 24 hours after he underwent liposuction of the upper extremities, breasts, and chest. He appears disoriented and confused. Examination shows a petechial rash over the anterior trunk and axilla. Which of the following is the most likely cause? A ) Allergic reaction to a medication B ) Fat embolism C ) Lidocaine toxicity D ) Thrombotic thrombocytopenic purpura E ) Transient ischemic attack
The correct response is Option C. Previously published work using radiologic volumetric data analysis with fat grafting for cosmetic augmentation mammaplasty demonstrates volume retention over 6 months of 64 ± 11%. Such loss of breast volume may be attributable to an element of apoptosis, a reduction in adipocyte volume after transplantation and survival, or a reduction in the fluid content of the grafted slurry. In reality, all three of these factors are likely to contribute to volume reduction over time.
2014 A 32-year-old woman comes to the office because of capsular contracture of the right breast. She underwent bilateral augmentation mammaplasty with saline implants 9 years ago. Revision surgery using simultaneous implant exchange with fat grafting is planned. Which of the following is the most likely fat retention volume over a 6-month period in this patient? A ) 25% B ) 45% C ) 65% D ) 85%
The correct response is Option C. Augmentation mammaplasty with autologous fat transfer has become an increasingly popular option for patients desiring modest volumetric improvement. Despite its popularity, there is still some concern regarding its safety and efficacy. ASPS offered guidelines on fat grafting for reconstructive procedures of the breast in 2009. However, caution is recommended in the setting of cosmetic procedures because the impact on radiologic changes in follow-up is still uncertain to date. Fat necrosis is a nonspecific histologic finding most commonly resulting from surgery, trauma, or radiation therapy. It is common after fat transfer procedures, though often is clinically occult, and detected through follow-up mammography. The mammographic images of fat necrosis range from lipid cysts to findings that are suspected for malignancy such as clustered microcalcifications or spiculated masses. The most frequent mammographic finding in the breast parenchyma after augmentation mammaplasty with fat transfer is bilateral scattered microcalcifications followed by radiolucent oil cysts with or without microcalcification. Microcalcifications represent an evolution in the mammographic appearance of fat necrosis and are usually not present in early postoperative screening, but rather are a relatively late finding that is present months to years after the inciting trauma. It is imperative that radiologists distinguish between benign and suspected microcalcifications in order to minimize the number of postoperative biopsies and frequent follow-up imaging. Although round, spherical, punctuate, and diffusely scattered calcifications are typical of benign processes, cluster, branching microcalcifications can be indicative of a malignant process and should be worked up. For this 33-year-old patient with no baseline mammography and a suspected lesion within 6 months of the procedure, routine or short-interval mammographic screening is not appropriate. A biopsy of the suspected area is required, and this patient should undergo a core needle biopsy of the clustered microcalcifications of the right breast, while the more benign-appearing calcifications within the upper outer quadrants can be observed.
2014 A 33-year-old woman with no family history of breast cancer undergoes bilateral augmentation mammaplasty with 300 mL of autologous fat per breast. Six months later, she has onset of pain in the right breast. Mammography shows linear clustered microcalcifications in the lower inner quadrant of the right breast, small lipid cysts bilaterally with scattered dystrophic rod-like calcifications in the upper outer quadrants bilaterally, and heterogeneity of the pectoral muscles. Which of the following is the most appropriate next step in management? A ) Baseline mammography between ages 35 and 40 and yearly thereafter B ) Core needle biopsy of the bilateral upper outer quadrants C ) Core needle biopsy of the right lower inner quadrant D ) Repeat mammography at 6 months and 12 months E ) Repeat mammography in 1 year
The correct response is Option D. Symptoms of septal perforations include crusting along the septal defect, bleeding, and whistling. The whistling sound is due to the altered airflow pattern. Perforations can be caused by trauma, cocaine snorting, and infectious or inflammatory causes. In this patient, surgical trauma is the most likely cause. Treatments for symptomatic septal perforations include flaps and grafts. Asymptomatic perforations do not require treatment. The symptoms of internal nasal valve collapse, and submucous hematoma would be restricted airflow. "L" strut fracture or collapse would occur with an external deformity and not the symptoms described. Exposed conchal bone is caused by overresection of inferior turbinate mucosa. When performing a submucosal resection of the inferior turbinate, this would not occur with a submucous resection of the septum.
2014 A 35-year-old man comes to the office 4 weeks after undergoing open rhinoplasty and submucous resection of a deviated septum. He reports nasal crusting, bleeding, and a whistling sound from his nose. Which of the following is the most likely diagnosis? A ) Exposed conchal bone B ) Internal nasal valve collapse C ) "L" strut fracture D ) Septal perforation E ) Submucous hematoma
The correct response is Option B. The classic features of a tuberous breast deformity include a constricted base with a high inframammary crease and herniation of breast parenchyma into the nipple-areola complex producing a large-diameter areola. Variable extent of micromastia is associated as well as breast asymmetry. When a patient has a high and tight inframammary crease, this crease must be released to accommodate an implant and allow correction of the deformity. If this native crease does not fully expand, then a double bubble will occur. Over time, the lower pole skin stretches in response to the implant and this double bubble often improves spontaneously. The incidence of capsule contracture, hematoma, nipple-areola depigmentation, and rippling should be similar to a patient who undergoes periareolar augmentation/mastopexy without a tuberous breast.
2014 A 35-year-old woman with tuberous breast deformity is scheduled to undergo augmentation/mastopexy. A smooth, round, cohesive gel implant will be used. This patient is at higher risk for which of the following complications when compared with augmentation/mastopexy performed on a patient without a tuberous breast? A ) Capsule contracture B ) Double bubble C ) Hematoma D ) Nipple-areola depigmentation E ) Rippling
The correct response is Option D Among the complications that have been reported following brachioplasty are injuries to the sensory nerves that traverse the medial arm. The most likely nerve to be injured is the medial antebrachial cutaneous nerve. It perforates the antebrachial fascia at a fairly constant distance of 14 cm proximal to the medial epicondyle, putting it at a greater risk of injury than either the ulnar or median nerves, both of which lie deep to the brachial fascia. It has a variable relationship with the basilic vein, but this structure is never transposed during brachioplasty. There is a greater danger to nerve injury when the dissection is conducted deeper than the Scarpa's fascia, therefore dissection deep to the brachial fascia or harvest of the intermuscular septum is contraindicated. A layer of subcutaneous fat at least 1 cm thick should be left over the deep fascia to minimize injury to the sensory nerves in the arm. Placing the longitudinal incision slightly posterior to the bicipital groove decreases chances for injury to the nerve as well.
2014 A 37-year-old man is scheduled to undergo bilateral brachioplasty to correct redundant arm skin due to massive weight loss following gastric bypass surgery. Which of the following methods is most appropriate to minimize sensory cutaneous nerve injury during this procedure? A ) Design the longitudinal incision anterior to the bicipital groove B ) Divide the intermuscular septum while resecting skin and fat C ) Elevate and transpose the basilic vein before closure of the brachioplasty defect D ) Leave at least 1 cm of fat over the brachial fascia in the proximal arm E ) Maintain a sharp dissection plane deep to the brachial fascia
The correct response is Option A. The most common complication associated with liposuction of the arm is areas of overresection resulting in contour irregularities. Because such irregularities can be due to postoperative swelling, they should be treated conservatively for at least 6 months after surgery. Treatment may consist of fat grafting if contour abnormalities persist after 6 months. Early treatment may include lymphatic massage. Although hyperpigmentation is possible, it is more commonly seen in the medial thigh in association with ultrasound-assisted liposuction. Seromas are rare in the upper extremity. Injury to the ulnar nerve at the elbow is possible, but it has not been reported. Care should be taken to avoid the nerve in placement of the cannula. While lymphedema is possible, if liposuction of the arm is limited to the posterior aspect, where major lymphatic channels are avoided, it can be prevented. Transient postoperative swelling is expected.
2014 A 38-year-old woman comes to the office because of excess fullness of the proximal posterior arm. Pinch test shows a thickness of greater than 2 cm, and skin laxity is not excessive. Liposuction is planned. Which of the following is the most likely complication of liposuction of the upper extremity? A ) Contour irregularities B ) Hyperpigmentation C ) Injury to the ulnar nerve D ) Lymphedema E ) Seroma
The correct response is Option D. Hernia repair has evolved considerably in the past decade. There are a number of different types of mesh, which can be generally classified as synthetic mesh and biologic or biodegradable mesh. Biologic or biodegradable mesh can stretch with time, and are not as durable as synthetic mesh. They are, however, indicated in contaminated wounds or hernias at high risk for contamination, such as in patients with multiple medical comorbidities. Polypropylene mesh, because of its high rate of tissue ingrowth, is recommended in many cases. Mesh underlay has been shown to be superior to mesh overlay, and a retrorectus repair is the recommended repair of the American Hernia Society. Mesh overlay is unlikely to be sufficient to repair the defect; and the patient had a failed mesh repair. Component separation will not address the absolute tissue loss of the rectus abdominis resection. Anterior component separation has a higher risk of hernia formation than a retrorectus repair. A component separation with biologic mesh might be appropriate in a contaminated wound, but this patient does not have a contaminated wound.
2014 A 39-year-old woman is evaluated because of an abdominal bulge after undergoing resection of a 6 × 6-cm desmoid tumor 2 years ago through a lower transverse incision. A 10 × 10-cm area of left rectus muscle and some right rectus muscle were resected, and 2 cm of normal fascia and muscle remain inferior to the costal margin and superior to the anterior superior iliac spine on the left side. The defect was initially bridged with synthetic mesh only. Which of the following is the most appropriate next step in management of this patient's condition? A ) Anterior component separation with biologic mesh overlay B ) Anterior component separation with biologic mesh underlay C ) Biologic mesh overlay only D ) Retrorectus component separation with synthetic mesh underlay E ) Rives-Stoppa technique of mesh placement only
The correct response is Option A. Performing abdominoplasty in combination with other procedures has become a much more common request from patients. Advantages to this approach would include one recovery period versus multiple (which would minimize time away from work and or family, financial advantage to the patient, and a reduced need for multiple hospitalizations and exposure to anesthesia). Several studies have proven the safety of performing abdominoplasty combined with intra-abdominal procedures such as total abdominal hysterectomy (TAH). While the safety profile of combining these surgeries has been well proven, the risk of thromboembolic events is increased due to the extended time of surgery, so that aggressive deep venous thrombus prophylaxis must be administered perioperatively such as set forth by the Plastic Surgery Task Force on Deep Venous Thrombosis Prophylaxis. A TAH may be done via open or laparoscopic approach, depending on the gynecologist's preference or patient's chosen method. In addition, as robotically assisted gynecologic procedures have gained widespread acceptance, this approach for a TAH may also be combined with abdominoplasty surgery. Typically, if a laparoscopic or robotic method were chosen, the gynecologist would start the procedure; this way the port scars may be planned such that they can be excised within the abdominoplasty flap to be removed. Although the abdominoplasty flap may be raised off of the fascia before port placement, this usually causes the ports to be more unstable without the added support of the skin and soft tissues to properly hold them in place.
2014 A 39-year-old woman is referred for abdominoplasty. She has a history of severe uterine fibroids, and her gynecologist is planning a total abdominal hysterectomy (TAH). The patient would like to have the abdominoplasty and the TAH performed at the same time. Which of the following is the most appropriate response to this patient's inquiry? A ) Combining the surgeries can be done safely via any approach for TAH B ) The gynecologist may perform TAH, but the abdominoplasty flap must be raised first C ) The patient's risk of a thromboembolic event is decreased by having one large surgery D ) TAH cannot be combined with abdominoplasty
The correct response is Option E. It is well known that tamoxifen can increase the risk of thrombembolic events. In a retrospective study at MD Anderson Cancer Center, it was shown that patients who received the drug close to the procedure had a significantly higher rate of complications. It is recommended that the patient stop tamoxifen at least 28 days before surgery. Other listed medications do not have a direct effect on thrombotic complications.
2014 A 42-year-old woman who underwent mastectomy of the right breast 6 months ago is evaluated for delayed autologous breast reconstruction with free tissue transfer. Which of the following medications should be discontinued preoperatively if she is routinely taking it? A ) Diltiazem B ) Fluoxetine C ) Metoprolol D ) Multivitamin E ) Tamoxifen
The correct response is Option A. Among familial breast cancers, 5 to 10% are considered to be hereditary. Breast cancer susceptibility genes (BRCA) belong to a class of genes known as tumor supressors. In healthy cells, BRCA1 and BRCA2 genes stabilize DNA and prevent uncontrolled cell growth. A woman's lifetime risk of developing breast and/or ovarian cancer is greatly increased if she inherits a mutation on BRCA1 or BRCA2. BRCA1- and BRCA2-related breast cancers occur in younger women and are often associated with estrogen receptor negative tumors. Before undergoing elective breast procedures, women older than 40 years of age should undergo screening. The American Cancer Society (ACS) recommends annual screening mammography beginning at age 40. In addition, the ACS recommends that certain women undergo screening MRI with mammography. These include women with known BRCA1 or BRCA2 mutation, a first-degree relative with known BRCA mutation, 20 to 25% lifetime risk of breast cancer as determined by risk models, history of radiation to the chest between the ages 10 and 30, or specific cancer syndromes.
2014 A 43-year-old woman comes to the office for consultation regarding augmentation mammaplasty. She has a history of BRCA1 mutation and a sister with a history of breast cancer. Which of the following is the most appropriate screening in this patient? A ) Mammography and MRI B ) Mammography and ultrasonography C ) Mammography only D ) MRI only E ) Ultrasonography only
The correct response is Option C. Adjacent tissue transfer is one of the most common procedures performed by plastic surgeons, and accurate coding is essential for accurate reimbursement, insurance integrity, and ethical reasons. The most appropriate method for calculating the area is to add the area of the defect to the area of the flap. The area of the defect is considered the "primary defect," and the flap alone is considered the "secondary defect." It is the combination of these that determines the area on which the current procedural terminology (CPT) codes are based. In this case the defect measures 5 × 10 cm, or 50 cm2. The flap itself measures 20 × 30 cm, or 600 cm2. Therefore, the total area used to assign the correct CPT code is 650 cm2. The skin graft is over part of the secondary defect, which is already covered by the secondary defect measurement. However, the skin graft is an additional code that should be added to the codes for adjacent tissue transfer.
2014 A 45-year-old man is referred for scalp reconstruction after undergoing Mohs micrographic surgery for removal of squamous cell carcinoma of the scalp. The defect measures 5 × 10 cm. The scalp is reconstructed with a rotation flap measuring 20 × 30 cm with a 5 × 2-cm split-thickness skin graft on the secondary donor defect. When assigning a current procedural terminology (CPT) code for the adjacent tissue transfer, which of the following is the correct area to use? A ) 50 cm2 B ) 600 cm2 C ) 650 cm2 D ) 810 cm2 E ) 1000 cm2
The correct response is Option A. The incision for a retroseptal approach is usually placed 4 to 5 mm below the tarsal border, or about 8 mm for the lid margin. The plan of dissection for a retroseptal approach is, by definition, deep to both the orbicularis muscle and septum. Because the fat pads are retroseptal, modification does not require entry through the septum when a retroseptal approach is used. The preseptal approach is typically chosen for modification of the lid-cheek junction, and facilitates fat pad redistribution and access to the midface. The retroseptal approach is used for reduction of fat pads only. The inferior oblique muscle is located between the central and medial compartments of fat. Multiple authors have shown that is it indeed safe to perform skin resurfacing with either chemical peel or laser simultaneously with a transconjunctival blepharoplasty.
2014 A 45-year-old woman comes to the office because of puffiness of both lower eyelids. Physical examination shows little lower lid skin excess, but prominent eyelid fat pads. Lid tone, snap back, and distraction test results show no abnormalities. Lid-cheek junction is smooth. A retroseptal transconjunctival approach of the lower lid is planned. Regarding the operative approach, which of the following statements is most accurate? A ) Fat pad reduction without violating the orbital septum is performed B ) Lower eyelid skin resurfacing with laser or chemical peel should not be performed concurrently with a retroseptal approach C ) The incision should be placed 1 to 2 mm below the tarsal border D ) The inferior oblique muscle will be noted between the central and lateral fat pad E ) The plane of dissection is deep to the orbicularis, but superficial to the orbital septum
The correct response is Option D. Liposuction has been reliably shown to improve lymphedema post-breast cancer therapy. Lymphatico-venous bypass is a procedure that is gaining popularity, but the literature is still controversial with regard to its efficacy. The Homan and Charles procedures have become less popular due to drastically increased morbidity compared with liposuction. Brachioplasty is a technique to address excess skin of the upper arm and will not improve lymphedema. Resection approach, or debulking, involves surgical excision of subcutaneous tissue, which may or may not include excision of the overlying skin. Charles first described this resection method in 1912, and variations of this technique of radical excision of the subcutaneous tissue and primary or delayed skin grafting are still used today. Debulking procedures are not designed to directly address lymphatic vessel dysfunction but instead provide improved comfort by removing redundant skin and subcutaneous tissues. Homan described and popularized subcutaneous excision beneath flaps. This may be performed in two stages on a single extremity. A medial resection may be performed first, as more tissue can be removed from the medial than from the lateral aspect of both the arm and the leg. If necessary, a lateral procedure may be performed 3 months after the initial operation. If bilateral disease is present, the operation may be performed on both involved limbs during the initial procedure, although in cases of massive edema, the prolonged operative time and excessive blood loss mitigate against this approach.
2014 A 45-year-old woman treated with axillary dissection and mastectomy for Stage II breast cancer has onset of lymphedema of the right upper extremity. Conservative management with compression garments and lymphatic massage has not led to adequate resolution. Which of the following procedures is most appropriate to address the lymphedema in this patient? A ) Brachioplasty B ) Charles procedure C ) Homan procedure D ) Liposuction E ) Lymphatico-venous bypass
The correct response is Option B. Transient lagophthalmos during sleep is not uncommon following blepharoplasty. During the first few weeks of recovery, it is important to protect the eyes with lubricating drops and ointment. Bell phenomenon, an upward and outer movement of the eye when the eye is closed, is a protective mechanism which keeps the cornea protected behind the upper eyelid. Bell phenomenon is absent in 10 to 15% of the population. Accommodation reflex, oculocardiac reflex, ocular convergence, and pupillary light response do not place the cornea at risk after blepharoplasty.
2014 A 48-year-old woman comes to the office because of pain and tearing of the right eye 1 week after undergoing upper eyelid blepharoplasty. Fluorescein stain test result is positive for corneal erosion. On physical examination, absence of which of the following is most likely to put this patient at risk for corneal ulceration? A ) Accommodation reflex B ) Bell phenomenon C ) Ocular convergence D ) Oculocardiac bradycardia E ) Pupillary light response
The correct response is Option D. The most important diagnosis that needs to be ruled out is anaplastic large T-cell lymphoma (ALCL). This is a rare (one per million) non-Hodgkin lymphoma that has been reported in women with and without breast implants. However, increasing case reports suggest an association with breast implants, although direct causation has not been established. In ALCL associated with breast implants, malignant cells infiltrate the periprosthetic capsule or the periprosthetic fluid collection. The criteria for diagnosis include malignant cytology, strong CD30 expression, and cytokeratin negativity. Therefore, ultrasound-directed aspiration and cytology is most appropriate. Reduction of activity, wearing a support brassiere, and follow-up in 2 weeks would be appropriate for perioperative tissue edema but is not appropriate treatment for late seromas. Needle aspiration with Gram staining and culture risks injury to the implant and would not give the cytology necessary to determine if ALCL were present. Oral administration of antibiotics and complete blood count would not be warranted in this situation where infection is unlikely by history and physical examination. Ultrasound-directed aspiration and drain placement alone would not give the cytology necessary to determine if ALCL were present.
2014 A 51-year-old woman comes to the office because of unilateral swelling of the breast 1 year after subglandular silicone augmentation mammaplasty. The patient says she has not had any recent trauma, fever, or myalgia. Physical examination shows a periprosthetic fluid collection. No erythema or edema is noted. Which of the following is the most appropriate next step in management? A ) Needle aspiration, Gram staining, and culture B ) Oral administration of antibiotics and complete blood count C ) Reduction of activity and use of a support brassiere D ) Ultrasound-directed aspiration and cytology
The correct response is Option A. The deep inferior epigastric perforator flap would give the patient autologous tissue reconstruction with ample tissue for skin resurfacing and soft tissue for volume. In this radiated patient with a tight skin envelope, a tissue expander/implant, with or without acellular dermal matrix, would be difficult to create an appropriately ample skin envelope and the patient would be at higher risk for wound-healing problems and capsular contracture. The latissimus dorsi musculocutaneous flap, although an autologous tissue reconstruction, would have insufficient volume to adequately match this patient's contralateral side. It would have to be combined with an implant.
2014 A 52-year-old woman is evaluated for breast reconstruction after modified radical mastectomy with adjuvant chemotherapy and radiation therapy 18 months ago. Her last radiation treatment was 8 months ago. BMI is 29 kg/m2. Examination today shows hyperpigmentation of the right chest wall with no redundancy of the mastectomy skin flaps. Her contralateral breast is a D cup with grade III ptosis. Which of the following methods will create the best symmetry for this patient? A ) Deep inferior epigastric perforator flap B ) Gel breast implant and acellular dermal matrix C ) Latissimus dorsi musculocutaneous flap D ) Tissue expander and acellular dermal matrix placement with planned staged exchange for permanent gel implant E ) Tissue expander placement alone with planned staged exchange for permanent gel implant
The correct response is Option A. Abdominoplasty in combination with liposuction has been associated with a higher risk of complications. Combining both procedures has been reported to increase the risk of delayed healing, thrombotic emboli, fat emboli, skin necrosis, and fat necrosis. Increased complication rates are reported in patients with risk factors such as obesity, smoking, and diabetes mellitus. Direct undermining of the abdominal skin combined with liposuction can lead to vascular compromise of the overlying skin. The blood supply to the abdominal wall is divided into three zones: zone I, mid abdomen supplied by the deep inferior epigastric artery; zone II, lower abdomen supplied by the external iliac artery; and zone III, lateral abdomen and flanks supplied by the intercostal, subcostal, and lumbar arteries. Zone IV has not been described. Following abdominoplasty, elevation of the abdominal flap disrupts the blood supply from zones I and II, leaving the flap to be perfused by blood vessels from zone III. Liposuction with abdominoplasty of the central abdomen, zone I, is associated with the highest rate of skin necrosis. "Safe zones" include the flanks and upper lateral abdomen.
2014 A 53-year-old woman comes to the office for evaluation of abdominal skin redundancy. Physical examination shows muscle laxity and lipodystrophy of the central abdomen and flank regions. Abdominoplasty and liposuction of the flank and anterior abdominal skin are planned. After this combined procedure, the patient is at greatest risk for skin necrosis of which of the following abdominal zones? A ) I B ) II C ) III D ) IV
The correct response is Option C. Fat transfer to the breast remains a controversial procedure. There are some concerns about the oncologic safety of fat transfer, and for this reason some authors do not recommend fat transfer in patients with a history of cancer. Another concern about fat transfer is the potential difficulty in screening for malignancy. Rubin, et al. compared mammographic changes after fat transfer with changes after reduction mammaplasty. In this blinded study, radiologists reviewed pre- and postoperative mammograms of patients who had undergone augmentation and fat transfer and reduction mammaplasty. In the reduction cohort, masses requiring biopsy and scarring were more common; other abnormalities, including oil cysts, benign calcifications, and calcifications requiring biopsy showed no differences between the groups.
2014 A 53-year-old woman comes to the office for evaluation of breast asymmetry. Reduction of the left breast and augmentation of the right breast with implant and autologous fat transfer are planned. She is concerned about fat injection and cancer risk. Which of the following is the most appropriate response regarding mammographic changes after fat transfer? A ) Calcifications warranting biopsy are more likely on the fat transfer side B ) Calcifications warranting biopsy are more likely on the reduction side C ) Masses requiring biopsy are more likely on the reduction side D ) Scarring will be decreased on the reduction side E ) There are no differences between mammographic findings in fat transfer and reduction
The correct response is Option D. Signs and symptoms of lidocaine toxicity include dizziness, agitation, lethargy, tinnitus, metallic taste, perioral paresthesia, slurred speech, euphoria, hypotension, and bradycardia. Tachycardia is not a sign of lidocaine toxicity. Bradycardia is more common. Bronchospasm and urticaria are not signs of lidocaine toxicity.
2014 A 53-year-old woman comes to the office for removal of multiple nevi. On injection of lidocaine, which of the following signs and symptoms is most likely to suggest lidocaine toxicity in this patient? A ) Bronchospasm B ) Hypertension C ) Tachycardia D ) Tinnitus E ) Urticaria
The correct response is Option D. The most appropriate management for this condition is brachioplasty. Surgical management of the upper arm, particularly after massive weight loss, is dependent upon the ratio of fat and skin laxity. There are several classifications, but all address this ratio. Those with a great deal of skin laxity and little fat are best treated by direct excision (brachioplasty). Those patients who have little skin laxity (and good skin tone) and marked fat may benefit from liposuction alone. Those patients in the middle, with skin laxity and residual upper arm fat, are likely to benefit from a combination approach, either in a single or staged fashion. In this case, the patient demonstrates considerable skin laxity and has little extra fat, so a dermatolipectomy alone will address her deformity. A limited medial brachioplasty is useful for patients with skin laxity primarily in the proximal third of the arm and involves resection of a vertical ellipse of skin, leaving the scar in the apex of the axilla. The patient described here has skin laxity throughout the upper arm, which would not be adequately addressed with a medial brachioplasty. Liposuction followed by brachioplasty in a staged fashion will not improve her outcome and will only serve to increase the cumulative risks of two surgeries. Liposuction alone does not address the underlying issue of skin laxity. The patient's age and her history of obesity/massive weight loss severely decrease the ability of her skin to retract after liposuction.
2014 A 56-year-old woman comes to the office after gastric bypass surgery with a weight loss of 155 lb (70 kg). Weight is 143 lb (65 kg) and BMI is 24 kg/m2. She desires an improved appearance of the upper arms. A photograph is shown. Which of the following is the most appropriate surgical correction of this deformity? A ) Liposuction alone B ) Liposuction followed by brachioplasty C ) Limited medial brachioplasty D ) Full brachioplasty
The correct response is Option A. In a cervical branch facial nerve injury, lip depression can be weak, but the mentalis and orbicularis oris innervation remain intact, so that the patient would be able to purse her lips. Neither the cervical nor marginal mandibular nerves provide sensation to the lip. It would not be necessary to obtain a nerve conduction study in this case, because physical examination would be enough to distinguish between injuries to these nerves. The mid-face motor nerves would not be involved.
2014 A 58-year-old woman comes to the office because of muscle weakness of the lower face after undergoing rhytidectomy. Which of the following findings is most likely to indicate an injury to the cervical branch facial nerve in this patient, rather than the marginal mandibular nerve? A ) The patient can still purse her lips B ) The patient has lower lip depression weakness C ) The patient has lower lip numbness D ) The patient has mid-facial weakness E ) The patient has upper lip numbness
The correct response is Option A. Tissue sealants, such as fibrin tissue adhesives and platelet-rich plasma, have been utilized to affect drainage, ecchymosis, and edema following rhytidectomy. Prospective studies have demonstrated decreased rate of ecchymosis, edema, seroma, and prolonged induration. Although no major differences exist, studies have shown only a trend toward drainage reduction. Tissue sealants have not been shown to affect scar formation.
2014 A 59-year-old woman comes to the office for evaluation of rhytidectomy for facial aging. Use of a fibrin glue during rhytidectomy is planned. Which of the following is most commonly associated with use of tissue sealants after rhytidectomy? A ) Decreased ecchymosis B ) Decreased period of induration C ) Decreased scar formation D ) Increased drainage E ) Increased edema
The correct response is Option C. Wound-healing issues and skin necrosis should initially be managed with local wound care. In many cases, the wounds will go on to heal without negative sequelae. In the remainder of the cases, a corticosteroid injection or scar revision may be all that is necessary. Debridement of the region is not recommended because the eschar acts as a biologic dressing. Skin grafting would be indicated for a very large area of full-thickness necrosis. Re-advancement of the flap would not be indicated at this time as conservative management works well. Furthermore, re-advancement of the flap at this time would likely place too much tension on the closure with its resulting stigmata. However, re-advancement may be indicated at the time of scar revision once the wound has healed and the skin laxity has returned.
2014 A 62-year-old woman comes to the clinic for postsurgical assessment 2 weeks after rhytidectomy. In the right preauricular region, there is a 2 × 3-cm area of ischemic changes to the skin with a central eschar. Which of the following is the most appropriate next step in management? A ) Debridement of the eschar B ) Full-thickness skin grafting C ) Local wound care D ) Re-advancement of the flap E ) Split-thickness skin grafting
The correct response is Option E. Young faces appear full because of well-supported facial fat. As one ages, deflation of facial fat occurs more visibly in areas with a high density of retaining ligaments (e.g., lateral chin and malar area). This deflation in conjunction with an intact mandibular ligament gives rise to marionette lines. Injectable fillers can minimize these lines. The integrity of the mandibular ligaments causes the marionette lines to be more prominent as they limit the descent of facial fat. Attenuation of these ligaments would soften the marionette lines. Viscoelastic stretching refers to the properties of skin when placed under tension (i.e., the relaxation of skin tightness following rhytidectomy). Dermal thinning occurs throughout the face and contributes to wrinkles. Repetitive mimetic muscle contraction is thought to contribute to the depth of nasolabial folds and facial radial expansion. It may contribute to marionette lines close to the oral commissure but is not the major contributing factor.
2014 A 62-year-old woman is concerned that she has developed deep creases from the corner of her mouth to her chin (marionette lines). Which of the following is the most likely cause of these findings? A ) Attenuation of mandibular ligaments B ) Dermal thinning C ) Mimetic muscle contraction D ) Viscoelastic stretching E ) Volume deflation
The correct response is Option A. Hering law describes equal innervation to the eyelids in that the signal to the levator is the same despite the potential need for each eyelid to work independently. In the scenario described, the patient had obvious ptosis of the right eyelid, and the signal to raise the eyelids was strong. When the right ptosis was corrected, the signal to raise the eyelids decreased, and the more mild ptosis of the left eyelid was uncovered. To help avoid this problem, a Hering test or a patch test can be performed. The Hering test is performed by elevating the ptotic eyelid and observing whether the other eyelid becomes ptotic. A patch test is when the ptotic eyelid is covered for a period of time (usually 15 minutes) and then observed for whether the non-ptotic eye becomes ptotic. The key to both tests is to decrease the excessive signal to raise the eyelids. Horner syndrome includes ptosis of the eyelid, constriction of the pupil, and decreased sweating due to disease in the sympathetic system. This can be due to a tumor, congenital or iatrogenic. von Graefe sign is lagophthalmos in downgaze. This is related to Graves disease. Müller maneuver is the reverse of the Valsalva maneuver. After a forced expiration, an attempt at inspiration is made with closed mouth and nose, thereby creating negative pressure in the chest and lungs. This maneuver is used to find weakened areas of the airway. Todd paresis is focal weakness following a seizure. This can affect eye
2014 A 65-year-old woman comes to the office for follow-up 6 days after undergoing bilateral upper eyelid blepharoplasty and repair of the right levator aponeurosis. Preoperatively, the patient had bilateral levator excursion of 13 mm and 4 mm of ptosis of the right eyelid. No ptosis of the left eyelid was noted. Physical examination today shows 2 mm of ptosis of the left upper eyelid. The right upper eyelid is well positioned. Which of the following is the most likely explanation for these findings? A ) Hering law B ) Horner syndrome C ) Müller maneuver D ) Todd paresis E ) von Graefe sign
The correct response is Option D. The high superficial musculoaponeurotic system (SMAS) technique, as described by Barton, divides the SMAS transversely at the superior-most portion of the zygomatic arch. Anatomical studies show that the procedure can be performed safely, as the frontal branch of the facial nerve runs in close proximity to the periosteum of the zygomatic arch, not within the SMAS, in the zone of SMAS transection. The most common cause of facial nerve inactivity in this situation is related to the transient effects of local anesthesia injected during surgery. Reexamination once the effect of the local anesthesia has worn off is recommended. Intravenous corticosteroids, in a randomized controlled study, did not reduce facial edema, and would not benefit this patient. The diagnostic studies, neurology consult, and surgical reexploration are premature at this point in the patient's course, but may be helpful later if there is no evidence of return of nerve function.
2014 A 65-year-old woman undergoes rhytidectomy using a high superficial musculoaponeurotic system technique. On examination 1 hour postoperatively, the patient is unable to raise her right eyebrow. No other abnormalities are noted. The patient's family is anxious, and they want to call a neurologist. Which of the following is the most appropriate course of action at this time? A ) Administer intravenous corticosteroids B ) Consult a neurologist C ) Order nerve conduction studies D ) Reexamine the patient in 24 hours E ) Surgical reexploration
The correct response is Option A. Mycobacteria are ubiquitous in soil and water, and infections caused by these organisms can complicate aesthetic liposuction and autologous fat-grafting procedures. The postoperative symptoms include cellulitis, abscess formation, draining sinuses, and postoperative wound infection. Patients often do not have fever, chills, or other signs of systemic infection. While patients on immunosuppressive medications seem to be at higher risk, the problem also occurs in patients with healthy immune systems. More than 50% of patients will test negative for acid-fast bacilli, in addition to negative routine culture results. Polymerase chain reaction assay testing is now available for the most common species of nontuberculous mycobacterium, and is a useful rapid screening test for patients suspected of having this diagnosis. Bacterial infections (MRSA) would tend to show symptoms earlier, have systemic signs, and are usually easily cultured. Herpetic infections usually present with fluid-filled vesicles earlier in the postoperative course, and often have pain as a primary complaint. There is no history of previous injectable fillers, or implants, to suggest a biofilm-related infection in this patient. Mucocutaneous candidiasis is a chronic disease of the skin, nails, and mucosal surfaces.
2014 A 67-year-old woman comes to the office for evaluation of abscesses on her face 5 weeks after undergoing autologous fat grafting for augmentation of the cheek. Physical examination shows multiple erythematous nodules, areas of induration, and microabscesses on both cheeks. Temperature is 99.5°F (37.5°C). Liposuction donor sites show no abnormalities. A 1-week course of ciprofloxacin prescribed by the patient's family doctor failed to resolve the problem. Results of Gram stain and routine culture and sensitivity are negative. Which of the following is the most likely diagnosis? A ) Atypical mycobacterial infection B ) Herpes zoster infection C ) MRSA infection D ) Mucocutaneous candidiasis E ) Staphylococcus epidermidis infection
The correct response is Option D. Croton oil is mixed with phenol to create a deeper peel. These are part of the ingredients in the Baker-Gordon and Hetter solutions. Hetter's studies demonstrated that it was the croton oil that controlled the depth of the peel. The deeper the chemical peel, the greater the risk of scarring and hyperpigmentation. Because of this increased risk of hyperpigmentation, deep chemical peels are best suited for patients that have Fitzpatrick Type I skin. Chemical peels vary in their depth of penetration into the dermis. Superficial peels penetrate to the epidermis. Alpha-hydroxyl acids (glycolic and lactic acid) and beta-hydroxy acid peels (salicylic acid) are superficial peeling agents. Jessner solution (14 g resorcinol, 14 g salicylic acid, 14 mL of lactic acid, and 100 mL of 95% ethanol) is also a superficial peel that can be used in conjunction with a trichloroacetic acid peel to achieve a deeper and more uniform peel. Superficial peels affect the epidermis and dermal-epidermal interface. Twenty percent trichloroacetic acid is a medium-depth peel that penetrates into the papillary dermis.
2014 A 67-year-old woman with Fitzpatrick Type I skin comes to the office because of deep rhytides and signs of photoaging. Which of the following chemical peels will penetrate to the reticular dermis? A ) Alpha-hydroxy acid B ) Beta-hydroxy acid C ) Jessner solution D ) Phenol-croton oil E ) 20% Trichloroacetic acid
The correct response is Option A. Secondary rhytidectomy patients are typically older than primary rhytidectomy patients, and have been demonstrated to have more comorbid medical diseases. Hence, a more thorough preoperative medical evaluation is prudent for these patients. One study found that depression, necessitating the use of a selective serotonin reuptake inhibitor, was the most common comorbid disease, in one quarter of the secondary rhytidectomy patients studied. Hypertension was the second most common medical condition. In secondary rhytidectomies, less skin is typically excised, but often, more care with tailoring and insetting the skin is required. The skin and superficial musculoaponeurotic system (SMAS) thicknesses are typically thinner than at a primary, which can make surgical elevation of SMAS flaps more difficult. Sub-SMAS scarring, however, does not preclude careful and safe re-elevation of a SMAS flap. Finally, vascular compromise of the skin is less likely in a secondary case, due to the delay phenomenon following the primary procedure.
2014 A 68-year-old woman comes to the office with recurrence of laxity of the neck and lower face following an uncomplicated rhytidectomy 10 years ago. She does not smoke cigarettes. Which of the following is most likely? A ) Associated comorbid medical conditions are now more likely to be present B ) More skin will typically be excised during the second rhytidectomy than the first rhytidectomy C ) Superficial musculoaponeurotic system (SMAS) layers scarring now precludes the re-elevation of a SMAS flap D ) The thickness of the skin and SMAS layers would be comparable to those seen during the first rhytidectomy E ) Vascular compromise of the skin flap is now more likely
The correct response is Option D. The most likely cause of this patient's complication is inadequate attention to injection delivery of microaliquots of fat, leading to fat necrosis. Liposculpting, or liposuction and fat grafting, for buttock contour improvement is increasing in popularity and becoming a frequently performed procedure. Good results can be obtained, and patient satisfaction can be high. However, complications may also occur and should not be ignored. Proper technique is an essential component of effective liposculpting. Delivery of overly large amounts of fat into inadequate substrate can lead to inadequate revascularization and fat necrosis, described in the scenario as "tenderness, firmness, and drainage of cloudy, oily fluid." Superinfection of nonviable tissue can occur, creating "limited erythema" responsive to "a short course of oral antibiotics," but the most likely cause is not related to preoperative or postoperative antibiotics because the patient received what can be considered appropriate antibiotic therapy for a "clean," elective case. Neither the described 3:1 fat-to-fluid ratio nor the use of gravity fat separation is considered an inappropriate liposuction technique.
2014 A healthy 30-year-old woman undergoes autologous fat grafting of the buttocks. The patient receives a dose of antibiotics before incision with sterile skin preparation and draping. Tumescent liposuction is performed. Gravity separation of the fat is performed intraoperatively, and 350 mL of fat is reinjected into each buttock. Postoperatively, the patient reports some areas of discrete tenderness, firmness, and limited erythema. Incision and drainage shows oily, cloudy fluid. Which of the following is the most likely cause of this patient's complication? A ) Inadequate administration of antibiotics preoperatively B ) Inappropriate length of time for tumescent effect before aspiration C ) Lack of antibiotic administration postoperatively D ) Method of fat injection delivery E ) Use of gravity fat separation rather than centrifugation
The correct response is Option A. If treatment is initiated within the first few days of life, auricular molding can adequately and permanently treat some congenital ear deformities. Treatment must be initiated before 3 days of age and continue to 6 months of age. The efficacy of this mode of treatment is attributed to cartilage pliability due to high concentrations of circulating maternal estrogen in the first few days of life. Maternal estrogen concentrations are highest in the first 3 days of life and begin to decrease thereafter. In order to achieve good results with auricular molding, treatment must be initiated while maternal estrogen concentrations are increased and therefore cartilage pliability is high.
2014 A male newborn is evaluated in the hospital because of prominent ears. Nonsurgical correction with auricular molding is recommended. In order to achieve optimal correction, therapy should be initiated at which of the following ages? A ) 3 days B ) 14 days C ) 1 month D ) 6 months E ) 1 year
The correct response is Option A. The plastic surgeon described is board eligible, not board certified, and a Candidate Member of the ASPS. She cannot use the logo until she is a member of the ASPS, and she must first be board-certified. She cannot assume she has passed her oral board examination until the board contacts her to let her know this. Using the ASPS logo in advertising is a breach of the ASPS Code of Ethics without board certification and full membership in the ASPS. Claims highlighting the fact that she is a female plastic surgeon to attract female patients do not represent a breach of the Code of Ethics, nor is sharing her educational history. She has enlisted her patient's endorsement, and use of a direct quotation as a testimonial from the patient is fine if it is clearly a quote from the patient. The plastic surgeon needs to be careful, because other plastic surgeons in her area may think she is breaching the Code of Ethics with claims of superiority, as stated in Section 2IG: The member, either personally or through a partner or associate or any physician or other affiliated health care provider, uses or participates in the use of any form of public or private communication (including computer imaging and electronic communications) containing a false, fraudulent, deceptive, or misleading statement or claim.
2014 A plastic surgeon flies home after taking her oral American Board of Plastic Surgeons (ABPS) examination. She is excited to finally start advertising to the public in order to grow her practice. She enlists an endorsement from one of her satisfied patients who underwent augmentation mammaplasty. The patient provides a quote and allows her photograph to be used. The plastic surgeon also wants to highlight her additional fellowship training in breast surgery. She is an American Society of Plastic Surgeons (ASPS) candidate member. Inclusion of which of the following items in her advertisement would be a breach of the ASPS Code of Ethics? A ) ASPS logo B ) Assertion of advanced skills in breast surgery as a result of her breast fellowship C ) A list of her plastic surgery training programs D ) Testimonial of the patient stating that this is the best plastic surgeon she knows
The correct response is Option E. Gynecomastia, enlargement of the male breast, may occur as physiologic temporary overgrowth of the adolescent breast or it may appear during adulthood as a result of numerous etiologies. It can consist of various proportions of excess subareolar fibrous breast tissue and adipose tissue in the periphery, with the extent often depending upon the habitus of the individual. Appearing at about age 14 in more than 65% of healthy boys, gynecomastia will typically resolve within 2 years of onset in otherwise healthy males. Persistence of adolescent-onset gynecomastia after age 21 is unlikely to resolve with conservative measures. Hormone blockers have no place in the management of persistent breast tissue in otherwise healthy individuals. A wide variety of surgical treatment options for resection of redundant male breast tissue have been published. These include direct excision through a number of described incisions, traditional and ultrasound-assisted liposuction, and recently, use of an arthoscopic shaver. Ultrasound-assisted liposuction may have an advantage in stimulating skin retraction for cases where mild to moderate ptosis exists when scar minimization is desired. For otherwise uncomplicated gynecomastia, excision of fragmented subareolar tissue via a limited areolar border incision with the pull-through technique is the most appropriate choice of therapy. When redundant skin exists in addition to excess male breast tissue, immediate skin resection via periareolar, transverse lenticular, omega-shaped, or Wise (inferior pedicle) incision are among the numerous methods described. Regardless of the resection method, an evolving trend favors allowing skin to retract for 6 to 9 months before determining whether there is a need to subject the individual to additional incisional scarring.
2014 An otherwise healthy 27-year-old man comes to the office for removal of asymptomatic enlarged breasts that have persisted unchanged since onset at age 14. He takes no medications and does not use recreational drugs. Physical examination shows symmetrical collections of rubbery, firm subareolar tissue 4.5 cm in diameter. There is scant surrounding fatty tissue and no ptosis; areolas are 28 mm in diameter. Which of the following is the most appropriate management of this patient's condition? A ) Circumferential periareolar resection with liposuction B ) Mastectomy via Wise pattern incision C ) Observation to allow for involution D ) Oral hormone-blocking medication E ) Subareolar tissue ressection via pull-through excision technique
The correct response is Option A. Evidence-based data to confirm the validity of screening patients with silicone implants are lacking. In 2011, the FDA issued recommendations for physicians on the use of silicone gel-filled implants. Recommendations included providing copies of educational brochures, giving appropriate informed consent, maintaining medical vigilance, and reporting adverse events. It also suggested that patients undergoing augmentation mammaplasty get an MRI 3 years after implant placement and every 2 years thereafter. The purpose of these recommendations is not to replace routine cancer surveillance.
2014 An otherwise healthy 40-year-old woman comes to the office for augmentation mammaplasty. Mammography 6 months ago showed no abnormalities. Family history is negative for breast cancer. She wants to know if silicone gel implants are safe and what she should do after the procedure to monitor the implant for evidence of rupture. According to the current federal guidelines, which of the following is the most appropriate recommendation to this patient regarding surveillance? A ) MRI 3 years after implantation and every 2 years thereafter B ) MRI every 10 years C ) MRI if symptoms such as chronic myalgia and fatigue develop D ) Yearly mammograms E ) Yearly MRI
The correct response is Option E. The ASPS has a clinical guideline summary on reduction mammaplasty, based on the available evidence. Although insurance companies often use resection weight as a criterion for coverage, resection weight is not necessarily associated with relief of symptoms; thus, predictions of relief of symptoms must be made based on the individual clinical picture. This evidence is graded B. The risk of complications, however, does increase with resection weight (Grade B evidence). This risk of complications must be weighed against the potential for relief of symptoms with large resection weights. A distinction is made between resection weight and BMI. The ASPS guideline found only "inconclusive" data on the association between BMI and the risk of complications.
2014 An otherwise healthy 44-year-old woman comes to the office for reduction mammaplasty consultation. She wears a size 44DD brassiere. Physical examination shows rashes underneath the breasts, shoulder grooving from brassiere straps, and shoulder pain. A reduction is planned with removal of 500 g of tissue bilaterally. The woman reveals that her sister underwent reduction mammaplasty with a much larger resection size and questions whether she will have the same relief of symptoms. Which of the following is the most appropriate response? A ) Larger reductions are associated with less marked relief of symptoms B ) Larger reductions are associated with more marked relief of symptoms C ) Smaller reductions are associated with less marked relief of symptoms D ) Smaller reductions are associated with more marked relief of symptoms E ) Resection size is not associated with relief of symptoms
The correct response is Option E. The congenitally inverted nipple is common and occurs in 4% of infants. It results from the failure of the mesenchyme to proliferate above the level of the skin. Amastia is absence of the entire breast. Athelia is absence of the nipple. Amazia is absence of the mammary gland with an intact nipple and areola. Polythelia, or accessory nipple, results from failure of regression along the milk line.
2014 During fetal development, failure of the mammary pit to elevate above the skin level results in which of the following deformities? A ) Accessory nipple B ) Amastia C ) Amazia D ) Athelia E ) Nipple inversion
The correct response is Option C. Intercanthal distance most closely approximates orbital fissure width. Normal facial values are often described as proportions rather than absolute numbers. Many texts describe normal intercanthal distance as between 30 and 35 mm, but some studies have shown intercanthal distances of up to 40 mm in healthy cohorts. Thus, using the facial features as referents can be helpful. The face is often divided into fifths for analysis of width, and the intercanthal distance represents one fifth, as does the orbital fissure width. The nasal bone width is narrower than the intercanthal distance, and the eyebrow length extends lateral to the lateral canthus, representing greater than a fifth of the facial width. Subnasale, stomion, and menton distances are most often used to calculate facial height proportions. Although there is no reason why, theoretically, a measurement of facial height could not correspond to a measurement of facial width, these values do not.
2014 In a patient with facial proportions within the normal ranges, which of the following measurements best approximates intercanthal distance? A ) Eyebrow length B ) Nasal bone width C ) Orbital fissure width D ) Stomion-to-menton distance E ) Subnasale-to-stomion distance
The correct response is Option C. A subcutaneous fascia partitions the superficial subcutaneous facial fat. Anatomically, this fascia was recognized as early as 1799, when it was referred to as a cellular membrane. In 1859, Gray described the layer as the superficial subcutaneous fascia. In 1960, the usefulness of including the subcutaneous fascial layer in plicating sutures was noted. Later that decade, Tessier and Skoog, apparently working independently in France and Sweden, respectively, described the benefit of undermining and imbrication of this fascial layer in rhytidectomies. Residents from Tessier's unit then performed a number of anatomical studies to define the extent of the superficial subcutaneous fascia. Their classic anatomical study, published in 1976, described a superficial subcutaneous fascia that invested the platysma muscle and fused to the external surface of the parotid fascia. They named this fascia the superficial musculoaponeurotic system (SMAS). These findings have been corroborated by other authors, but the original study was not able to define the exact anterior extent of the SMAS. One of Tessier's residents later challenged this concept. He contended that there was no distinct parotid fascia and that the SMAS, rather than being an extension of the cervical investing fascia, was an embryologically distinct "primitive platysma." Controversy over the exact nature and extent of the SMAS persists. However, the consensus of surgical opinion seems to be that the SMAS represents the facial extension of the cervical investing fascia. As such, the SMAS envelops the platysma in the neck and cheek. Anteriorly, the SMAS becomes attenuated but terminates as the investing layer of the superficial layer of the mimetic muscles. Laterally, the SMAS fuses with the multilayer parotid capsule. Superiorly, the SMAS passes over the zygomatic arch to join the superficial temporal fascia (temporoparietalis and galea).
2014 The superficial musculoaponeurotic system invests the platysmal muscle and fuses to the external surface of which of the following? A ) Cervical investing fascia B ) Galea C ) Parotid masseteric fascia D ) Superficial temporal fascia E ) Temporoparietal fascia
The correct response is Option A. For modified radical and simple mastectomies, the landmarks of dissection are: superiorly to the clavicle, medially to the sternum, inferiorly to the inframammary fold, and laterally to the border of the latissimus dorsi muscle. The pectoralis major muscle fascia is resected with the specimen. The recreation of the inframammary fold is important for shaping in breast reconstruction and care must be taken to evaluate and repair both the inferior and lateral components of the inframammary fold.
2014 When performing immediate breast reconstruction, it is important to reconstruct the lateral inframammary fold. This is because the oncologic extirpation of the breast is carried out to which of the following anatomic landmarks? A ) Anterior edge of the latissimus dorsi muscle B ) Anterior edge of the serratus muscle C ) Lateral edge of the pectoralis major muscle D ) Lateral edge of the pectoralis minor muscle E ) Posterior edge of the serratus muscle
The correct response is Option C. Breast development is a complex interplay of multiple factors. Estrogen and progesterone play a significant role in breast development—not only at puberty, but also during and after pregnancy, and during and after menopause. In general, estrogen causes ductal proliferation, while progesterone causes glandular proliferations. Progesterone is similarly responsible for periductal stromal development. Estrogen increases, not decreases, cell division, and is also associated with increased, not decreased, fibrocystic changes.
2014 Which of the following best describes the role of estrogen in breast function? A ) Decreases cell division B ) Decreases fibrocystic changes C ) Facilitates ductal growth D ) Facilitates glandular growth E ) Facilitates periductal stromal development
The correct response is Option C. Tears are a trilaminar fluid. The precorneal layer is formed by mucin-secreting goblet cells in the conjunctiva. This inner layer of the tear film covers the cornea and promotes the dispersion of the overlying aqueous layer. The lacrimal gland secretes the middle layer. This aqueous layer is made of water and proteins. This layer promotes osmotic regulation and the control of infectious agents. The meibomian glands produce the outer lipid layer. This oil layer helps to prevent the evaporation of the tear film. As a result, dysfunction of the meibomian glands can lead to dry eyes.
2014 Which of the following is the function of secretions of the meibomian glands? A ) Coats cornea as inner layer of tear film B ) Lubricates eyelid skin C ) Prevents evaporation of tear film D ) Promotes control of infectious agents E ) Promotes dispersion of tear film
The correct response is Option D. Subjectively, the nasal valve can be assessed using the Cottle test. Anterior rhinoscopy is an objective way to evaluate the nasal cavity; however, the examiner's assessment of how much of the nasal cavity is obstructed or patent is subjective. Nasal endoscopy, CT scan, and MRI are described as tests capable to assess the nasal cavities, helping in the diagnosis of anatomical variations associated with nasal disorders. Objectively speaking, rhinomanometry is a dynamic way to assess nasal cavity patency and nasal function; it aims at establishing nasal resistance, which is the difficulty of passing air through the nose, through the measurement of transnasal pressure and airflow. Analysis of sound waves is a static way to assess nasal patency and geometry quantifying the areas of nostril cross section all the way to the nasopharynx and nasal cavity volume between the two cross-sectional areas chosen.
2014 Which of the following is the most appropriate method for demonstrating objective, dynamic nasal cavity patency and nasal function? A ) Anterior rhinoscopy B ) Cottle maneuver C ) Nasal endoscopy D ) Rhinomanometry E ) Sound wave analysis
The correct response is Option C. The only FDA-approved cosmetic indication for the use of onabotulinumtoxinA and abobotulinumtoxinA is to temporarily improve the appearance of moderate to severe glabellar lines. Although these products are widely used to treat other cosmetic concerns, these are all considered off-label uses. To this point, there have been no definitive adverse event reports of distant spread of botulinum toxin products when used at appropriate doses for dermatologic indications. There have been reports of adverse events in doses used for dystonia, especially in children. RimabotulinumtoxinB is only approved for treating cervical dystonia. On November 22, 2011, the FDA released a report renaming botulinum toxin type A and botulinum toxin type B to ensure their safe use (see table). Some of the reasons were to emphasize the differences in dosing and indications, and that these products are not interchangeable. The FDA recently approved application of Botox Cosmetic for smile lines related to activity of the lateral orbicularis oculi. Summary of FDA-Approved Botulinum Toxin Products Trade Drug Name Drug Name Indication Name* Botox OnabotulinumtoxinA Botulinum toxin type A Cervical dystonia, Severe primary axillary hyperhidrosis, Strabismus, Blepharospasm Botox OnabotulinumtoxinA Botulinum Temporary improvement in the Cosmetic toxin type A appearance of moderate to severe glabellar lines Dysport AbobotulinumtoxinA Botulinum toxin type A Myobloc RimabotulinumtoxinB Botulinum toxin type B Cervical dystonia, Temporary improvement in the appearance of moderate to severe glabellar lines Cervical dystonia * The marketed trade names and the product formulations have not changed.
2014 Which of the following is the only cosmetic indication for which onabotulinumtoxinA (Botox Cosmetic) and abobotulinumtoxinA (Dysport) have been approved by the US Food and Drug Administration? A ) Cervical dystonia B ) Forehead lines C ) Glabellar lines D ) Vertical lip creases
The correct response is Option E. In the patient described, the most appropriate surgical management is submucous septal resection. The important structures that affect nasal airflow and lead to obstruction are the internal and external nasal valves, the inferior turbinates, and the nasal septum. According to the classifications of the deviated nose, the patient described exhibits caudal septal deviation, with a concave (C-shaped) deformity of the septum. This is the most likely cause of this patient's obstruction symptoms. In the absence of internal (negative Cottle maneuver) and external nasal valve collapse, resection of the deviated septum is the maneuver most likely to improve the patient's nasal airflow and alleviate obstruction symptoms. It is of the utmost importance to preserve 9 to 10 mm L-strut of septal cartilage to maintain structural integrity. Alar batten grafts are placed in a pocket extending from the piriform aperture to a paramedian position in the alar sidewall. They prevent lateral nasal wall collapse and alar retraction during inspiration. They are also effective in providing strength and competency to the external nasal valves. The patient has competent external nasal valves, and thus, alar batten grafts alone will not address his nasal airway obstruction, which is caused by his deviated septum. Alar batten grafts are also used as an adjunct graft to correct a caudal septum deviation after septal resection. Columellar strut graft is placed between the medial crura for nasal tip shaping and support. Open rhinoplasty approach may cause mild loss of tip projection caused by disruption of ligamentous support and increased skin undermining, and a columellar strut will help maintain tip support. It can be used to increase nasal tip projection effectively. It does not play a role in alleviating airway obstruction caused by septal deviation. Septoplasty is the scoring of the quadrangle cartilage to influence its shape, in an attempt to straighten it. The cartilage will bend away from the scored surface. It is an important adjunct to septal resection to shapen and straighten a deviated septum. It is likely not powerful or predictable enough to correct a septal C-shaped deformity on its own and alleviate nasal obstruction. Spreader grafts are usually paired, longitudinal grafts placed between the dorsal septum and the upper lateral cartilages in a submucoperichondrial pocket. They are used to restore or maintain the internal nasal valve, straighten a deviated dorsal septum, improve the dorsal aesthetic lines, and reconstruct an open roof deformity. They are often placed in addition to septal resection. In the patient described, without internal nasal valve collapse and a deviated septum, spreader grafts alone will not likely improve nasal airway obstruction.
2015 A 30-year-old man comes to the office because of symptoms of nasal airway obstruction. Physical examination shows a septal C-shaped deformity without dorsal deviation; Cottle maneuver is negative, and external nasal valves are competent. Which of the following is the most appropriate surgical management? A ) Alar batten grafting B ) Columellar strut grafting C ) Septoplasty D ) Spreader grafting E ) Submucous septal resection
The correct response is Option D. The cumulative lifetime risk for a 35-year-old woman whose mother and sister had breast cancer is estimated to be approximately 15%. The risk may increase to as high as 45% if those cancers were premenopausal and bilateral. BRCA hereditary cancer is characterized by autosomal dominant genetics with multiple family members in each generation being affected. For patients with BRCA1 mutation, the risk of breast cancer has been estimated to be between 50 and 80% by age 65 years. The risk of developing ovarian cancer has been estimated to be 10% by age 60 years.
2015 A 35-year-old woman comes to the office for consultation regarding prophylactic mastectomy and breast reconstruction. The patient's mother and sister were diagnosed with bilateral breast cancer in their premenopausal years. Genetic testing for BRCA mutations is negative. Which of the following best estimates this patient's lifetime risk of breast cancer? A ) 5% B ) 13% C ) 20% D ) 45% E ) 80%
The correct response is Option C. The Regnault classification of breast ptosis is based on the relationship of the nipple to the inframammary fold (IMF) and to the lower contour of the gland. Pseudoptosis is the not true ptosis. In this situation, the nipple is above the level of the IMF but the breast parenchyma has descended below the IMF . Grade I is minor ptosis with the nipple at the level of the IMF and above the lower contour of the gland. Grade II is moderate ptosis with the nipple below the level of the IMF and above the lower contour of the gland. Grade III is major ptosis with the nipple below the level of the IMF and at the lower contour of the gland.
2015 A 37-year-old woman comes to the clinic to be evaluated for augmentation mammaplasty to improve her breast shape. She is gravida 3, para 3, and breast-fed all of her children. On examination, she has decreased superior pole volume, and the distance from nipple to sternal notch is 28 cm. The nipple-areola complex is below the inframammary fold by 4 cm and is at the lower contour of the breast. Which of the following Regnault classifications of ptosis best describes these findings? A ) Grade I B ) Grade II C ) Grade III D ) Pseudoptosis
The correct response is Option B. Early menarche is the highest risk factor for breast cancer of the options listed. Late first pregnancy, late menopause, no breast-feeding, and recent oral contraceptive use are also risk factors for breast cancer but are not as high risk.
2015 A 52-year-old woman receives a diagnosis of invasive ductal carcinoma of the right breast. Which of the following details from this patient's history is the strongest risk factor for this diagnosis? A ) Early first pregnancy (less than 30 years) B ) Early menarche (less than 12 years) C ) Early menopause (less than 55 years) D ) Multiple episodes of breast-feeding E ) Remote oral contraceptive use
The correct response is Option D. To achieve proper contour, the primary lifting suture should be placed on the vertical plane of the mid pupil. It is not uncommon for a patient with involutional ptosis to also have lateral displacement of the tarsal plate. If the central lifting suture is placed at the apex of the tarsal plate, the contour will be abnormally shifted laterally. The medial and lateral limbus position will also cause abnormal contour shifts. The midline of the central fat compartment has an inconsistent location in relation to the pupil.
2015 A 65-year-old woman desires correction of her "sleepy eyes." Physical examination shows bilateral moderate involutional ptosis of the upper eyelids. Mild lateral displacement of the tarsal plate is noted. Repair of the levator aponeurosis using an anterior approach is planned. Which of the following best describes the proper vertical plane to position the lifting suture in a single-suture technique? A ) Apex of the tarsal plate B ) Lateral limbus C ) Medial limbus D ) Mid pupil E ) Midline of central fat compartment
The correct response is Option B. Anabolic steroids have sex-specific adverse effects. Development of breast tissue in men, a condition called gynecomastia (which is usually caused by high levels of circulating estradiol), may arise because of increased conversion of testosterone to estradiol by the enzyme aromatase. Decreased sexual function and temporary infertility can also occur in men.
2015 Gynecomastia that occurs in men who take anabolic steroids is caused by increased levels of which of the following? A ) Aromatase B ) Estradiol C ) High-density lipoprotein D ) Testosterone
The correct response is Option A. The lateral breast is mobile, unlike the inframammary fold. Therefore, the footprint can vary slightly and be just behind the anterior axillary fold, but it does not extend to the midaxillary line. The breast footprint is not described as relating to either the latissimus dorsi or the clavicle.
2015 Which of the following represents the lateral border of the breast footprint? A ) Anterior axillary line B ) Anterior edge of the latissimus dorsi muscle C ) Lateral clavicle D ) Midaxillary line E ) Posterior axillary line
The correct response is Option D. Augmentation mammaplasty is known to have high rates of complications including reoperation. Infection, seroma, hematoma, and early implant rupture are rare in elective, cosmetic augmentation mammaplasty.
2016 A 35-year-old woman, gravida 2, para 2, seeks implant-based augmentation mammaplasty. She breastfed both her children. Which of the following is the most common complication of this procedure? A ) Early implant rupture B ) Hematoma C ) Infection D ) Lifetime need for reoperation E ) Seroma
The correct response is Option D. The safe dosage of lidocaine in liposuction is 35 to 55 mg/kg. Peak lidocaine levels are reported to be 8 to 18 hours after infiltration. Oftentimes, patients are discharged to home when peak levels occur. Surgeons should keep this in mind when calculating lidocaine dosage.
2016 A 48-year-old woman undergoes liposuction of the abdomen, flanks, and thighs. Liposuction is performed using 4 L of infiltration fluid. Each liter is mixed with 50 mL of 2% plain lidocaine and 1 mL of 1:1000 epinephrine. At which of the following times after infiltration are concentrations of lidocaine in the blood expected to be the highest in this patient? A ) Immediately after injection B ) 1 hour after surgery C ) 2 to 4 hours after surgery D ) 8 to 18 hours after surgery E ) 24 to 48 hours after surgery
The correct response is Option B. All of the answers are examples of benign breast disease. However, atypical lobular hyperplasia (ALH) is associated with an increased risk for breast cancer. Depending on other patient risk factors, chemoprevention with anti-estrogen medications and increased surveillance may be recommended.
2016 A 56-year-old woman undergoes bilateral reduction mammaplasty. Eight hundred grams per breast is removed and sent to permanent pathology. Which of the following results most likely requires further discussion with a breast surgeon? A ) Apocrine metaplasia B ) Atypical lobular hyperplasia C ) Fibroadenoma D ) Papillomatosis E ) Sclerosing adenosis
The correct response is Option C. A prospective, randomized, double-blind study involving human subjects compared the effects of laser-assisted lipoplasty with suction-assisted lipoplasty. No significant difference was noted between the two groups with respect to cosmetic outcome, ecchymosis, edema, skin retraction, or surgical time. The only measured potential benefit of the laser-assisted technique was an overall decrease in postoperative pain
2016 Compared with standard suction-assisted lipectomy, laser-assisted liposuction has been shown to decrease which of the following? A ) Contour irregularities B ) Ecchymosis C ) Postoperative pain D ) Skin necrosis E ) Swelling
The correct response is Option E. According to the Diagnostic and Statistical Manual of Mental Disorders, 5th edition, (DSM-V) the following criteria define the diagnosis of body dysmorphic disorder (BDD): • Preoccupation with one or more perceived defects or flaws in physical appearance that are not observable or appear slight to others. • At some point during the course of the disorder, the individual has performed repetitive behaviors (e.g., mirror checking, excessive grooming, skin picking, reassurance seeking) or mental acts (e.g., comparing his or her appearance with that of others) in response to the appearance concerns. • The preoccupation causes clinically significant distress and impairment in daily function. • An eating disorder may be seen with body dysmorphia, but is not pivotal to its diagnosis. A diagnostic criterion describing repetitive behaviors or mental acts in response to preoccupations with perceived defects or flaws in physical appearance has been added since the DSM-IV-TR, consistent with data indicating the prevalence and importance of this symptom. Prior history of cosmetic surgery (rhinoplasty in this case) is not part of the diagnostic criteria for body dysmorphic disorder.
2017 A 20-year-old woman comes to the office with her parents because of her significant concern with the appearance of a scar on her forehead of 1 year's duration. Physical examination shows a well-healed scar that blends in nicely with the surrounding skin and is difficult to see at conversation distance. The patient's parents do not see the need for any intervention since they also find the scar difficult to see. Which of the following must be present to confirm a diagnosis of body dysmorphic disorder in this patient? A) History of treatment for an eating disorder B) Occasional social anxiety C) Preoccupation with obvious flaws in her appearance D) Prior rhinoplasty E) Repetitive behavior related to her appearance concerns
The correct response is Option A. The auricular branch of the vagus (X) nerve (Arnold's nerve) innervates the external auditory canal and conchal area of the ear. This would not be blocked with a ring block. Direct infiltration of this area is needed. The lesser occipital nerve innervates the superior pinna. A branch of the glossopharyngeal nerve innervates the middle ear. The trigeminal nerve does not innervate the ear. The great auricular nerve innervates the lobule and the majority of the pinna.
2017 A 28-year-old man is evaluated after sustaining a laceration across the ear from the tragus to the antitragus. A ring block of the ear using 1% lidocaine is performed prior to repair. The patient reports pain when the conchal skin near the external auditory canal is sutured. Which of the following nerves is most likely NOT anesthetized in this patient? A) Auricular branch of the vagus (X) nerve B) Branch of the glossopharyngeal (IX) nerve C) Great auricular (C2, C3) nerve D) Lesser occipital (C2, C3) nerve E) Trigeminal (V2) nerve
The correct response is Option E. Liposuction of the central supraumbilical flap may further impair blood supply to the area farthest from the remaining blood supply after undermining and is the most likely of the choices to cause wound healing complications. Extensive undermining over the hips is usually well tolerated and a component of most abdominoplasties. Central rectus plication, if too tight, may contribute to respiratory difficulty or umbilical loss due to constricted blood supply, but it is unlikely to directly relate to poor healing of the midline incisional skin and fat. Liposuction of the bilateral mid flanks (in contrast to the central supraumbilical flap) is usually well tolerated as it allows preservation of the blood supply traveling from the costal region into the flap. While wide undermining over the costal margins may promote poor wound healing of the advanced tissue at the superior aspect of the incision, tunneling or discontinuous release aimed at perforator preservation is a described technique to increase the ability to contour the upper abdomen without concomitant significant increase in wound healing complications.
2017 A 28-year-old woman suffered a wound dehiscence after an abdominoplasty. History includes a 70-lb (32-kg) weight loss over the past 5 years through diet and exercise. She is angry and wants to know why the separation happened. Which of the following is the most likely contributor to the development of this complication? A) Central rectus plication B) Discontinuous release of tissue over the costal margins C) Extensive undermining over the hips D) Liposuction of the bilateral mid flanks E) Liposuction of the central supraumbilical flap
The correct response is Option D. Postoperative cerebrospinal fluid (CSF) leak is a rare but known complication following septoplasty. It is related to an error in surgical technique, with overly forceful manipulation of the perpendicular plate region resulting in a cribriform plate defect. The cardinal symptoms are frontal headache and a clear, watery persistent rhinorrhea. If nasal packing is present, the patient may report a metallic or salty-tasting post-nasal drip. Prompt diagnosis is required to avoid complications, particularly meningitis and pneumocephalus. CSF rhinorrhea is more common on the right side, reflecting a predominance of left-sided surgical approaches. It may present in an early manner, as in this case, or have a delayed presentation; some documented reports have a 20-year delay between septoplasty and diagnosis of CSF leak. While imaging is beneficial, initial diagnosis of CSF leak can be made with the beta-2 transferrin or Beta-trace protein testing, which are both specific and sensitive for CSF. Both are more accurate than the traditional "halo" sign or measuring the glucose level of the fluid. The other options listed are incorrect, as they do not diagnose or effectively treat the underlying problem.
2017 A 30-year-old man comes to the office because of a frontal headache and persistent watery drainage from the right nostril 2 weeks after undergoing septorhinoplasty. Which of the following is the most appropriate next step in management? A) Place nasal packing for 48 hours B) Start oral antihistamines C) Start vasoconstrictor nasal spray D) Test nasal discharge for beta-2 transferrin E) Reassure the patient that these symptoms are normal
The correct response is Option A. In the 2005 prospective, multicenter trial of 179 patients by Cunningham, Gear, Kerrigan and Collins, reduction mammaplasty had an overall complication rate of 43%. The most common complication was delayed wound healing (21.6%), followed by spitting sutures (9.2%), hematoma (3.7%), nipple necrosis (3.6%), hypertrophic scars (2.5%), fat necrosis (1.8%), seroma (1.2%), and infection (1.2%). Delayed wound healing correlated directly with average preoperative breast volume, average resection weight per breast, and smoking; and inversely with patient age. In this study, vertical techniques had a higher overall rate of complications. In the Stevens, et al. report of their 11-year experience with outpatient breast reduction, delayed wound healing was also the most common complication.
2017 A 30-year-old woman with symptomatic macromastia is scheduled for a Wise pattern breast reduction. Which of the following postoperative complications is most likely in this patient? A) Delayed wound healing B) Fat necrosis C) Hematoma D) Hypertrophic scarring E) Tear-drop deformity of the nipple-areola complex
The correct response is Option A. Hair shaft loss is to be expected after the first month following implantation as the hair follicles enter into the telogen phase. Initially, hair grafts enter the catagen phase of hair growth, which is a transitional phase. Implanted hair can grow 3 to 4 mm at this time. Following this stage, implanted hair grafts enter the telogen phase or the "resting" phase, which lasts for 3 to 4 months. During this phase, the attachment of the hair at the base of the follicle becomes weaker until the hair finally sheds, and the hair follicle is inactive and hair growth ceases. Almost every implanted hair falls out during this time, and patients must be prepared for this preoperatively. This phase can last up to 6 to 9 weeks, and this is when patients worry most about whether the surgery was indeed a success. Following this time, the anagen phase begins, generating new hair growth, which is initially quite thin. This hair then grows thicker with time. Final results of hair transplantation are not evident until 6 to 8 months in men and 10 to 12 months in women.
2017 A 34-year-old man comes to the office for a routine follow-up examination 1 month after undergoing extensive micrograft hair transplantation. He is extremely upset, complaining that "all the grafts have fallen out and are gone." Which of the following is the most appropriate response to this patient's concerns? A) Loss of hair shaft is expected and part of the normal healing process B) Loss of the grafts is a potential complication that was explained preoperatively C) More grafts will be performed without additional surgical fees D) The patient should wait and see what happens and return to the office in several months E) The patient will be started on finasteride immediately
The correct response is Option E. Although safe doses of lidocaine in tumescent solution can reach 35 mg/kg, lidocaine toxicity is still a risk. Peak plasma levels of lidocaine, when injected into fatty tissue, occur 10 to 14 hours after infiltration. Lidocaine toxicity symptoms include neurologic or cardiac toxicity. In the early stages, the complications are primarily neurologic, and can include slurred speech, restlessness, tinnitus, and a metallic taste, as well as numbness of the mouth. As the concentrations increase, the neurologic concentrations become more severe, and can progress to muscle twitching, seizures, and cardiac arrest. Treatment of lidocaine toxicity is supportive. A petechial rash of the neck and axillae is a typical sign of fat embolism, and tachypnea is a typical sign of a pulmonary embolism. Anisocoria can be a benign finding, although if it is a new finding, it is suggestive of Horner syndrome or a brain aneurysm. Supraventricular tachycardia (SVT) refers to tachycardia that occurs above or at the atrioventricular node. Paroxysmal SVT, atrial fibrillation, and Wolff-Parkinson-White syndrome are the most common forms of SVT. SVT can cause syncope and long-term cardiac damage.
2017 A 35-year-old woman undergoes suction-assisted lipectomy of the bilateral arms using a tumescent technique. A total of 500 mL of aspirate is recovered. She is discharged home the same day, and returns to the emergency department 8 hours after discharge. Lidocaine toxicity is suspected. Which of the following symptoms is most likely in this patient? A) Anisocoria B) Petechial rash of the neck and axillae C) Supraventricular tachycardia D) Tachypnea E) Tinnitus
The correct response is Option C. Lipoaspirate volumes over 5000 mL are what are widely considered to define "large volume" liposuction. This distinction has relevance as it relates to postoperative care. Patients undergoing "large volume" liposuction are often considered for overnight observation as an inpatient admission. The other volumes (1000 mL, 2500 mL, and 7500 mL) are not the cutoffs for consideration as "large volume" liposuction.
2017 A 36-year-old woman with a BMI of 35 kg/m2 undergoes liposuction of the anterior trunk, posterior trunk, and circumferential thighs. Which of the following is the minimum volume of lipoaspirate that can be classified as "large volume" liposuction? A) 1000 mL B) 2500 mL C) 5000 mL D) 7500 mL
The correction response is Option E. A dual-plane approach is subpectoral in the superior pole and subglandular in the inferior pole. This is to afford more subcutaneous coverage in the superior pole. Tebbetts recommends pinching the skin and subcutaneous tissues of the superior pole for a "pinch test." For thickness less than 2 cm, he recommends a dual-plane placement for adequate soft tissue coverage. This is not affected by the grade of ptosis, need for mastopexy, history of loss of superior pole volume, or nipple to notch distance.
2017 A 36-year-old woman, gravida 3, para 3, comes to the physician because she desires larger breasts. She has breast-fed three children. Physical examination shows grade 3 ptosis and loss of superior pole volume. The distance from nipple to sternal notch is 26 cm. Result of upper pole pinch test is 1.5 cm. A dual-plane augmentation/mastopexy is planned. Which of the following is the strongest indicator for subpectoral placement of the implant in this patient? A) Concurrent mastopexy B) Grade 3 ptosis C) Loss of superior pole volume D) Nipple to sternal notch distance of 26 cm E) Pinch test result of 1.5 cm
The correct response is Option B. The lateral cutaneous branch of the fourth intercostal nerve is most commonly responsible for nipple and areola sensitivity. The other intercostal nerve branches listed do contribute to breast sensitivity but are less often thought to be the primary innervation to the nipple and areola. The intercostobrachial nerve supplies innervation to the upper medial arm.
2017 A 38-year-old woman reports decreased areola sensitivity after undergoing mastopexy. Intraoperative injury to which of the following nerves is the most likely cause of this patient's reduced sensitivity? A) Intercostobrachial nerve B) Lateral cutaneous branch of the fourth intercostal nerve C) Lateral cutaneous branch of the sixth intercostal nerve D) Medial cutaneous branch of the fifth intercostal nerve E) Medial cutaneous branch of the third intercostal nerve
The correct response is Option C. The medial antebrachial cutaneous (MABC) nerve arises from the medial cord of the brachial plexus and innervates the medial arm and forearm. Distal to the axilla, the MABC nerve travels with the basilic vein. Anatomic studies have shown that the nerve penetrates the deep fascia approximately 14 cm proximal to the medial epicondyle relatively consistently. It is vulnerable during brachioplasty because of this position. The recommended technique to protect the medial antebrachial cutaneous (MABC) nerve is to leave a 1-cm cuff of fat overlying the deep fascia. A smaller cuff is inadequate to protect the nerve. In cases where the brachioplasty dissection must extend distal to the elbow, one can minimize injury to the nerve by maintaining a dissection plane superficial to the deep brachial fascia and transitioning to the subcutaneous plane below the elbow. A skin-only resection is unlikely to adequately correct the defect. Dissecting adjacent to, or deep to, the subcutaneous fascia places the nerve at risk because of the depth of the dissection.
2017 A 42-year-old woman comes to the clinic because of numbness of the right arm that extends from the mid arm to the medial aspect of the forearm to the wrist 5 weeks after undergoing bilateral brachioplasty. Which of the following operative techniques is most likely to decrease the risk for this numbness while adequately correcting the deformity? A) Dissecting adjacent to the muscle fascia throughout the length of the arm B) Dissecting deep to the muscle fascia throughout the length of the arm C) Leaving a 1-cm cuff of fat overlying the deep fascia throughout the length of the arm D) Performing a skin-only resection
The correct response is Option B. Minigrafts consist of 3 to 4 follicular units and are commonly used in concert with micrografts which contain 1 to 2 follicular units. Larger numbers of follicular units have been associated with artificial appearing outcomes (plug look).
2017 A 45-year-old man comes to the office for consultation regarding hair loss concerns. He reports that he has a friend who underwent a hair transplantation procedure that looks unnatural, especially at the front hairline. Using the minigraft and micrograft strategy, how many follicles should be in a minigraft? A) 1 to 2 B) 3 to 4 C) 5 to 6 D) 7 to 8 E) 9 to 10
The correct response is Option E. The deep white frost indicates the endpoint for the depth of skin penetration with a deep rhytid chemical peel such as the combination Jessner/35% trichloroacetic acid solution. This indicates that the peel has penetrated into the upper reticular dermis. There is no pink hue because at this level there is vasospasm of the capillaries in the papillary dermis. This depth of penetration is for moderate and deep rhytides. This is a transient phenomenon. Capillary refill should return within 20 to 40 minutes. A transparent frost with a pink background is the endpoint for a superficial peel, such as that done with a trichloroacetic solution (<30%) alone. This frost is due to the coagulation of proteins in the dermis and epidermis. The pink hue emanates from blood vessels that remain intact in the papillary dermis. A grey hue indicates a deeper penetration with destruction to the dermis. This level of penetration could lead to abnormal healing, scarring, pigmentation, and texturing changes. Hypopigmentation and superficial epidermolysis are potential complications of a chemical peel and not clinical endpoints to determine depth of penetration.
2017 A 45-year-old woman comes to the office because of deep rhytides caused by photoaging. Topical application of 35% trichloroacetic acid in combination with Jessner solution is planned. Which of the following best describes the clinical endpoint during application of this chemical peel? A) Dark firm eschar B) Grey hue C) Hypopigmentation D) Transparent frost with a pink background E) Uniform deep white frost
The correct response is Option B. Breast ptosis is a complex interaction of events, informed by breast size, gravity, aging, lactation, and parity. It occurs through a combination of atrophy of the breast tissue, loss of elasticity of the skin envelope, and attenuation of Cooper ligaments. While the breast is surrounded by fascia, the continuation of Scarpa's fascia forms the posterior capsule of the breast. The lactiferous ducts and breast acini do not contribute significantly to ptosis. Subdermal plexus provides vascularity rather than support to the breast.
2017 A 47-year-old woman, gravida 2, para 2, who has grade III breast ptosis is evaluated for mastopexy. Attenuation of which of the following structures is the most likely cause of the ptosis? A) Breast acini B) Cooper ligaments C) Lactiferous ducts D) Scarpa's fascia E) Subdermal plexus
The correct response is Option A. Because of the restrictive and malabsorptive components of the procedure, iron deficiency anemia is very common in post-bariatric patients. Iron deficiency occurs in 30 to 50% of post-bariatric patients despite a recommended daily multivitamin. Roux-en-Y gastric bypass promotes weight loss through both a restrictive component and a malabsorptive component. This has implications for micronutrient absorption, because pancreatic release of enzymes is no longer synced with gastric emptying, and factors involved in the processing and absorption of micronutrients are decreased. Micronutrient deficiencies are common after bariatric surgery, and while many are asymptomatic, they can have effects on wound healing after surgery. Vitamin B12 deficiencies are most common at least 12 months after surgery and range from 3.6 to 37%, but they depend on chronicity, degree of supplementation, and type of bypass. Brolin, et al, in 2002, compared patients with a distal Roux-en-Y gastric bypass to those with short Roux limbs (150 cm and 50 to 75 cm). Vitamin B12 deficiency was most common in patients who underwent Roux-en-Y gastric bypass, and B12 deficiency was most common after surgery with short limbs (50 to 75 cm). Vitamin B6 (pyridoxine) deficiency is approximately 17.6%. Vitamin B1, (thiamine), deficiency is estimated at 18.3% of post-bariatric patients. Most B complex deficiencies are asymptomatic. Vitamin B12 (riboflavin) deficiency is estimated to be 13.6% one year after bariatric surgery. Because of the malabsorptive component of Roux-en-Y gastric bypass, fat-soluble vitamin deficiencies can also occur in the post-bariatric patient. Vitamin A deficiency can be progressive, and deficiency is estimated to be 11% one year after surgery and 69% four years after surgery. Vitamin E deficiency is relatively uncommon in post-bariatric patients who are taking supplementation.
2017 A 48-year-old man is evaluated for a panniculectomy for symptomatic panniculitis 18 months status post Roux-en-Y gastric bypass. The patient has lost 150 lb (68 kg), and BMI is 28 kg/m2. He reports taking a multivitamin every day and exercising for 30 minutes three times a week. Physical examination shows no abnormalities. Preoperative nutritional evaluation will most likely show a deficiency in which of the following micronutrients? A) Iron B) Vitamin A C) Vitamin B1 (thiamine) D) Vitamin B12 (cobalamin) E) Vitamin D
The correct response is Option B. The increasing popularity of bariatric surgical procedures for the management of obesity has generated a plethora of plastic surgical techniques to deal with the sequelae of massive weight loss. A thorough analysis of the anatomical result of weight loss is key in determining the ideal procedure for each individual's situation and goals. While liposuction might be helpful as an adjunct to surgical excision, it does not usually produce measurable skin tightening in this group of patients. Skin resection options have been devised based on the skin laxity pattern. When significant horizontal and vertical skin excess is present, a corset trunkplasty, devised by A.P. Moya, is the most likely of the choices to result in a trimmer abdomen shape. This option addresses the upper abdominal skin laxity that is not routinely addressed with traditional abdominoplasty. Circumferential dermolipectomy, also known as belt lipectomy, creates a vertical tightening but not a horizontal tightening of loose skin. Laser lipolysis applied externally is not indicated for the management of excessive skin laxity. This patient has achieved an acceptable result (BMI of 29) of bariatric surgery and therefore no revision of the bypass operation is indicated.
2017 A 48-year-old woman with a history of a 160-lb (73-kg) weight loss after gastric bypass surgery comes to the office because she seeks a trimmer torso. Current BMI is 29 kg/m2. Physical examination shows excessive skin on the horizontal and vertical dimensions of the abdomen. Which of the following procedures is most likely to result in aesthetic correction of the deformity? A) Circumferential dermolipectomy B) Corset abdominoplasty C) Externally applied laser lipolysis D) Liposuction of the anterior abdomen E) Revision of the gastric bypass
The correct response is Option B. Combining an augmentation with a mastopexy has long been considered risky because the surgeon is addressing two opposing forces during the same operation: the ptosis and volume, for which the placement of additional weight may exacerbate ptosis. Studies have shown, however, that the two operations can safely be combined. During the planning, particularly for severe ptosis, the surgeon must be careful not to overresect skin that will be critical for closure over an implant. Compared with staged procedures, mastopexy-augmentation has a higher rate of need for revision procedures. Patients should be counseled about the potential need for revisions. Seroma and hematoma are not increased when combining the procedures, and nipple-areola complex necrosis is a function of pedicle size and patient-specific factors such as obesity and tobacco use, rather than the combination of procedures. Similarly, transection of intercostal nerves is associated more closely with pedicle type than with combining procedures.
2017 A 48-year-old woman, gravida 3, para 3, who wears a size 36B bra comes to the physician for evaluation of breast ptosis. BMI is 24 kg/m2. Physical examination shows the distance from sternal notch to nipple is 28 cm, and there is grade 2 breast ptosis with skin laxity. A combined augmentation/mastopexy is planned. Which of the following is the biggest risk of combining the procedures rather than staging them? A) Hematoma B) Need for revision procedure C) Nipple-areola complex necrosis D) Seroma E) Transection of lateral intercostal nerves
The correct response is Option D. Gynecomastia is benign proliferation of breast tissue in men. It is present in 40 to 50% of men over 40 years of age. It can manifest in pubertal boys and in men of advanced years. The etiology can be variable and may be due to excess circulating estrogen, decreased circulating androgens, or a deficiency of androgen receptors. However, in middle-aged and older men, it is most commonly due to the excessive aromatization of androgens to estrogens. Initial evaluation requires a detailed history and physical examination to differentiate between fatty tissue, parenchymal enlargement, and a tumor. Mammography may be useful and biopsy may be indicated in some cases.
2017 A 50-year-old man with a BMI of 36 kg/m2 comes to the office for consultation regarding gynecomastia. He takes no medications. Physical examination shows no other abnormalities. Which of the following is the most likely cause of this patient's gynecomastia? A) Decreased circulating estrogen B) Decreased estrogen receptors C) Excessive androgen receptors D) Excessive aromatization of androgen to estrogen E) Increased circulating androgen
The correct response is Option E. The orbicularis retaining ligament or the orbital malar ligament is a bilaminar membrane that spans from the periosteum of the inferior orbital rim to the fascia on the underside of the orbicularis. During aging, this ligament accentuates the orbital malar depression and restricts the orbital fat from blending with the sub oribularis oculi fat (SOOF). This patient presents with a prominent orbitomalar sulcus and tear trough deformity. Release of the medial portion of the orbicularis oculi muscle and the orbicularis retaining ligament allows fat transposition over the orbital rim, thus softening and improving this deformity. This procedure can be performed through either a transcutaneous or transconjunctival approach. The capsulopalprebral fascia is a retractor of the lower eyelid. It is incised during a transconjunctival fat excision. It is not incised in a transcutaneous approach. The inferior oblique tendon of the inferior oblique muscle should be protected during lower eyelid surgery. Lockwood's ligament is a supportive structure of the globe. The inferior limb of the lateral canthal tendon is not released for fat transposition. It may be released for canthal support and repositioning.
2017 A 50-year-old woman comes to the office seeking cosmetic improvement of the lower eyelids. Transposition of a pedicled postseptal fat graft from the central compartment is planned through a transcutaneous approach. A preoperative photograph is shown. Which of the following is the most appropriate structure to release for transposition? A) Capsulopalpebral fascia B) Inferior oblique tendon C) Lockwood ligament D) Lower slip of the lateral canthal tendon E) Orbicularis retaining ligament
The correct response is Option B. This patient presents with signs and symptoms consistent with an intra-arterial injection of hyaluronic acid (HA), which requires emergency treatment to restore circulation. Accidental intra-arterial injection of fillers leading to arterial compromise is a rare occurrence. When it occurs, devastating outcomes include blindness, stroke, skin necrosis, and permanent scarring. The presenting signs and symptoms may include pain, skin blanching, mottled skin discoloration, and slow capillary refill. The mainstay of treatment for intra-arterial injection of HA products is local injection of hyaluronidase into the site of injection and the local area of skin mottling. Hyaluronidase is an enzyme that catalyzes HA hydrolysis. Other treatments include massaging the area in order to promote distribution of hyaluronidase, topical nitropaste, aspirin, and warm compresses. Secondary treatments that can be considered after hyaluronidase injection include hyperbaric oxygen, papavarin, prostaglandin E1, heparin, and lidocaine. Radiologic evaluation with Magnetic resonance angiogram (MRA) or Doppler ultrasound would only delay treatment. Massage alone will not benefit this patient. Recommendations for risk reduction include the following: using large-bore blunt cannulas (27 gauge and larger), injecting less than 0.1-mL bolus in any single injection site, avoiding high-pressure injections, awareness of likely position of named vessels in the treatment area, using local anesthesia with epinephrine, and caution with deep injections around the radix, lateral nasal wall, and periorbital area.
2017 A 50-year-old woman receives an injection of 0.5 mL of hyaluronic acid filler into each nasolabial fold. She returns to the office 30 minutes later because of pain and mottled skin discoloration of the nasal tip and right ala nasi. Which of the following is the most appropriate next step in management? A) Doppler ultrasonography B) Hyaluronidase injection C) Lidocaine injection D) Massage E) Prostaglandin E1 injection
The correct response is Option D. There are several treatment modalities that may be properly used to resurface lower eyelid skin. The skin in this special area is quite thin and caution must be exercised to not cause scarring or retraction issues. Trichloroacetic acid (TCA) is a chemical peel agent that is commonly used to resurface wrinkles on the face. The solution comes in several strengths, and concentrations of 20 to 40% are commonly utilized in the thinner lower eyelid skin. Phenol-croton oil chemical peels have seen a dramatic increase in their use for facial surfacing once details were worked out to vary the strength of the solution. Previously, old-fashioned phenol peels contained a standard amount of croton oil as an irritant, and these peels produced a vigorous and drastic "all or none" effect. Once Hetter described techniques that involve varying the concentration of croton oil within the peel, it became possible to utilize this peel to resurface all areas of the face effectively, using varying concentrations on different parts of the face depending on the thickness of the skin within each region. For the lower eyelids, 0.1% or less is considered standard, and the use of 0.4% should be reserved for thicker skin such as that around the cheeks or the brow or perioral region. Use of this higher concentration would have a high likelihood of producing scarring within the ultrathin lower eyelid skin. Dermabrasion is a highly effective skin resurfacing modality that produces damages via an abrasive process to nearly any depth of the skin depending on the force applied to the head. This modality is most commonly used in the perioral area. Lower eyelid skin rhytides cannot be properly treated with standard dermabrasion techniques given the high risk for inducing damage to the sensitive and fragile structure of the lower eyelid and an inability to hold this skin taut enough to perform dermabrasion effectively and safely. While microdermabrasion can safely be used to resurface the lower eyelid skin, this modality penetrates the most superficial layer of skin only, affecting only the epidermis and not traversing the reticular dermis, which would be required to effect change in the wrinkles on the skin. Ultrasonic based skin tightening (e.g. Ultherapy) involves the use of ultrasonic energy to bypass the skin and affect deeper soft tissues and cause them to contract, giving the appearance of a "lift." It has no effect on the outer appearance of the skin.
2017 A 51-year-old woman with Fitzpatrick type I skin comes to the office seeking improvement for fine rhytides on the lower eyelid. She was started on isotretinoin and hydroquinone 6 weeks ago by the aesthetician and has been compliant with the regimen. Which of the following treatments is most appropriate for optimal improvement of the lower eyelid in this patient? A) Dermabrasion B) Microdermabrasion C) Phenol-croton oil peel 0.4% solution D) Trichloroacetic acid peel 30% solution E) Ultrasound-based skin tightening
The correct response is Option A. This patient has developed true eyelid ptosis from her botulinum toxin type A treatments coming into contact and affecting the levator palpebrae superioris muscle within the eyelid. Although the effects of botulinum toxin type A on any muscle are irreversible with medications, attempts to lessen the severity of the ptosis have been made with the use of eyedrops to stimulate the Müller muscle, which is located deep to the levator. Apraclonidine is an alpha-adrenergic agonist and as such stimulates the Müller muscle to contract. This contraction may elevate the eyelid 1 to 3 mm and lessen the amount of ptosis to varying degrees in order to make the overall appearance of the eyelids more tolerable to the patient until the effects of botulinum toxin type A wear off on their own and levator function naturally returns. The most common dose of apraclonidine is 1 to 2 drops three times daily until ptosis resolves. Tetracaine is a commonly used numbing agent for the corneal surface that enhances the comfort of using corneal protectors for periorbital surgery. Tobradex eyedrops are a combination of tobramycin and dexamethasone used for treatment of infection and/or its anti-inflammatory effect in the periorbital region. It has no effect on eyelid ptosis. Artificial tears are lubricating drops and have no effect on muscular action.
2017 A 52-year-old woman comes to the office to receive botulinum toxin type A injections to the corrugator and procerus. She returns to the office 1 week later because she is upset that her eyelids on both sides are droopy. Physical examination shows bilateral ptosis. Which of the following is the most appropriate treatment to improve this patient's condition until the effects of the botulinum toxin type A subside? A) Apraclonidine B) Artificial tears C) Botulinum toxin type A to the lateral orbicularis oculi D) Ophthalmic tobramycin and dexamethasone E) Tetracaine
The correct response is Option B. Nipple-sparing mastectomy and direct to implant reconstruction is becoming increasingly popular in the setting of prophylactic mastectomies. The criteria for nipple-sparing mastectomies have been increased to tumors not involving the nipple-areola complex, with some surgeons requiring a distance of 2 cm from the nipple and others espousing just a clean surgical margin at the nipple. Nipple-sparing mastectomies do not require recreating the breast envelope and therefore direct to implant reconstruction is possible. Increased risk for incidence of nipple necrosis in the setting of direct to implant reconstruction is associated with mastectomy incision involving the areola. The use of smooth round gel implants, grade 2 ptosis, tumor size, or previous breast biopsy scars are not associated with nipple necrosis.
2017 A 54-year-old woman with BRCA mutation is scheduled for a bilateral nipple-sparing mastectomy. Which of the following would lead to the highest risk for postoperative nipple necrosis? A) Grade 2 breast ptosis B) Periareolar mastectomy incision C) Previous excisional breast scar D) Tumor greater than 1.5 cm E) Use of smooth round gel implants
The correct response is Option A. The risk for deep venous thrombosis (DVT) is decreased in many plastic surgical procedures when intravenous sedation is used instead of general anesthesia. Intravenous sedation avoids the need for muscle relaxant and the associated loss of peripheral vascular resistance. Many rhytidectomy patients are at elevated risk for DVT; however, utilizing chemoprophylaxis will increase the hematoma risk and potentially jeopardize the final surgical result. Switching to an intravenous sedation technique is a safe and easy alternative for lowering DVT risk. The incidence of hematoma, infection, and skin necrosis have not been linked with this type of anesthesia. Intravenous sedation can be associated with increased operative time for the rare patient who is resistant to sedation.
2017 A 55-year-old woman comes to the office for facial rejuvenation surgery. Rhytidectomy with a high superficial musculoaponeurotic system flap and plication of the platysma are planned. In comparison with general anesthesia, the primary advantage of intravenous sedation in this patient is decreased risk for which of the following? A) Deep venous thrombosis B) Hematoma C) Infection D) Prolonged operative time E) Skin necrosis
The correct response is Option C. Unlike in men, androgenic alopecia in women tends to spare the frontal hairline and tends to affect the frontoparietal scalp. Like in men, it is a chronically progressive disease and causes miniaturization of scalp follicles through exposure of endogenous dihydrotestosterone in patients with increased scalp androgen receptors and 5-reductase concentrations. Topical minoxidil has been shown to increase scalp blood supply and slow hair loss in both males and females.
2017 A 55-year-old woman presents with androgenic alopecia. Her husband had a hair transplant 5 years ago. Which of the following characteristics of androgenic alopecia is more common in women than men? A) More rapidly progressive B) Not induced by dihydrotestosterone C) Thinning tends to be frontoparietal D) Topical minoxidil is ineffective
The correct response is Option A. Regnault classification of breast ptosis, based on the position of the nipple-areola complex (NAC) relative to the inframammary fold (IMF): Scale: Criteria Pseudoptosis: NAC is above the IMF Type I (mild): NAC is at or 1 cm below the IMF Type II (moderate): NAC is 1 to 3 cm below the IMF Type III (severe): NAC is at the lowest portion of the breast The type of mastopexy performed will depend on the degree of breast ptosis. Breast ptosis is graded using Regnault classification. Type I can be treated with a crescent mastopexy, when the degree of nipple-areola complex elevation does not exceed 1 cm. Type I or II ptosis can be treated with a periareolar mastopexy, when the distance of nipple-areola complex elevation ranges from 1 to 2 cm. Type II and III ptosis is amenable to the inverted-T technique, where the horizontal incision will reduce the distance from the nipple-areola complex to the inframammary fold, while the vertical incision will reduce the base diameter.
2017 A 58-year-old woman with moderate ptosis is evaluated for mastopexy. According to Regnault classification, which of the following best describes the location of the nipple-areola complex in type II breast ptosis? A) 1 to 3 cm inferior to the inframammary fold B) 4 cm inferior to the inframammary fold C) 6 cm inferior to the inframammary fold D) At or 1 cm inferior to the inframammary fold E) Superior to the inframammary fold
The correct response is Option D. Leuprolide acetate (Lupron) is used in the treatment of certain cancers, including prostate, endometriosis, and precocious puberty. Because it is a synthetic gonadotropin-releasing hormone (GnRH), it acts as an agonist of pituitary GnRH receptors. Its ultimate effect is a lowering of estradiol and testosterone levels through downregulation of luteinizing hormone and follicle-stimulating hormone secretion. Gynecomastia and breast tenderness are known side effects of leuprolide treatment for prostate cancer. Selective estrogen receptor modulators (SERMs), such as tamoxifen, are recommended for the prevention of gynecomastia as a result of antiandrogen monotherapy. Radiation as well has been shown to decrease the breast side effects of leuprolide acetate. Thus, an increase in size, rather than a decrease in size is expected. Darkening of the nipple areolar complex is often associated with pregnancy, but not leuprolide acetate. Galactorrhea is associated with drugs—commonly drugs that contain dopamine depleting agents, such as methyldopa, that cause receptor bock (such as Reglan), and that inhibit release, such as codeine and morphine. Histamine receptor blockade, from drugs such as cimetidine, can also cause galactorrhea. Pituitary tumors are also a cause of galactorrhea. A thorough assessment of all medications and medical history is necessary for evaluation of a patient with gynecomastia and mastodynia.
2017 A 65-year-old man is prescribed leuprolide acetate for prostate cancer. Which of the following is the most likely effect the drug will have on this patient's breasts? A) Darkening of the nipple-areola complex B) Decrease in size C) Galactorrhea D) Mastodynia E) Petechiae
The correct response is Option B. Adverse scarring is a common complication after brachioplasty and can warrant surgical revision in some cases. Liposuction of the arm with concomitant brachioplasty is a well-described and safe procedure. Along with seromas, recent studies show that hypertrophic scarring is the most common adverse outcome after this combined procedure. Other adverse outcomes such as hematoma, infection, and wound dehiscence are less frequently encountered. Sensory nerve injury, especially the medial antebrachial nerve, is possible, although it is much less likely than the incidence of hypertrophic scarring.
2017 A healthy 40-year-old woman is scheduled for bilateral brachioplasty along with liposuction of the upper arms. Which of the following is the most common unfavorable outcome of this combined procedure? A) Hematoma B) Hypertrophic scar C) Infection D) Sensory nerve injury E) Wound dehiscence
The correct response is Option A. The histologic finding is consistent with a diagnosis of ductal carcinoma in situ (DCIS). The incidence of occult breast cancer in reduction mammaplasty specimens most closely approximates 1%. Invasive ductal carcinoma is the most common malignant lesion identified, but DCIS, lobular carcinoma in situ, Paget disease, and fibrosarcoma have also been reported less frequently. All breast tissue removed from women older than 40 years should be sent to pathology for microscopic analysis.
2017 A healthy 42-year-old woman undergoes bilateral reduction mammaplasty for symptomatic breast hypertrophy. Pathologic analysis of one of the tissue specimens shows ductal carcinoma in situ. Which of the following percentages best represents the incidence of this finding in a reduction mammaplasty specimen? A) 1% B) 7% C) 10% D) 12% E) 25%
The correct response is Option D. The dorsal nasal artery is a distal continuation of the ophthalmic artery from the internal carotid artery. During nasal dorsum augmentation, accidental injection of filler into this artery under pressure can push the filler retrograde into the ophthalmic artery. When the pressure is returned to normal, the filler then can flow distally occluding the retinal artery and causing blindness. Intravascular injection of fillers into the angular artery of the nasolabial line can also cause blindness, although this would more commonly result in skin mottling and necrosis of the nasal tip skin. The cheek area overlying the malar bone, with few deep vessels, is a relatively safe place for injectables. Intravascular injection of the geniomandibular groove and lateral lip commissure is more commonly associated with vascular compromise of the lips and chin.
2017 Accidental injection of hyaluronic acid filler into which of the following areas is most likely to cause blindness by retrograde occlusion of the central retinal artery? A) Cheek B) Geniomandibular groove C) Lateral lip commissure D) Nasal dorsum E) Nasolabial fold
The correct response is Option D. Green tattoo ink responds effectively to treatment with a 755-nm Q-switched alexandrite laser. As of 2013, a picosecond-domain alexandrite laser became commercially available, giving 75% clearance of green pigment in just one to two treatments in fair-skinned patients. Alternatively, ruby lasers, with a 694-nm wavelength, can be used. The 1064-nm wavelength is very effective for black and other dark colors when a Q-switched or picosecond machine is used. Typically, this wavelength is less effective for green, with this color commonly being left behind after completion of a 1064-nm treatment series. The long-pulse Nd:YAG is used for hair removal and varicose veins. It does not have the short pulse width required for effective tattoo removal. Similarly, intense pulsed light (IPL), even when filtered to the correct wavelength, doesn't give the short pulse width required for tattoo removal. Long-pulse laser or IPL pulses in the millisecond domain usually result in incomplete tattoo clearance, thermal damage to surrounding tissues, and scarring. Trichloroacetic acid (TCA) peels are not pigment-specific. They have become popular with the do-it-yourself patient population, with unregulated sales over the Internet, leading to reports of hypertrophic scarring and chemical burns requiring formal excision and skin grafting. A TCA peel is not recommended as a tattoo treatment, even in the more commonly used concentrations of 30 to 40%, which are used for facial resurfacing. Carbon dioxide laser is not effective at targeting tattoo pigment.
2017 After laser removal of an elaborate multi-color tattoo, a patient has residual green ink remaining. Which of the following is the most appropriate treatment for this residual pigment? A) Carbon dioxide laser B) Intense pulsed light C) Long pulse Nd:YAG (1064-nm) laser D) Q-switched alexandrite (755-nm) laser E) 70% Trichloroacetic acid peel
The correct response is Option C. The patient appears to have benign pubertal gynecomastia, and many of these cases resolve spontaneously. The recommended treatment is observation and reassessment. Early surgery in these circumstances is usually reserved for patients experiencing more extreme psychological impact. Testosterone replacement would not be useful since by examination the patient appears to have an otherwise normal physiology. The three surgical interventions are all potential treatments for gynecomastia, but surgery is usually not recommended within the first six months of onset of gynecomastia in a pubertal male. The skin excision and nipple grafting option would be most useful if the patient had large quantities of fat and skin. The two techniques employing liposuction both might be adequate surgical techniques to address the type of gynecomastia this patient demonstrates, should surgery eventually become recommended.
2017 An otherwise healthy 14-year-old boy is evaluated because of bilateral breast enlargement over the past 3 months. He is worried that his friends will notice. He reports no illicit drug use and takes no medications. Physical examination shows normal hair distribution for the patient's age, no testicular masses, and firm discs of tissue under the areola of each breast. Which of the following is the most appropriate next step in treatment? A) Direct excision in the subareolar area, leaving a small button of tissue, and marginal breast liposuction B) Liposuction of entire breast with use of cutting cannula under the areola C) Reassessment in 9 months D) Testosterone replacement E) Wise pattern skin excision with nipple grafting
The correct response is Option A. A common fibroadenoma is the most likely diagnosis of this patient. Common fibroadenoma is the most common breast tumor in adolescent females and present between the ages of 14 and 16. Juvenile fibroadenoma is a variant of fibroadenoma and is usually seen in adolescents and young adults. It is associated with a normal stromal/epithelial balance, which distinguishes it from phyllodes tumor, and has both stromal and epithelial hyperplasia. In addition, juvenile fibroadenomas are characterized by rapid growth. A giant fibroadenoma is a clinical diagnosis, rather than a pathologic diagnosis. It is characterized by its size, usually greater than 5 cm. Complex fibroadenoma is characterized by fibrocystic changes on glandular tissue with underlying features of common fibroadenoma on pathologic analysis. A tubular adenoma has glandular proliferation on pathologic analysis, and while it is a subtype of fibroadenoma, it is not as common as common fibroadenoma. A lactating adenoma, similarly, is defined by the presence of secretory hyperplasia of lobules on pathologic analysis. Lactating adenomas are so defined because of the histologic presence of secretory hyperplasia, and they commonly occur in pregnant or lactating women. Many lactating adenomas will spontaneously regress. A phyllodes tumor is typically a large, rapidly growing lesion and can be either benign or malignant. It is rare in adolescents, but when found, is usually aggressive. Phyllodes tumor is related to fibroadenoma and is distinct from other forms of breast cancer. Treatment is wide local excision.
2017 An otherwise healthy 16-year-old girl comes to the office because of a painless mass in the left breast. Physical examination of the left breast discloses a circumscribed firm, rubbery, 3-cm mass without overlying skin changes, and no axillary lymphadenopathy. Results of a pregnancy test are negative. Which of the following is the most likely diagnosis? A) Common fibroadenoma B) Giant fibroadenoma C) Lactating adenoma D) Phyllodes tumor E) Tubular adenoma
The correct response is Option A. This patient presents with ectopic breast tissue. In utero, the milk line (galactic band) forms at 5 weeks of gestation. This bilateral structure courses from the axillae to the groin, and normal breasts form in the prepectoral region after there has been regression of the rest of the galactic band. When there is failure of this regression, breast tissue remains in locations outside of the normal breast. The most common location for ectopic breast tissue is in the axillae, although it can be found anywhere along the milk line from the axillae to the groin. Ectopic breast tissue outside of the milk line has been described and is termed aberrant breast tissue. The tissue found in these ectopic locations is breast tissue with the same characteristics and propensity for disease as normally located breast tissue, and breast cancer has been described in these tissues. In the absence of pathologic findings such as a mass, pain, and skin changes that are associated with breast cancer, there is no strong oncologic indication for excision. If there are findings concerning for a neoplasm, then work-up should be initiated and might include further imaging, core biopsy, and surgery. However, most cases present without pathologic findings and are excised to achieve a more reasonable appearance for the patient, the ability to don clothing more comfortably, and for the obvious social advantages. In this case, the patient is young, has no complaints, and has no physical findings to suggest a neoplasm. Excision should be offered. Obtaining bilateral mammograms is incorrect because there is no indication for imaging in this patient based on her age, history, and physical examination. In addition, mammograms of axillary breast tissue are technically unfeasible. Performing a core biopsy is incorrect as there is no concern for malignancy in this case. In the case of a mass noted within the ectopic axillary breast tissue, then an oncologic workup should be initiated which might include a core biopsy. An MRI of the chest is incorrect because there is no indication for imaging in this patient based on her age, history, and physical examination. Reassuring the patient with no further action is not the most appropriate management, as it will not address the patient's concerns and desires. In the patient who does not request excision or is not an appropriate surgical candidate, then reassurance and surveillance are appropriate. As this is a young female with axillary breast tissue, liposuction will not improve the excess breast tissue or skin.
2017 An otherwise healthy 28-year-old woman comes to the physician requesting removal of bilateral axillary masses. She states that the masses fluctuate in size and tenderness with her menstrual cycle. She reports that the masses have not had discharge or drainage. Physical examination shows smooth, spongy masses in both axillae. A photograph is shown. Which of the following is the most appropriate next step in management of this patient? A) Excise the bilateral axillary masses and skin B) Obtain bilateral mammograms of the axillary masses C) Order an MRI of the chest D) Perform a core biopsy of both axillary masses E) Perform liposuction
The correct response is Option C. In the absence of other postoperative findings, damage to or entrapment of the lateral femoral cutaneous nerve (LFCN) is the most likely cause of this patient's symptoms. The LFCN exits the abdomen near the anterior superior iliac spine and is the most commonly injured nerve during abdominoplasty (incidence of 1.36%). If a nerve injury is suspected, the diagnosis can be confirmed by injection of local anesthetic just proximal to the location of the pain or Tinel sign. Conservative treatment includes scar massage and physical therapy aimed at desensitization techniques. These nonoperative treatments can be combined with an oral anticonvulsant such as gabapentin for pain management in the short term. More severe or debilitating symptoms may warrant earlier surgical intervention. However, a local anesthetic nerve block is diagnostic and is indicated prior to either conservative or surgical treatment. A compression garment would be neither therapeutic nor diagnostic for this patient.
2017 An overweight 36-year-old woman who underwent full abdominoplasty by another surgeon 6 months ago is referred to the office from the emergency department because of persistent pain around the right lower quadrant of the abdomen with concomitant paresthesia to the lateral thigh. The patient reports her postoperative course was unremarkable. Abdominal CT scan from the emergency department shows postsurgical changes and an 8 x 1-cm linear fluid collection subjacent to the incision. Which of the following is the most appropriate next step in management? A) Administer oral gabapentin B) Apply compression garment C) Inject a local anesthetic D) Massage the scar E) Surgically explore the wound
The correct response is Option D. The keystone area of the nose is where the nasal bones overlap the upper lateral cartilages. This is usually the widest part of the nasal dorsum. At the dorsal keystone area, the nasal bones overlap the upper lateral cartilages for a distance of 4 to 14 mm. When reducing a dorsal hump, rasping of the bone at the keystone area uncovers the underlying cartilages, which often dictate the width of this area. Once uncovered, these structures may need to be reduced for a more aesthetic dorsal line. In large cadaver studies, all noses demonstrate that the upper lateral cartilages are posterior to the nasal bones at the keystone area, and as such all other answer choices are incorrect.
2017 At the keystone area of the nose, which of the following most accurately describes the anatomic position of the upper lateral cartilage in relation to the nasal bones? A) Anterior to the nasal bones B) Caudal to the nasal bones with a 1 to 2 mm fibrous gap C) Edge to edge with the nasal bones with no overlap D) Posterior to the nasal bones
The correct response is Option E. The overwhelming majority of reported cases of breast implant-associated anaplastic large cell lymphoma (BIA-ALCL) have been associated with textured surface implants. Anaplastic large cell lymphoma in association with breast implants is a rare occurrence; however, when it does appear, the course is usually less aggressive with a better prognosis than when it is unrelated to breast implants. Recent studies suggest that the breast implant shell causes a chronic T-cell stimulation. This reaction is thought to be caused by an interaction of textured surface characteristics and associated biofilm. BIA-ALCL associated with smooth shell implants has occurred; however, it is disproportionately rare. ALCL is seen with both silicone- and saline-filled implants. These numbers are highly influenced by the specific popularity of each implant. Specific implant dimensions, be it projection or width, are not uniquely associated with ALCL.
2017 Breast implant-associated anaplastic large cell lymphoma is most closely associated with which of the following implant characteristics? A) High-profile dimensions B) Saline filling C) Silicone gel filling D) Smooth shell E) Textured shell
The correct response is Option A. Liposuction-only reduction mammaplasty does not involve the use of a pedicle to ensure blood supply to the nipple. The nature of liposuction involves leaving major vessels and nerves intact and therefore does not pose an increased risk for blood supply loss to the nipple-areola complex. In addition, sensation to the breast as well as to the nipple-areola complex is not typically impaired following liposuction-only reduction mammaplasty. For similar reasons, breast-feeding potential is not typically compromised. The recovery from liposuction-only reduction mammaplasty can be quite significant, and it can take about 6 weeks for bruising and swelling to decrease and about 6 months for the breast to soften and for lumpiness to settle. Liposuction-only reduction mammaplasty does not work well when breast tissue is mostly glandular and thus has limited usefulness in those patients in whom it may seem most desirable, such as teenagers. Liposuction-only reduction mammaplasty does not more effectively treat ptosis.
2017 Compared with reduction mammaplasty excisional techniques, which of the following is characteristic of liposuction-only reduction mammaplasty? A) Does not typically impair breast-feeding potential B) More effectively treats ptosis C) Poses a higher risk to the blood supply of the nipple-areola complex D) Results in a higher incidence of impaired sensation to the nipple-areola complex E) Results in minimal swelling postoperatively
The correct response is Option C. Aesthetic analysis of the face may be simplified by dividing the face into equal horizontal thirds and vertical fifths. The length of the face is divided into equal thirds as follows: The upper third includes the forehead and brows, extending from the anterior hairline to the glabella and brows. The middle third includes the midface, eyes, and nose and extends from the glabella to the subnasale. The lower third includes the lower cheeks, jawline, and neck and extends from the subnasale to the menton. The width of the face may be divided into equal fifths by lines dropped from the lateral canthi and lines dropped from the medial canthi, with each partition approximating the width of the horizontal palpebral fissure. Of note, the lines dropped from the lateral canthi should approximate the width of the neck and the lines dropped from the medial canthi should approximate the distance between the left and right alar-facial grooves.
2017 For aesthetic analysis, the face can be divided into which of the following segments? A) Equal horizontal fifths and vertical fifths B) Equal horizontal fifths and vertical thirds C) Equal horizontal thirds and vertical fifths D) Equal horizontal thirds and vertical thirds E) There is no regular division of the face into horizontal or vertical proportions
The correct response is Option C. Studies have demonstrated the ideal anatomical characteristics of the breast to include: an upward pointing nipple, a straight or mildly concave upper pole slope, smooth lower pole convexity and fuller lower pole compared to upper pole. Breasts with an upper pole-to-lower pole ratio of 45:55 were identified as defining the ideal breast. The ratio was defined ideal by respondents including women, men, plastic surgeons, and individuals of ethnic diversity.
2017 In augmentation mammaplasty, which of the following is the ideal upper pole to lower pole anatomic ratio? A) 25:75 B) 35:65 C) 45:55 D) 50:50 E) 55:45
The correct response is Option D. Component separation for abdominal wall reconstruction involves release of the fascia lateral to the rectus abdominus muscles, just lateral to the semilunar line, dissecting the external oblique off the internal oblique muscles. This creates innervated musculofascial flaps that can be advanced medially for closure of ventral hernias. If the posterior rectus sheath is also dissected free, further advancements can be gained. Per rectus muscle, approximately 4 cm can be gained at the epigastric and suprapubic areas; 10 cm can be advanced at the waist. Therefore, the most advancement can be gained at the umbilicus, which is in the area of the waist. The xyphoid and ligament of Treitz are in the epigastric area, while the arcuate line is in the suprapubic area.
2017 In bilateral component separation for abdominal wall reconstruction, which of the following points is most likely to be the area of greatest advancement? A) Arcuate line B) Ligament of Treitz C) Suprapubic D) Umbilicus E) Xyphoid
The correct response is Option B. Muntan, et al. described breast development as starting at the fifth or sixth week of development, when outgrowths from the ectodermal skin layer penetrate into the underlying mesoderm, forming the mammary ridge or milk line. The ectodermal thickenings along the mammary line regress between gestational months 2 and 4, except for two of them in the region of the third and fourth ribs. The ectoderm keeps on extending into the underlying mesoderm at the fifth month, and a branching network forms what will eventually become the lactiferous system. The supportive connective and adipose tissue of the breast develops from the surrounding mesenchyme.
2017 In embryologic breast development, which of the following best describes the formation of the mammary ridge? A) Starts at the fifth or sixth week of fetal development, when buds of mesoderm grow into the overlying ectodermal skin layer B) Starts at the fifth or sixth week of fetal development, when outgrowths from the ectodermal skin layer penetrate into the mesoderm C) Starts at the seventh or eighth week of fetal development, when buds of mesoderm grow into the overlying ectodermal skin layer D) Starts at the seventh or eighth week of fetal development, when outgrowths from the ectodermal skin layer penetrate into the mesoderm E) Starts at the third or fourth week of fetal development, when buds of mesoderm grow into the overlying ectodermal skin layer
he correct response is Option C. Newborn infant ear deformities, in order of frequency, are prominent/cup ear, lop ear, mixed deformities, Stahl ear, helical rim anomalies, conchal crus deformity, and cryptotia. Bilateral congenital ear deformities occurred in 70% of 340 patients in one study, with unilateral involvement in 30%. While surgical correction of deformed ears constituted the mainstay of therapy for decades, more recent developments in ear molding techniques have demonstrated 90% success in some studies. The presence of circulating maternal estrogen is blamed for lack of helical cartilage rigidity. This fact allows clinicians to shape the ear and, if pursued for a sufficient period of time, therapy will eliminate the deformity without need for surgical intervention. For the technique to be maximally effective, molding should be initiated by the end of the first week after birth. When begun even a few weeks later, the success rate falls to 50%. Surgical intervention is more likely to be performed just before the child reaches preschool age, allowing the ear to reach most of its adult size first. Helmets may have applicability in cranial reshaping but are not suited for management of deformed ears. Oral medication of any kind is unlikely to affect ear shape. Awaiting spontaneous correction as the child ages is the least likely means of achieving corrected ear shaping.
2017 Parents of a healthy 3-day-old male newborn request consultation for management of their child's bilateral lop ear deformities. Which of the following is the most appropriate next step in management? A) Await spontaneous correction B) Fit the infant's head and ears for a helmet C) Initiate ear molding devices immediately D) Prescribe oral estrogen blocker therapy E) Schedule bilateral otoplasty surgery at age 3 months
The correct response is Option E. Botulinum toxin type A works by binding to receptor sites in presynaptic nerves to prevent the release of acetylcholine into the synapse of neuromuscular junctions.
2017 Which of the following best describes the mechanism of inhibition of muscle contraction by botulinum toxin type A? A) Binds to acetylcholine in presynaptic nerves, degrading acetylcholine B) Binds to acetylcholine in postsynaptic nerves, degrading acetylcholine C) Binds to acetylcholine in postsynaptic nerves, preventing acetylcholine interaction with nicotinic receptors D) Binds to nicotinic receptor sites on the neuromuscular junction, blocking acetylcholine stimulation E) Binds to receptor sites in presynaptic nerve terminals, blocking acetycholine release
The correct response is Option E. Evidence suggests that chronic inflammation is the stimulus responsible for the development of breast implant-associated anaplastic large cell lymphoma (ALCL) and T-cells are the predominant cell type responding to this antigenic stimulus. B-cells have been implicated in orthopedic implant lymphomas. The other cell types are involved in inflammation, but they are not associated with breast implant-associated ALCL.
2017 Which of the following cell types is most associated with the chronic inflammation that leads to breast implant-associated anaplastic large cell lymphoma? A) B-cells B) Monocytes C) Neutrophils D) Red blood cells E) T-cells
The correct response is Option E. Post-menopausal obesity is associated with increased adipose production of estrogen, which can increase the risk for hormone-sensitive breast cancer. Other options (late menarche, early menopause, and breast feeding) decrease the number of menstrual cycles, and therefore may decrease the risk of breast cancer. Early age at first pregnancy is also associated with decreased risk for hormone-sensitive breast cancers.
2017 Which of the following is a risk factor for hormone-sensitive breast cancer? A) Breast-feeding B) Early age at first pregnancy C) Early menopause D) Late menarche E) Post-menopausal obesity
The correct response is Option B. The rest of the options have been shown to increase the incidence of capsular contracture. Textured silicone implants, inframammary incisions, and submuscular implant placement have been shown to decrease the incidence of capsular contracture. The use of a surgical brassiere postoperatively has not been shown to decrease incidence of capsular contracture as well.
2017 Which of the following is most commonly associated with decreased incidence of capsular contracture? A) Formation of biofilm B) Placement of textured silicone device C) Subglandular placement of the implant D) Use of a periareolar incision E) Use of a postoperative surgical brassiere
The correct response is Option D. Plastic surgery faces one of the highest proportions of malpractice claims compared with other medical specialties. A number of studies have revealed that breast-related surgeries account for 37% of overall claims against plastic surgeons. The most common cause of action is negligence, related either to lack of appropriate knowledge or skill or to failing to meet the standard of care. The second most common cause of action is lack of informed consent. Lack of informed consent results from the failure of the physician to thoroughly discuss the risks associated with surgery and the options of alternative therapies. Other causes of action include failure to diagnose or treat injury related to the procedure, retained surgical materials, assault, and distortion of physician's credentials.
2017 Which of the following is the most common cause of litigation in cosmetic breast surgery? A) Assault B) Failure to diagnose or treat an injury related to the procedure C) Lack of informed consent D) Negligence E) Retained surgical instrument
The correct response is Option C. Retinoids pass through the cell wall via nonreceptor-mediated endocytosis, are carried to the nucleus on cellular retinoic acid-binding proteins (CRABP, CRABP-II), and exert their effect through binding to retinoic acid receptors and retinoid X receptors. The retinoid-receptor complex binds to the promoter gene in the region of the retinoid response elements, resulting in production of proteins responsible for effects we see histologically and grossly. In sun-damaged skin, the major findings histologically are reduced collagen quantity and dermal collagen disorder. Retinoids have come to be a mainstay in the treatment of photodamaged skin due to their ability to repair this damage. The effects noted histologically of retinoids on photodamaged skin include increased quantity of collagen (types I, III, and VII), greater organization of the collagen within the dermis, improved organization of elastic tissue, epidermal hyperplasia, increased mucin deposition (epidermal and dermal), and decreased melanin, among others. These histologic changes translate into improvement in rhytides, smoother skin, and correction of dyschromia. Thinning of the epidermis is incorrect because retinoids result in epidermal hyperplasia. Decrease in mucin deposition is incorrect because retinoids result in increased mucin deposition. Decrease in angiogenesis is incorrect because retinoids result in increased angiogenesis in the skin. Increase in melanin is incorrect because retinoids result in a decrease in melanin content of the skin.
2017 Which of the following is the most common histologic effect of skin treatment with tretinoin? A) Decrease in angiogenesis B) Decrease in mucin C) Increase in collagen D) Increase in melanin E) Thinning of the epidermis
The correct response is Option B. The internal nasal valve is an anatomical structure composed of several parts. Its cross-sectional dimension determines the quantity of airflow that passes through the nose while breathing both at rest and during exercise. Because it is the narrowest part of the entire airway, compromise of any of its elements correlates with symptoms of nasal obstruction. The internal nasal valve is bounded by the caudal border of the upper lateral cartilage superiorly, the nasal septum medially, the floor of the nasal vestibule inferiorly, the anterior part of the inferior turbinate posteriorly, and the bony edge of the pyriform aperture laterally. Internal nasal valve narrowing may be due to one or more of several factors, including septal deviation, turbinate hypertrophy, collapse of the upper lateral cartilage due to surgical disruption of the ligaments which support it to adjacent structures, and loss of cartilage strength due to aging. A variety of techniques have been described for restoration of the internal valve function. Most of these involve stiffening the upper lateral cartilage with cartilage graft struts, submucous resection of the nasal septum, reduction of hypertrophic turbinates, or combinations of these procedures.
2017 Which of the following structures is an anatomical component of the internal nasal valve? A) Anterior portion of the middle turbinate B) Caudal edge of the upper lateral cartilage C) Cribriform plate of the ethmoid D) Ostium of the maxillary sinus E) Superior border of the nasal bone
The correct response is Option E. Traditionally, the supraumbilical abdomen has been considered the area where liposuction might further disrupt blood supply already interrupted by the abdominoplasty undermining. Techniques to minimize undermining and preserve blood supply to allow more aggressive liposuction in this area have been reported and are in use. The mons pubis, lateral hip, flank, and lateral thigh blood supplies are less affected by abdominoplasty, and therefore, these areas have traditionally been less prone to complications when liposuction is performed in them at the time of abdominoplasty.
2018 A 32-year-old woman is interested in post-pregnancy body contouring. BMI is 34 kg/m2. When combined with abdominoplasty, liposuction of which of the following areas has an increased risk for wound healing complications? A) Flank B) Lateral hip C) Lateral thigh D) Mons pubis E) Supraumbilical abdomen
The correct response is Option B. The right breast is not significantly different from the left breast. The linguine sign describes multiple low-signal curvilinear lines on MRI that correlated to the collapsed implant shell. It is an indication of intracapsular rupture. Physical examination alone is not specific or sensitive enough to diagnose all cases of intracapsular rupture. Ultrasound and/or MRI is recommended. The physical examination finding of one breast that is smaller, firmer, and higher than the other is indicative of capsular contracture. MRI is not a sensitive predictor of capsular contracture. A right breast that is significantly larger than the left breast would indicate a late seroma and a workup for breast implant-associated anaplastic large-cell lymphoma would be indicated. A right breast that is significantly smaller than the left breast would be indicative of a ruptured saline implant. Increased rippling is not expected with an intracapsular rupture.
2018 A 35-year-old woman is evaluated for long-term follow-up 9 years after undergoing bilateral augmentation mammaplasty for hypomastia by another surgeon. The mammaplasty was performed with 450-mL smooth, round, silicone subglandular implants. The patient recently found out she is BRCA2 positive and underwent MRI of the breasts as part of a surveillance study. The MRI showed a "linguine sign" in the right breast. Which of the following findings on physical examination is most consistent with the diagnosis associated with the "linguine sign"? A) The right breast has more rippling than the left breast B) The right breast is not significantly different from the left breast C) The right breast is significantly larger than the left breast D) The right breast is significantly smaller than the left breast
The correct response is Option B. All of the answers listed can play a part in this patient's nasal obstruction and poor airflow in the right nostril. Of the answers listed, the bony hypertrophy of the inferior turbinate is the finding which most suggests the need for a submucous resection. The anterior extension of the inferior turbinate certainly can play a role in decreased air flow, but this by itself does not suggest the need for submucous resection. Simple mucosal thickening of the inferior turbinate without bony hypertrophy can be addressed with outfracture. Septal deviation can be addressed with septoplasty. Decreased internal nasal valve angle can be addressed with, for instance, a spreader graft.
2018 A patient is evaluated because of nasal airway obstruction that is worse on the right side. Physical examination shows the inferior nasal turbinate has significant anterior extension and mucosal thickening with bony hypertrophy. There is a posttraumatic septal deviation and a 10-degree internal nasal valve angle. In consideration of surgery to improve the nasal airflow, which of the following factors is most important in determining the need for a submucous resection of the turbinate? A) Anterior extension of the turbinate B) Bony hypertrophy C) Decreased internal nasal valve angle D) Deviated septum E) Mucosal thickening
The correct response is Option D. The classic Regnault definition of breast ptosis classifications are as follows: Grade I: Nipple at the level of the inframammary fold Grade II: Nipple between the level of the inframammary fold and the lowest contour of the breast Grade III: Nipple at the lowest contour of the breast
2018 In grade II ptosis of the breast, which of the following best describes the position of the nipple? A) At the apex of the breast mound B) At the lowest contour of the breast C) At the transposed inframammary fold D) Between the inframammary fold and the lowest contour of the breast E) On the posterior aspect of the breast as it rests on the chest wall
The correct response is Option A. Liposuction is often used in conjunction with excisional brachioplasty to facilitate dissection of the soft tissue, improve contour, and decrease the risks for nerve injury and lymphedema. There are many different techniques posed in the literature. Liposuction of the planned tissues of excision can ease the dissection planes. It can also be performed in the posterior upper arm to improve contour of the remaining tissues. Nerves and lymphatics are more protected by using liposuction instead of excision to debulk the arm. The use of liposuction does not increase the incidence of skin necrosis or wound dehiscence.
2018 Which of the following is the most likely result of performing liposuction in conjunction with brachioplasty? A) Facilitated tissue dissection B) Lymphedema C) Skin necrosis D) Upper arm paresthesia E) Wound dehiscence
The correct response is Option E. Gynecomastia is benign enlargement of the glandular breast tissue in a man. It is often related to factors that either results in an increase in estrogen production, or a decrease in androgen production. This can be due to the transient imbalances during puberty, or secondary to medication use, drug use (such as marijuana), testicular pathology, thyroid disease, liver disease, breast cancer, etc. This is in contrast to pseudogynecomastia, which is enlargement of the breast due to fat deposition in the absence of glandular hypertrophy. Pseudogynecomastia is typically seen in the patient who is overweight or obese. Pseudogynecomastia is distinguished from gynecomastia by physical examination. In true gynecomastia, one will palpate enlarged firm glandular breast tissue, as opposed to in pseudogynecomastia, where palpation will reveal a soft fatty breast throughout with no enlargement of the subareolar tissue. History may suggest a pathologic etiology that will be confirmed on laboratory testing. Medication and drug use questioning will show if the gynecomastia is pharmacologic in origin. Mammography and MRI will aid in determining if there is a malignant etiology causing the breast tissue enlargement. However, it is not standard for gynecomastia evaluation.
2018 A 22-year-old man comes to the office reporting breast enlargement that began 7 years ago. Physical examination shows bilateral breast enlargement. Testicular examination is normal. BMI is 27 kg/m2. Which of the following evaluations is best for distinguishing gynecomastia from pseudogynecomastia in this patient? A) History of medication use B) Laboratory testing C) Mammography D) MRI E) Physical examination
The correct response is Option D. There are many options for materials to use for augmentation rhinoplasty. Most options fall into one of several categories including autologous soft tissue, cartilage, and bone; processed allografts such as acellular dermal matrix; homologous grafts, most often irradiated rib; and alloplastic materials including silicone, ePTFE (expanded polytetrafluoroethylene sheeting), and porous polyethylene. There are advantages and disadvantages of each. In this case, the patient refuses a donor site outside of the nasal surgery, which eliminates the options of ear or rib cartilage and the option of cultured autologous auricular chondrocytes, as this too requires harvest of ear cartilage prior to the rhinoplasty. Homologous irradiated rib cartilage is a good alternative to autologous cartilage grafts and requires no donor site but has a somewhat higher risk profile than autologous septal cartilage. Silicone implants, and alloplastic implants in general, are popular in Asian augmentation rhinoplasties and have the advantage of availability, affordability, and no donor site, but they carry a higher risk for postoperative complications, including extrusion, skin changes, and infection, all of which create significant issues when they require secondary rhinoplasty procedures. Acellular dermal matrix has the disadvantage of a resorption rate of 20 to 30 percent, too much to perform precise corrections. An ear cartilage graft would be from a secondary site.
2018 A 23-year-old woman of Asian ancestry comes to the office to discuss augmentation rhinoplasty to address dorsal projection. She refuses harvest of graft material from a secondary donor site. Which of the following materials would most likely provide the desired augmentation, the least surgical risk, and the greatest longevity of result? A) Ear cartilage B) Homologous irradiated rib cartilage C) Porous polyethylene implant D) Septal cartilage E) Stacked acellular dermal matrix
The correct response is Option B. One of the most often mentioned potential risk factors for capsular contracture is biofilm, and this may be related to bacterial contamination. History of infection is unlikely to impact nipple sensation, implant rupture, true breast gland ptosis, or effacement of the inframammary fold with downward descent of the implant
2018 A 23-year-old woman undergoes augmentation mammaplasty with round, smooth silicone implants placed in the dual-plane position. Postoperatively, unilateral erythema and warmth are noted, and they slowly resolve over 10 days of oral antibiotic treatment. The patient asks what this might mean for future satisfaction with the outcomes. Which of the following is the most likely sequela of this patient's clinical course? A) Breast gland ptosis B) Capsular contracture C) Double-bubble appearance D) Implant rupture E) Nipple numbness
The correct response is Option A. The tuberous breast deformity was first described by Aston and Rees in 1976. While most of the surgical approaches listed, with the exception of implants together with Wise-pattern mastopexy, have been described for the spectrum of tuberous breast deformities, the key is to select the right procedure for the right patient. In this case, a mild form of the deformity is described. Implant placement alone, even with parenchymal scoring and lowering of the inframammary crease, is unlikely to correct the deformity of the nipple-areola complex. In cases of severe ptosis, vertical mastopexy may be used but would be unnecessary in this patient with mild ptosis. In severely deficient cases, a two-stage approach with tissue expansion may be necessary, but it would be over-operating in this mildly deficient patient. Recently, fat grafting has also been advocated for this procedure. In the case described, which is a common presentation, a periareolar approach is typically used to place the implant in a dual-plane configuration. Subglandular placement is also described. The inframammary crease is commonly adjusted downward. Radial scoring of the parenchyma and a circumareolar mastopexy are typically performed. In the recent review by Kolker and Collins, 92% of tuberous breast patients had a one-stage procedure. Ninety-six percent of these were treated with implant placement and circumareolar mastopexy, combined with inframammary crease adjustment and radial scoring of the parenchyma.
2018 A 23-year-old woman with micromastia and bilaterally tuberous breasts comes to the office to discuss augmentation mammaplasty and improving the overall appearance of her breasts. Physical examination shows bilateral mildly ptotic breasts with glandular tissue herniating through the nipple-areola complex. The lower pole appears mildly deficient in the lower medial and lateral quadrants, and the distance from the nipple to the inframammary crease is 5.5 cm on stretch. Which of the following is the most appropriate operative approach for this patient? A) Implant placement with circumareolar mastopexy B) Implant placement without mastopexy C) Implant placement with vertical mastopexy D) Implant placement with Wise-pattern mastopexy E) Two-stage reconstruction with tissue expander, followed by placement of a permanent implant
The correct response is Option D. The main advantage of ultrasound-assisted liposuction over suction-assisted liposuction is less surgeon fatigue. In traditional or suction-assisted liposuction, the fat is removed by the repetitive arm movements breaking up the fat. The suction then aspirates the loosened fat. In ultrasoundassisted liposuction, ultrasound energy breaks the fat apart and emulsifies it, thus allowing it to be removed by the suction cannula. The main advantage of this is to decrease surgeon fatigue from the repetitive arm movements and also to help break apart the fat in fibrous areas or areas of scar from prior procedures. Peer-reviewed studies have not shown any consistent evidence-based benefits for the use of ultrasound-assisted liposuction over suction-assisted liposuction. The aesthetic outcomes, patient satisfaction, and rates of long-term complications appear to be more related to technique and not technology. As a result, suction-assisted liposuction continues to remain the predominant technique of liposuction due to its relatively low cost and the rapid rate at which new technologies are introduced with little objective evidence supporting any additional benefit.
2018 A 28-year-old woman comes to the office for a consultation on liposuction of the abdomen. She has already had consultations with several other physicians and is unsure whether to undergo suction-assisted liposuction (SAL) or ultrasound-assisted liposuction (UAL). Which of the following is the most significant advantage of choosing UAL over SAL? A) Better aesthetic outcome B) Better skin tightening C) Greater patient satisfaction D) Less surgeon fatigue E) Lower complication rates
The correct response is Option D. In patients who have undergone gastric bypass surgery, consensus recommendations are to wait to proceed with body contouring surgery until the patient is at least 1 year from surgery. Additionally, recommendations are to wait until the patient has had a stable weight for at least 3 months and some authors advocate 6 months of weight stability. This is due to the fact that additional weight loss after surgery may result in recurrence of skin laxity. Stability of the long-term result is best achieved when there is stability of the underlying weight. Furthermore, risk of surgery increases with increasing BMI, thus waiting for the patient to achieve their lowest BMI prior to performing surgery will limit weight-associated risks. It is for these reasons that a return visit for reassessment in 6 months is the best choice as opposed to proceeding to panniculectomy at this time. A CT scan is indicated in those patients where a physical examination is unable to rule out an abdominal wall hernia. In an otherwise asymptomatic patient from a hernia standpoint, reassessment of the physical examination once the patient has achieved a stable weight is the time to determine if there is an abdominal wall hernia that would be addressed at the time of body contouring surgery. At this point in time, the patient may have lost enough weight to adequately assess the abdominal wall for a hernia, and thus make a CT scan unnecessary. Most post-bariatric patients will be obtaining laboratory assessments from their bariatric surgeon to assess for nutritional deficiencies and will be on supplements as indicated. After undergoing bariatric surgery, many patients have iron deficiency anemia. A complete blood count may be indicated for those who have symptoms or signs of anemia, or in whom a significant blood loss is anticipated. In relation to the body contouring surgery, this test is best undertaken once the plan to proceed with surgery is made. Again, this would occur once a stable weight has been achieved. Many patients with weight control issues suffer from mental health pathology. A referral to a psychiatrist would be indicated in any patient who has a history of mental health issues or demonstrates signs or symptoms of psychopathology prior to embarking on post bariatric body contouring. As this patient has no medical issues noted, the routine referral to a mental health provider is not indicated.
2018 A 32-year-old woman is evaluated for trunk contouring 12 months after undergoing laparoscopic gastric bypass surgery. Hypertension and sleep apnea have resolved. Current BMI is 32.7 kg/m2 . She reports an 80-lb (36.2-kg) weight loss and is still actively losing weight. Which of the following is the most appropriate next step in management? A) Complete blood cell count B) CT scan of the abdomen C) Panniculectomy D) Re-evaluation in 6 months E) Referral to psychiatry
The correct response is Option C. The lateral crural strut graft is a strip of septal cartilage that (if available for harvest) is sutured to the underside of the lower lateral cartilage and provides increased support and position control. It is a very powerful graft that can reposition lower lateral cartilages, correct alar retraction, and correct external valve collapse. The columella strut graft is for increased tip projection and support. The composite alar rim graft includes both skin and cartilage. This graft is used for severe alar retraction and soft tissue loss of the alar rim. Spreader grafts are used to increase airflow through the internal nasal valve as well as straighten a deviated dorsal septum and improve dorsal aesthetic lines. The subdomal graft is placed under the domes of the lower cartilages and can correct asymmetry of the nasal tip and improve a pinched tip.
2018 A 35-year-old man comes to the office for a consultation because he is dissatisfied with the result of a rhinoplasty performed 2 years ago. The patient reports that his nasal openings collapse on deep inspiration and his nasal tip is deformed. Physical examination shows collapse of the external nasal valve on deep inspiration and bilateral asymmetric alar rim collapse with alar retraction of 1 to 2 mm. Revision rhinoplasty is planned. Which of the following grafts is most appropriate to correct these conditions? A) Columella strut B) Composite alar rim C) Lateral crural strut D) Spreader E) Subdomal
The correct response is Option D. Each of the complications listed in this question has a significant occurrence with thighplasty in the massive-weight-loss population, but prolonged edema has been shown to be a particular risk factor in patients getting a full-length vertical component in their thighplasty, presumably due to circumferential compression of the low pressure lymphatic system. Labial spreading is possible but not likely.
2018 A 35-year-old woman comes to the office to discuss improving the contour of her thighs. History includes gastric bypass surgery two years ago, followed by a stable 150- lb (68-kg) weight loss. Along with moderate horizontal excess skin, she has significant vertical excess skin, and a full-length vertical thighplasty is considered. This patient is at greatest risk for which of the following complications? A) Hematoma B) Infection C) Labial spreading D) Prolonged edema E) Seroma
The correct response is Option B. Patients who present with a late seroma should be evaluated for possible breast implantassociated anaplastic large cell lymphoma (BI-ALCL). A late seroma is usually accepted as occurring 1 year following surgery; however, there are cases of BI-ALCL seromas that have presented as early as 4 months. The first step in evaluating BI-ALCL is ultrasonography, followed by fine-needle aspiration if indicated. The fluid requires evaluation beyond routine cell cytology. Immunohistochemistry test for CD30 was the most commonly positive marker for BIALCL. Immunohistochemistry stains specific antigens in cells by binding to this antigen in an antibody/antigen reaction. The specific stain can then be seen under light microscopy. The CD30 antibody labels anaplastic large cell lymphoma cells. CD30 is a transmembrane cytokine receptor belonging to the tumor necrosis factor receptor family. CK20 and CCD79a were negative for tested BI-ALCL specimens. P63 stains myoepithelial cells and is used to rule out invasive breast tumors. E-cadherin helps distinguish ductal from lobular carcinoma.
2018 A 35-year-old woman is evaluated because of swelling of the right breast 3 years after undergoing augmentation mammaplasty. The implant type is unknown. Ultrasonography shows a seroma, and a fine-needle aspiration is performed. Which of the following immunohistochemical stains of the aspirate is most appropriate? A) CCD79a B) CD30 C) CK20 D) E-cadherin E) p63
The correct response is Option E. For this patient in whom postoperative radiation therapy is possible, the best first-stage, immediate reconstructive approach is placement of tissue expanders with acellular dermal matrix. The outcome of immediate autologous flap reconstruction may be compromised if subjected to adjuvant radiation therapy and is best delayed until after such treatment has been rendered. Although successful, cost-effective outcomes are possible with a singlestage, direct-to-implant approach, this patient has risk factors for early revision and implant failure due to her large breasts and history of smoking.
2018 A 35-year-old woman with a Stage T2 infiltrating ductal carcinoma is scheduled to undergo a skin-sparing, right total mastectomy and a nipple-sparing, left prophylactic mastectomy. The possibility of adjuvant radiation therapy to the right breast depends on the final surgical pathology. The patient has a history of smoking. BMI is 28 kg/m2. She wears a brassiere with a D cup and would like the postoperative result to be of a similar size. Which of the following immediate bilateral reconstructive techniques is most appropriate for this patient? A) Abdominal-based free flaps B) Gluteal-based free flaps C) Latissimus dorsi myocutaneous flaps and silicone implants D) Silicone implants and acellular dermal matrix E) Tissue expanders and acellular dermal matrix
The correct response is Option B. Abdominoplasty is known to have a higher complication rate than many common aesthetic procedures. A recent study by Grotting and associates examining complication rates in more than 25,000 abdominoplasties in a multi-surgeon database confirmed that the risk for a complication requiring hospitalization or reoperation increases significantly when abdominoplasty was combined with other surgeries. In particular, abdominoplasty alone had a complication rate of 3.1%. Abdominoplasty combined with liposuction - 3.8% Abdominoplasty combined with a breast procedure - 4.3% Abdominoplasty combined with a breast procedure and liposuction - 4.6% Abdominoplasty combined with liposuction and a body contouring procedure - 10.4% Abdominoplasty combined with liposuction, a breast procedure, and a body procedure - 12.0% In this study, body procedures included brachioplasty, buttock lift, calf implant, labiaplasty, lower body lift, thigh lift, and upper body lifts. Hematoma, infection, and suspected or confirmed venous thromboembolism represented 31.5%, 27.2%, and 20.2% of overall abdominoplasty complications in this study. A second study, by the same group, looking at 129,000 cosmetic surgery patients as a group also confirmed an increased risk for major surgical site infections in cosmetic patients undergoing multiple simultaneous procedures. Thus, caution is advised when considering multiple procedures concurrently in a higher risk patient.
2018 A 35-year-old woman, gravida 3, para 3, is interested in a "mommy makeover," with liposuction of the flanks, extended tummy tuck, brachioplasty, and augmentation mammaplasty. In this combination of surgeries, which of the following percentages best represents the cumulative risk in the general population for postoperative complications requiring reoperation or hospitalization? A) 3% B) 12% C) 25% D) 35% E) 50%
The correct response is Option C. This patient presents with multiple risk factors for deep vein thrombosis (DVT). According to the 2005 Caprini Tool for DVT Risk Assessment, this patient has a minimum of 7 points (3 points for family history of thrombosis, 1 point multiple miscarriages (>3 miscarriages), 1 point for BMI > 25, and 2 points for Major Surgery > 45 minutes). With both a family history of thrombosis and multiple miscarriages, there is a significant likelihood that the patient has a genetic thrombophilia. A hematologist will be able to diagnose and quantify the significance of a genetic thrombophilia. A genetic thrombophilia such as Positive factor V Leiden gene mutation, if present, would add an additional 3 points of risk for DVT and pulmonary embolus, bringing the total risk score from 7 to 10. If abdominoplasty were performed, chemoprophylaxis would be highly recommended with these risk factors. The mechanism of multiple miscarriages in women with thrombophilia is clotting of the placenta and subsequent fetal loss. These women are given anticoagulation chemoprophylaxis during pregnancy to help prevent placental thrombosis. The most common inherited thrombophilia is factor V Leiden, which is present in 3 to 7% of the Caucasian population. Multiple inherited thrombophilia conditions can be present in the same individual. Consultations with a sleep apnea specialist, cardiologist, dermatologist, and nutritionist are all reasonable considerations; however, the evaluation for genetic thrombophilia is most crucial to this patient's evaluation for abdominoplasty.
2018 A 39-year-old woman comes to the office for a consultation regarding abdominoplasty. Three years ago, she underwent gastric sleeve bariatric surgery, which was followed by a 100-lb (43-kg) weight loss. Weight has been stable for 1 year. Past medical history includes mitral valve prolapse, sleep apnea, and exercise-induced asthma. The patient is gravida 5, para 2, with 3 miscarriages. Her mother had deep vein thrombosis at 70 years of age while on a 5-hour flight. BMI is 31 kg/m2. Physical examination shows an overhanging panniculus with intertrigo. An evaluation by which of the following is the most appropriate next step? A) Cardiologist B) Dermatologist C) Hematologist D) Nutritionist E) Sleep specialist
The correct response is Option A. Site change and implant exchange are the only factors that have consistently been shown to decrease recurrence of capsular contracture, although other factors including use of a textured implant and fat grafting used are in augmentation mammoplasty revision. Botulinum toxin type A has been described for prevention of capsular contracture however; no consensus that these treatments decrease recurrence of capsular contracture exists. Furthermore, there are no data to support performing total versus partial capsulectomy, or even the superiority of capsulectomy over capsulotomy.
2018 A 40-year-old woman who underwent a subglandular augmentation mammaplasty with smooth round silicone breast implants 5 years ago returns to the office for evaluation of an increasingly firm left breast. Surgical revision of the left breast is planned. Which of the following measures is most likely to decrease the recurrence of the symptoms? A) Conversion to a new plane or pocket B) Performing a total capsulectomy C) Using botulinum toxin type A in and around the implant pocket D) Using fat grafting in and around the implant pocket
The correct response is Option E. Fat necrosis is one of the more common complications associated with reduction mammaplasty. Regardless of technique, the rates of fat necrosis have been reported in the 2 to 10% range. Fat necrosis presents as firm, soft-tissue masses that usually resolve spontaneously. It can be associated with redness and mild discomfort and may be confused with an infectious process. The literature is inconsistent with respect to detailed cause and effect or definitive correlations between fat necrosis and risk factors. However, some are generally agreed upon as significant and are mostly consistent in studies. Some of these risk factors for the development of fat necrosis include greater BMI, larger resection weights (both absolute and controlled for preoperative breast size), and long suprasternal notch to nipple distance (especially over 37 cm). Other factors that less clearly affect the rate of fat necrosis are smoking and the surgical technique employed. Multiple studies have demonstrated increased risk of fat necrosis with greater obesity. Although there have been some studies that suggest a correlation between fat necrosis and age, the studies have not reached a consensus nor have they shown statistical significance when evaluated in a controlled fashion. One study by Shermak et al. looked specifically at age-related risks and was not able to find a significant correlation with fat necrosis. There have been no studies or associations found to link exogenous hormone therapy or oral contraceptive use to increased fat necrosis. Alternatively, there has been some evidence to suggest that hormone supplementation might decrease the rate of infectious complications. The rate of fat necrosis and complication in general increases with longer suprasternal notch to nipple distances, most notably greater than 37 cm. Massive weight loss is not associated with an increased risk for fat necrosis in the breast.
2018 A 43-year-old woman comes to the office for consultation for reduction mammaplasty. She wears a DDD brassiere and wants her brassiere size to be decreased to a C cup. Current medications include oral contraceptive pills. She does not smoke cigarettes. Height is 5 ft 5 in (167 cm) and weight is 145 lb (65.7 kg). BMI is 23.8 kg/m2. Physical examination shows the suprasternal notch to nipple distance is 29 cm and inframammary fold to nipple distance is 16 cm. Regardless of technique, which of the following factors is most likely to result in an increased risk for postoperative fat necrosis in this patient? A) Massive weight loss B) Oral contraceptive use C) Patient age D) Suprasternal notch to nipple distance E) Tissue resection weight
The correct response is Option B. In general, the indication to use biologic scaffolds in hernia repairs and abdominal reconstruction is in contaminated beds. Biologic scaffolds provide an intact extracellular matrix and support tissue regeneration, and are more resistant to infection than synthetic mesh, likely because of this tissue ingrowth. Biologic scaffolds, unlike synthetic mesh, are degraded over time by collagenase. This accounts for at least some of the higher recurrence rates seen with biologics compared to synthetic mesh. Cross-linking of the scaffold may provide resistance to collagenase and improve long-term stability. Thus, better resistance to infection is a property of biologic scaffolds, as is collagen crosslinking, although the degree of cross-linking varies based on the particular scaffold. They are associated with a higher cost and an increased recurrence rate. Their high cost compared with synthetic mesh does not justify their routine use, and the recommendation for their use by the Ventral Hernia Working Group is in contaminated fields, with infected mesh and septic dehiscence. The need for components separation is based on the properties of the hernia itself and is independent of the type of mesh used. Components separation without mesh is, however, associated with a high recurrence rate.
2018 A 43-year-old woman, gravida 3, para 3, undergoes a combined panniculectomy and hernia repair. The planned hernia repair is a retrorectus repair without components separation with polypropylene mesh. Which of the following best describes an advantage of using polypropylene mesh compared with a biologic scaffold? A) Decreased need for components separation B) Decreased recurrence rate C) Increased collagen cross-linking D) Increased resistance to infection
The correct response is Option C. The American Society of Plastic Surgeons participates in the Choosing Wisely campaign, which advocates for evidence-based guidelines in determining diagnostic and therapeutic interventions. The American College of Surgeons' recommendations for breast cancer screening in average-risk, asymptomatic women are for an opportunity for a baseline mammogram at 40 to 44 years of age, annual screening from 45 to 54 years of age, and biennial screening for women older than 55 years of age who are in good health and have a life expectancy of at least ten years. This patient is asymptomatic, and is not a BRCA carrier, and meets criteria for a screening mammogram. A diagnostic mammogram is performed to evaluate abnormalities found on screening mammogram, in the context of breast cancer history, or with physical exam findings such as a breast mass, nipple discharge, or breast pain. MRI is recommended as an adjunct to mammography in the case of a known BRCA mutation, if the first-degree relative is known to have the BRCA mutation but the patient is untested, or if there is a lifetime risk of 20 to 25% of breast cancer. Thermography uses an infrared camera to show patterns of blood flow and heat on the surface of the breast. It is not a replacement for mammography, not recommended as part of screening protocols, and is not associated currently with any quality studies that demonstrate that it can be used effectively as a screening tool for breast cancer. Ultrasonography is also used as an adjunct to screening mammography. It is important to note that different societies have different recommendations on timing of screening mammography. The ACS recommends screening at 45 years of age, the American College of Radiology recommends screening starting at 40 years of age, and the USPSTF recommends biennial mammograms between 50 and 74 years of age.
2018 A 44-year-old woman, gravida 4, para 3, is evaluated because of symptomatic macromastia. Bilateral reduction mammaplasty is planned. The patient's mother was diagnosed with postmenopausal breast cancer at 53 years of age; the patient underwent genetic testing which was negative for BRCA mutation. Physical examination shows the patient wears a size 32F brassiere and has grade III ptosis, shoulder grooving, dense breast tissue without palpable masses or nipple discharge, and intertrigo. According to current American Cancer Society recommendations, which of the following breast imaging methods should be used before the planned reduction mammaplasty in this patient? A) Diagnostic mammography B) MRI C) Screening mammography D) Thermography E) Ultrasonography
The correct response is Option C. Contralateral breast procedures are frequently necessary to achieve symmetry following mastectomy and reconstruction. Options to achieve symmetry include mastopexy, reduction mammaplasty, or augmentation mammaplasty combined with mastopexy. Volume symmetry can be achieved through reduction mammaplasty or fat injections. Positional asymmetry of the contralateral breast and the nipple-areola complex may require correction with mastopexy. Augmentation mammaplasty with mastopexy may be indicated for the correction of volume and positional asymmetry. The most appropriate contralateral procedure in a patient with symmetric volumes of the breast and ptosis of the contralateral breast is mastopexy.
2018 A 46-year-old woman with cancer of the right breast comes to the office to discuss a right mastectomy with immediate implant reconstruction and a symmetry procedure for the contralateral breast. Physical examination shows bilateral Grade 3 ptosis with volume symmetry. The patient currently wears a size 34C brassiere and desires to remain the same size. Which of the following is the most appropriate procedure for the contralateral breast? A) Augmentation mammaplasty B) Fat injections C) Mastopexy D) Reduction mammaplasty
The correct response is Option B. Male gender is an increased risk factor for hematoma and seroma in body contouring patients, independent of hypertension. Age, incision site, and postoperative BMI have not been shown to increase the risk for hematoma.
2018 A 48-year-old man who underwent laparoscopic gastric bypass surgery 2 years ago is evaluated for a belt lipectomy after a massive weight loss of 150 lb (68 kg). He has maintained a stable weight for over 6 months. Current BMI is 30 kg/m2. Which of the following factors most increases the risk for hematoma in this patient? A) Age B) Gender C) Location of incision D) Postoperative BMI
The correct response is Option C. Intravascular injection of hyaluronic acid (HA) fillers can lead to devastating outcomes including soft tissue necrosis, scarring, blindness, and stroke. The pivotal treatment for this complication is injection of hyaluronidase into the injection site as well as into the area of vascular compromise. Hyaluronidase is able to degrade the HA. There is also evidence that this effect can degrade the HA embolus. The recommended initial dose is 150 to 300 units. These doses can be repeated as needed. Smaller doses can be used if non-intravascular occlusion is suspected. Doses of 10 to 20 and 50 to 100 units would be appropriate for overfilling deformities or overfilling external occlusions. Patients with intravascular occlusion may require doses of 600 to 750 and 1000 to 1500 units over hours or days; however, this would not be the initial dose.
2018 A 48-year-old woman returns to the office 1 hour after receiving hyaluronic acid filler into the nasolabial area. She reports pain in the right cheek and a mottled appearance of the nose and cheek. Physical examination shows reticulated discoloration of the nose and right cheek. An intravascular injection with occlusion is suspected. Which of the following is the most appropriate initial dose in units of hyaluronidase for this patient? A) 10 to 20 B) 50 to 100 C) 150 to 300 D) 600 to 750 E) 1000 to 1500
The correct response is Option C . Central forehead paresthesias are typically related to traction injury to the supraorbital nerve, a division of ophthalmic (V1) nerve. The supraorbital and supratrochlear nerves supply the central forehead. The frontal branch of the facial nerve is a motor nerve to the frontalis muscle and, although at risk during brow lift, it has no sensory supply. The zygomaticotemporal nerve supplies the anterior temporal area. The zygomaticofacial nerve supplies the skin of the lateral orbit and cheek. The infraorbital nerve supplies the central face and upper lip.
2018 A 50-year-old woman returns to the office 2 weeks after undergoing an endoscopic brow lift and reports numbness in the central forehead. Which of the following nerves is most likely injured? A) Frontal branch of the facial B) Infraorbital C) Supraorbital D) Zygomaticofacial E) Zygomaticotemporal
The correct response is Option C. Aromatase inhibitors such as anastrozole impair conversion of androgens to estrogens. Estrogens promote normal breast tissue growth as well as growth of many breast cancers. The other options (taxanes, anthracyclines, alkylating agents, and platinum agents) are all chemotherapeutic agents that do not particularly target hormones involved in normal breast growth mechanisms.
2018 A 52-year-old woman comes to the office to discuss revision of breast reconstruction following mastectomy for breast cancer. She is undergoing adjuvant treatment with an agent that interferes with her body's natural mechanisms that promote native breast growth, but she cannot remember its name. She is most likely being treated with which of the following agents? A) Alkylating agent (cyclophosphamide) B) Anthracycline (doxorubicin) C) Aromatase inhibitor (anastrozole) D) Platinum agent (cisplatin) E) Taxane (paclitaxel)
The correct response is Option C. Component separation of the abdominal wall was initially described for the anterior components, that is, those which are located anterior to the rectus muscle. A more recent development involves component separation of the layers located posterior to the rectus fascia. The technique of posterior component separation begins with a vertical incision of the posterior rectus sheath 0.5 cm medial to the linea semilunaris and continues laterally in the avascular plane posterior to the transversalis muscle. It can extend as far posteriorly as the psoas muscle if needed. In the event that the posterior layer cannot be approximated in the midline, an interposition patch of omental fat, hernia sac, or absorbable mesh is used. Concurrent use of non-absorbable mesh to reconstruct deficient anterior layers may be used in conjunction with the posterior separation technique as long as the mesh is separated from viscera with an intact posterior layer. A benefit of the posterior separation technique is the preservation of the lateral neurovascular bundles preserving the dynamic function of the rectus muscles. The technique may be utilized even when fascial defects are not situated in the midline or are located adjacent to bony landmarks.
2018 A 59-year-old man is scheduled for reconstruction of a central abdominal wall hernia measuring 10 cm wide × 30 cm long. A surgical approach using posterior component separation is planned. Which of the following locations for fascial incision most accurately describes the technical considerations of this procedure? A) Across the lateral intercostal neurovascular bundles B) Along the mid-axillary line C) Medial to the linea semilunaris D) Parallel to the subcostal border E) Vertical bisection of the rectus muscles
The correct response is Option B. A patient with significant excess neck skin laxity requires excision of skin. The goals of this surgery must be rejuvenation while maintaining a normal appearing hairline with inconspicuous scars. An incision that follows the hairline of the posterior scalp allows neck skin to be removed without creating irregular and misplaced hair lines. The final extension into the scalp hair allows better camouflage of the end of the scar. Incision A is appropriate when the excess neck skin is mild to moderate. When there is a large neck skin excess, this incision design will pull non-hair-bearing skin into the scalp. Incision C will effectively remove excess neck skin; however, the low transverse component leaves a scar that is easily seen. Incision D stops at the lower retro-auricular sulcus. This incision is used in a short scar facelift. It is effective for improving jowling; however, only minor neck skin laxity can be improved with this technique. A superior vector pull of the face/neck skin is necessary to tighten the neck in a short scar technique. Skin bunching at the ear lobule and skin draping deformities are a risk.
2018 A 65-year-old woman comes to the office for consultation regarding rhytidectomy. On physical examination, skin pinch demonstrates greater than 2 inches of excess neck skin on each side of the face. In the illustration shown, all incisions (A to D) begin at the pre-hairline of the temporal area, extend to the tragal edge, and then go around the ear lobule to the posterior auricular sulcus. Which of the following incision paths is most appropriate to correct the patient's neck deformity? A) High transverse extending into the posterior scalp B) Posterior scalp pre-hairline with an inferior extension into the hair-bearing scalp C) Low transverse in the non-hair-bearing neck skin D) Extension to the lower aspect of the posterior auricular sulcus
The correct response is Option C. Women who were previously treated for breast cancer are at increased risk for development of a metachronous lesion of the contralateral breast. Cancers of the breast are typically adenocarcinomas, arising from the glandular tissue such as the ducts or lobules. Paget disease of the breast would involve the areolar skin but is fairly uncommon. Sarcomas arising from the connective tissue (such as adipose or muscle) are also rare. Breast adenocarcinomas do not originate from lymphatic tissue.
2018 A 65-year-old woman with a history of left mastectomy for breast cancer 10 years ago undergoes biopsy of a suspicious lesion of the right breast found on a recent mammogram. Examination of the biopsy specimen confirms a right breast carcinoma. This lesion most likely originated from which of the following structures? A) Adipose tissue B) Areolar skin C) Lactiferous duct D) Lymph node E) Pectoralis major muscle
The correct response is Option C. Cardiotoxicity is an uncommon but feared complication of chemical peels that, like the Baker-Gordon formula, contain phenol. EKG changes can include tachycardia, frequent premature ventricular contractions, premature atrial contractions, bigeminy, and ventricular tachycardia. In the study by Landau, the incidence of these issues was 6.6% and was increased in patients with diabetes, hypertension, and depression. Of the 12 patients out of 181 who developed arrhythmias in this study, eight were successfully treated with lidocaine, and the other four resolved without treatment. EKG monitoring is therefore mandatory whenever a phenol-containing peel is performed. Performing the peel in segments, in no less than 45 to 60 minutes, also has been reported to reduce the chances of EKG changes. Good intravenous hydration is also recommended. If these precautions are taken, it is rare to have to treat cardiac arrhythmias. Of the choices listed, the most appropriate advanced cardiac life support drug would be lidocaine 1 mg/kg. While the other drugs are part of the typical crash cart, they are not indicated in this case. Atropine is used for symptomatic bradycardia. Adenosine is used for narrow-complex supraventricular tachycardias, but the correct dose is 6 to 12 mg, not 4 mg. Magnesium is used in cardiac arrest only if the patient is suspected of having hypomagnesemia, in torsades de pointes, or in cases of suspected digoxin toxicity. Lipid emulsion is used for local anesthetic toxicity, which is not suspected in this case. Procainamide or amiodarone would be other antiarrhythmic agents that could be considered for this case, but they are not listed in the choices.
2018 A 65-year-old woman with extensive photodamage undergoes a deep chemical peel of the face and forehead using the Baker-Gordon peel formula during general anesthesia. The treatment takes approximately 20 minutes. Near completion, the anesthesiologist reports that the EKG monitor has shown frequent premature ventricular contractions for the last 2 to 3 minutes, then multiple sustained runs of bigeminy, followed by one short run of ventricular tachycardia. Oxygen saturation is 98%, and blood pressure is 105/70 mmHg. Intravenous administration of which of the following is the most appropriate management? A) Adenosine 4 mg B) Atropine 1 mg C) Lidocaine 1 mg/kg D) Lipid emulsion 100 mL E) Magnesium sulfate 1 g
The correct response is Option C. While there are a number of successful methods to perform a facelift, there are certainly common principles they share. These include: • Making an artfully placed incision which follows anatomic contours • Skin elevation to allow access to the superficial musculoaponeurotic system (SMAS) and release of any tethered points of the facial skin following SMAS manipulation • Some method (elevation, plication, imbrication, or direct suturing) of tightening the mobile SMAS • Anchoring of the SMAS in its new position with some stable method of fixation • Re-draping the soft-tissues using appropriate vectors • Careful skin closure where minimal tension is placed on the earlobe and posterior hairline incision Placement of significant tension on the earlobe caused by excessive trimming of the skin flap in the region of the otobasion inferius is well known to commonly produce a distinctive postoperative distortion of this structure, also known as a "pixie ear." The study by Mowlavi and associates showed an incidence of approximately 6% in a facelift population. A tension-free closure of the earlobe to the skin flap should be the goal, avoiding tension on the delicate earlobe tissues from the cheek flap, to avoid the issue of inferior and anterior migration of the otobasion. A number of corrective techniques have been described, ranging from local V-Y closures to readvancement of the facelift flap. Earlobe ptosis, in which the vertical height of the earlobe increases with aging, is unchanged by skin trimming errors. Telephone and reverse telephone deformities are complications in otoplasty for prominent ears, and do not apply to this case. Lop ear deformity is a congenital ear deformity involving the superior portion of the helix, and is not relevant to this case.
2018 A 67-year-old woman with marked lower facial and neck laxity is scheduled to undergo an extended superficial musculoaponeurotic system (SMAS) rhytidectomy. Excessive trimming of the skin flap adjacent to the base of the earlobe is most likely to result in which of the following outcomes? A) Earlobe ptosis B) Lop ear deformity C) Pixie ear deformity D) Reverse telephone ear deformity E) Telephone ear deformity
The correct response is Option A. While there is evidence that doses of up to 55 mg/kg can be safely used in liposuction, the safe dose is likely dependent on the vascularity of the tissue injected rather than aspiration of the local anesthetic during the procedure. Studies have shown that between approximately 10 and 30% of local anesthetic is present in the aspirate, and one study showed that a mean of 9.8% of wetting solution was removed. The knowledge that the majority of tumescent solution remains in vivo is important because lidocaine toxicity is a risk after tumescent liposuction.
2018 A healthy 29-year-old woman undergoes suction-assisted lipectomy with a tumescent solution for thigh lipodystrophy. A maximal dose of lidocaine 55 mg/kg is planned through the tumescent solution, and aspiration is planned through a 4-mm blunt-tipped cannula. Which of the following best approximates the expected percentage of local anesthetic in the aspirated material? A) 20% B) 50% C) 70% D) 90%
The correct response is Option D. This case illustrates superior nipple malposition. The distance from nipple to inframammary (IMF) fold of 14 cm is much too long. The correct answer is to resect the lower pole skin at the IMF in order to move the nipple down. This would create a "T" scar and improve nipple position. Vertical mastopexies and reduction mammaplasties have a learning curve and much of this is predicting the nipple position postoperatively. The nipple should be designed lower on the breast than is done during marking a Wise pattern. At 6 months, it is unlikely the nipple position will change dramatically, so observation is not recommended. Addition of an implant will not help the nipple position. Conversion to free nipple grafts, while possible, will not lead to an aesthetic scar pattern.
2018 A patient comes to the office 6 months after undergoing bilateral vertical mastopexy because she is dissatisfied with her postoperative appearance. Height is 5 ft 5 in (165 cm). Physical examination shows the distance from nipple to sternal notch is 16 cm bilaterally, and the distance from nipple to inframammary fold is 14 cm bilaterally. Which of the following is the most appropriate next step in management? A) Conversion to free nipple grafts B) Placement of a dual-plane breast implant C) Placement of a subglandular breast implant D) Resection of excess skin at the level of the inframammary fold E) Reassurance, massage, and observation
The correct response is Option B. The cutaneous innervation of the female breast is derived medially from the anterior cutaneous branches of the first to sixth intercostal nerves and laterally from the lateral cutaneous branches of the second to seventh intercostal nerves. The nipple-areola complex is physiologically innervated by the lateral and anterior branches of the third to fifth intercostal nerves. The fourth intercostal nerve has further shown to be most consistent in various anatomical studies. However, the anterior branches take a superficial course within the subcutaneous tissues of the medial breast while the lateral branches take a deep course within the pectoral fascia and reach the nipple via the breast parenchyma. Therefore, the lateral branches are most likely resected during mastectomy and contribute little to the postoperative innervation of the nipple-areola complex.
2018 After a nipple-sparing mastectomy, which of the following branches of an intercostal nerve predominantly provides remaining sensation to the nipple-areola complex? A) Anterior branch of the fifth B) Anterior branch of the fourth C) Lateral branch of the fifth D) Lateral branch of the fourth E) Lateral branch of the third
The correct response is Option E. A detailed knowledge of the nasal tip blood supply is critical for safe conduct when using a transcolumellar incision during primary or secondary rhinoplasty. There are several sources of arterial blood supply to the nasal tip. Some of these can be sacrificed without compromising the viability of the nasal tip skin. Rohrich et al. determined that the nasal tip has a dual blood supply derived from the ophthalmic and facial arteries. While contributions from the ophthalmic circulation's anterior ethmoid, dorsal nasal, and external nasal arteries are present, the dominant supply is derived from branches of the facial artery. Its branches, the columellar artery (present in 68.2% of cadavers in one study) and the lateral nasal artery (present in 100% of cadavers), are more likely to provide the nasal tip with inflow even if the ophthalmic arterial branches are sacrificed during the dissection. Regardless of the presence of a prior transcolumellar incision, the nasal tip blood supply is secure if the lateral nasal arteries are preserved. Guidelines for assuring that the lateral nasal arteries remain uninjured include "hugging" the cartilage of the lateral crura in a subperichondrial plane, limiting dissection superiorly to the level of the alar groove, limiting alar base excision to a level below the alar grooves, and limited defatting of the subdermal plane of the tip.
2018 During secondary open rhinoplasty through an existing transcolumellar incision, division of which of the following arteries is most likely to result in vascular ischemia of the nasal tip? A) Anterior ethmoid B) Columellar C) Dorsal nasal D) External nasal E) Lateral nasal
The correct response is Option D. Satisfaction with reconstruction, quality of life, and feeling of completeness with reconstruction have all been shown to be linked to nipple and/or areola reconstruction. The reconstruction does not restore nipple function; therefore, breast feeding, sexual sensation, and reactivity to touch are not accomplished with nipple and/or areola reconstruction. Often the nipple reconstruction, if using local tissue techniques, may actually flatten the anterior aspect of the breast and is not thought to positively impact the breast mound shape.
2018 Improvement in which of the following is an advantage of nipple-areola complex reconstruction? A) Breast feeding ability B) Breast mound shape C) Reactivity to touch D) Satisfaction with reconstruction E) Sexual sensation
The correct response is Option D.. The nipple-areola complex is innervated by the lateral cutaneous branch of the fourth intercostal nerve as well as the terminal branches of the fourth and fifth anterior intercostal nerves. However, when a superior medial pedicle in a reduction mammaplasty is used, the contribution from the lateral branch of the fourth intercostal nerve is excised. The lateral cutaneous branch of the second intercostal nerve is also known as the intercostobrachial nerve, which provides sensation to the medial and posterior upper arm. The medial pectoral nerve innervates portions of the pectoralis major and minor.
2018 In a Wise-pattern reduction mammaplasty using the superior medial pedicle, sensation is provided to the nipple-areola complex by which of the following nerves? A) Lateral cutaneous branch of the fourth intercostal nerve B) Lateral cutaneous branch of the second intercostal nerve C) Medial pectoral nerve D) Terminal branches of the fourth and fifth anterior intercostal nerves E) Terminal branches of the second and third anterior intercostal nerves
The correct response is Option E. Poland syndrome is the rare congenital condition describing the absence of the breast or nipple, hypoplasia of subcutaneous tissue, absence of the costosternal portion of the pectoralis major muscle, absence of the pectoralis minor muscle, and absence of costal cartilages or ribs 2, 3, and 4 or 3, 4, and 5, occasionally even rib 6. Its aesthetic and functional impairment varies between individuals and highly depends on the severity of the disease. Boys are three times as likely to be affected as girls. Incidence ranges between 1:100,000 to 1:7,000 cases per live births. Various concomitant anomalies have been published in association with Poland syndrome that typically necessitate a multi-disciplinary approach towards the diagnosis and treatment of this congenital illness. Upper extremity anomalies, renal malformation, lung hernia, and dextrocardia have all been described in literature as related to Poland syndrome. Anomalies of the upper extremity are the most common concomitant malformation besides the chest wall defects.
2018 In addition to the typical chest wall deformities, which of the following is the most common associated clinical presentation of patients suffering from Poland syndrome? A) Dextrocardia B) Lung hernia C) Renal malformation D) Thyroid malignancies E) Upper extremity anomalies
The correct response is Option C. Blepharoptosis occurs as a result of acquired or congenital causes. Among the former causes is the most common one, involutional (senile), as well as traumatic, mechanical, neurogenic, and myogenic. The latter category includes myogenic or neurogenic causes only. Because surgical correction is the only effective, definitive therapy for acquired ptosis, it is important for the clinician to be able to differentiate between the causes. The involutional variety is due to attrition or dehiscence of the levator aponeurosis from the anterior upper part of the tarsal plate. Diagnosis of ptosis is confirmed when there is less than 2.5 mm of distance between the upper eyelid margin and the papillary light reflex. Physical findings that support the diagnosis of involutional ptosis include a lid drop during downgaze, a higher than normal upper lid crease, visibility of the eye through the thinned upper eyelid, and preservation of good levator excursion (greater than 10 mm). Rating the degree of ptosis is based on the upper eyelid margin to pupillary light reflex distance: greater than 4 mm is considered normal. Excess upper eyelid skin can obstruct visual field; however, it is not indicative of involutional ptosis.
2018 In an adult patient, which of the following physical findings is most likely to support a diagnosis of involutional blepharoptosis of the upper eyelids? A) Excess upper eyelid skin B) Eyelid margin-to-reflex distance of 4.5 mm C) Higher than normal tarsal crease D) Less than 8 mm of levator function E) Upper eyelid eversion upon downgaze
The correct response is Option D. Early reports have raised concerns about the safety of combined augmentation mammaplasty with mastopexy surgeries. However, in patients who wish to correct their breast ptosis these two procedures are often combined to a one-stage surgery and can show favorable outcomes. Nevertheless, plastic surgeons advocate that these cases should only be performed by experienced physicians. Reasons are that the overall aesthetic results are harder to predict in one-stage augmentation/mastopexy procedures compared to mastopexy alone or even the two-stage augmentation mammaplasty followed by mastopexy. Both complication and revision rates are highest in the one-stage approach that combines augmentation mammaplasty with mastopexy. Longer operative time and the need for implants naturally increase operative costs. The advantage of the simultaneous insertion of implants is the improved superior pole projection that cannot be achieved by mastopexy alone.
2018 In women with breast ptosis, which of the following is an advantage of performing combined one-stage augmentation mammaplasty with mastopexy compared with mastopexy alone? A) Better predictability of cosmetic outcome B) Decreased complication rates C) Decreased revision rates D) Improved upper pole projection E) Lower operative costs
The correct response is Option E. The superficial musculoaponeurotic system is continuous with the temporoparietal fascia superiorily and the platysma inferiorly. Superiorly to inferiorly, the superficial layer continuous with the superficial musculoaponeurotic system consists of galea, temporoparietal fascia, cheek superficial musculoaponeurotic system, platysma, and superficial cervical fascia. The deep cervical fascia (DCF) makes up the most inferior extent of the layer deep to the superficial musculoaponeurotic system. Superiorly to inferiorly, this layer consists of cranial periosteum, deep temporal fascia (DTF), parotidomasseteric fascia, and DCF. The DTF splits into two layers, superficial and deep, which surround the superficial temporal fat pad as they extend inferiorly toward the zygomatic arch. The superficial and deep layers of the DTF extend anteriorly and posteriorly to the zygomatic arch, respectively. The superficial layer then becomes the parotidomasseteric fascia, and the deep layer becomes the posterior masseteric fascia. The temporalis muscle lies deep to the DTF and, therefore, is also deep to the superficial temporal fascia, which is continuous with the superficial musculoaponeurotic system.
2018 The superficial musculoaponeurotic system is continuous with which of the following? A) Parotidomasseteric fascia and deep cervical fascia B) Superficial layer of deep temporal fascia and deep cervical fascia C) Superficial layer of deep temporal fascia and platysma D) Temporalis muscle and platysma E) Temporal parietal fascia and platysma
The correct response is Option D. Alloplastic breast reconstruction increases the risk for capsular contracture in the setting of adjuvant radiation therapy. It can also increase the risks for seroma, wound healing complications, and infections. Radiation would not decrease the rates of implant rupture and would not increase patient satisfaction.
2018 Which of the following is most commonly associated with alloplastic breast reconstruction in the setting of adjuvant radiation therapy? A) Decreased rates of implant rupture B) Decreased rates of seroma C) Increased patient satisfaction D) Increased risk for capsular contracture
The correct response is Option E. Cryolipolysis is a noninvasive technique to preferentially destroy adipose cells through controlled thermal reduction. Exposure of adipose cells to below normal temperatures results in apoptosis-mediated cell death. Adipose cells are more susceptible to thermal reduction as compared with adjacent tissue. The subsequent inflammatory response results in the removal of damaged adipose cells within 3 months. The most common complication following cryolipolysis is hypoesthesia or decreased sensation of the treated areas, which resolves within 6 months. Other complications, which are less common, include paradoxical adipose hyperplasia, surface contour irregularities, and chronic pain. No hematomas or seromas have been reported in the literature.
2018 Which of the following is the most common complication after cryolipolysis? A) Hematoma B) Paradoxical adipose hyperplasia C) Seroma D) Surface contour irregularities E) Transient hypoesthesia
The correct response is Option B. Overinjection of hyaluronic acid can result in subcutaneous nodules that are often visible and/or palpable. Isolated nodules can be treated with simple incision and drainage using a sharp needle. However, multiple nodules due to excessive injection may require treatment with hyaluronidase. Hyaluronidase is a mucolytic enzyme that hydrolyzes hyaluronic acid dermal fillers. The recommended treatment is injection of 25 to 100 IU into the affected area followed by gentle massage. The most common complication following the injection of hyaluronidase is local allergic reaction. Hyperpigmentation, skin necrosis, and scar contracture have not been reported with the use of hyaluronidase. Neurapraxia is an unusual complication.
2018 Which of the following is the most common complication after undergoing hyaluronidase injection for the treatment of HA nodules? A) Hyperpigmentation B) Local allergic reaction C) Neurapraxia D) Scar contracture E) Skin necrosis
The correct response is Option E. Seroma is the most common complication following abdominoplasty. Progressive tension sutures and/or use of drains have been shown to be effective in preventing this complication. Use of antibiotics is intended to prevent infection. Discontinuation of NSAIDs, fish oil, and herbal supplements is intended to decrease the incidence of hematoma. Offloading pressure points and extremities in neutral position is to decrease the incidence of neuropathies postoperatively. Pneumatic compression devices and subcutaneous heparin and enoxaparin are intended to decrease the incidence of venous thromboembolism.
2018 Which of the following measures reduces the risk for the most common complication in abdominoplasty? A) Administration of preoperative intravenous antibiotics within 1 hour of incision B) Attention to offloading pressure points and keeping upper extremities in neutral position C) Discontinuation of NSAIDs and herbal supplements 4 weeks before surgery D) Use of pneumatic compression devices before the induction of general anesthesia E) Use of progressive tension sutures and drain placement
The correct response is Option A. The corrugator supercilii is a small and narrow muscle in the upper third of the face. It arises from the medial supraorbital ridge and stretches laterally and upward between the frontalis and orbicularis oculi muscles. Its almost horizontal position causes the vertical glabellar wrinkles between the eyebrows upon contracture. The natural function of that muscle is to move the eyebrow both medially and downward. The frontalis muscle is a brow elevator and causes horizontal forehead wrinkles, the procerus muscle leads to transverse glabellar lines, the oribicularis oculi muscle results in lateral canthal, so-called "crow's feet" wrinkles, and the nasalis muscle can cause small nasal lines upon contracting.
2018 Which of the following muscles is responsible for vertical glabellar rhytides and can be inhibited in its function by injection of botulinum toxin type A for aesthetic purposes? A) Corrugator supercilii B) Frontalis C) Nasalis D) Orbicularis oculi E) Procerus
The correct answer is Option C. The superior orbital fissure is a bony hiatus situated between the lesser and greater wings of the sphenoid bone. Through it pass multiple vascular and neural structures from the middle cranial fossa into the orbit. These include the superior and inferior divisions of the ophthalmic vein, sympathetic fibers from the cavernous plexus, and several cranial nerves. The mnemonic "Live Frankly To See Absolutely No Insult" is a helpful aid to identify the nerves which traverse the superior orbital fissure: L- lacrimal division, ophthalmic nerve (V1) F- frontal division, ophthalmic nerve (V1) T- trochlear nerve (IV) S- superior division of the oculomotor nerve (III) A- abducens (VI) N- nasociliary branch of the ophthalmic nerve (V1) I- inferior division of the oculomotor nerve (III) The optic nerve enters the orbit via the optic foramen which lies at the apex of the orbit and perforates the sphenoid bone. Fractures of the orbit which involve the superior orbital fissure may have severe consequences for the involved soft tissue structures. Injury to one or more of the structures passing through the superior orbital fissure may cause pain, ophthalmoplegia, or even blindness
2018 Which of the following nerves passes from the middle cranial fossa into the orbit via the superior orbital fissure? A) Mandibular (cranial V) B) Maxillary (cranial V) C) Oculomotor (cranial III) D) Olfactory (cranial I) E) Optic (cranial II)
The correct response is Option D. In patients with a significant glandular component, it is important to do a subtotal glandular resection to prevent recurrence and provide the best chance at adequate contour. Ultrasonic liposuction has been used to attempt subtotal removal of gland tissue, but it does not reliably produce a subtotal resection. Areolar reduction is usually not necessary due to the contractile nature of the tissue. Skin resection may be necessary, most commonly in the massive weight loss patient, but does not necessarily play a role in the glandular component of the resection. Lateral suction lipectomy does not correct glandular hypertrophy.
2018 Which of the following surgical options is most important for gynecomastia patients with significant glandular hypertrophy? A) Areolar reduction B) Excess skin removal C) Lateral lipectomy D) Subtotal glandular resection E) Ultrasonic liposuction
The correct response is Option E. Studies using three-dimensional CT scanning have demonstrated five consistent findings in unilateral cleft lip nasal deformity: 1. The columellar base is deviated to the noncleft side 2. The cleft-side alar base is more posterior than the noncleft-side alar base 3. The noncleft-side alar base is farther from the midline than the cleft-side alar base 4. The cleft-side piriform margin is farther posterior than the noncleft-side piriform margin 5. The angle made by the medial and lateral crura would be more obtuse on the cleft side
2018 - not graded A 15-year-old girl presents with a cleft lip nasal deformity. In infancy, she was treated in a cleft center with appropriate management and surgical correction of unilateral cleft lip and palate. Which of the following current findings is most consistent with the deformity? A) Cleft-side alar base is more anterior than the noncleft-side alar base B) Cleft-side piriform margin is farther anterior than the noncleft-side piriform margin C) Columellar base is deviated to the cleft side D) Medial to lateral crural angle is more acute on the cleft side E) Noncleft-side alar base is farther from the midline than the cleft-side alar base
The correct response is Option C. The correct response is a patient with ear deformation at 1 week of age. When deciding on molding, the first question to ask is what is the difference between malformation and deformation. An ear malformation is when there is a partial absence of either the skin or cartilage of the external ear. These patients tend to be less optimal candidates for molding. Patients with ear deformations have fully developed but misshapen ears and therefore are better candidates for molding. The plasticity of the cartilage is due to maternal circulating estrogen which peaks at day 3 and returns to baseline at week 6, and up to 30% of infants will self correct in the first week of life. One-day-old is too soon to begin molding because in 24 hours, some of these deformities will spontaneously resolve.
2018 - not scored Which of the following newborns would be the best candidate for ear molding? A) One-day-old newborn with helical rim deformation B) One-day-old newborn with helical rim malformation C) One-week-old newborn with conchal deformation D) One-week-old newborn with conchal malformation
The correct response is Option E. The chest findings described are consistent with Poland syndrome with absence of the sternocostal head of the pectoralis major muscle. Poland syndrome can be associated with hand abnormalities, including shortened digits. Cleft lip, accessory nipple, clubfoot, and microtia are not known to be associated with Poland syndrome.
2015 A 16-year-old girl comes to the clinic because she is dissatisfied with the asymmetric appearance of her breasts. Physical examination shows the right nipple-areola complex is more superiorly located and the breast volume is small. There is absence of the right anterior axillary fold. Which of the following other physical examination findings is most likely? A ) Accessory nipple B ) Microtia C ) Right clubfoot D ) Scars consistent with repaired cleft lip E ) Shortened right-hand digits
The correct response is Option C. Breast hypertrophy can affect girls as young as 10 to 15 years old and can result in massive breast development that can have profound physical and psychological impact on the patient. Reduction mammaplasty is indicated for these patients, despite the risk of breast growth postoperatively, which may necessitate a secondary operation. As long as the patient has a mature attitude, understands the permanence of the scars involved, and has parents who are supportive of her decision, then surgery should not be delayed simply to wait until the patient reaches a certain age. The large breasts are already a major problem and further growth will compound the problem and make it difficult to manage later. With the rise in childhood obesity, there has been an increase in the number of patients seeking adolescent reduction mammaplasty. The cause of macromastia in pubertal and parapubertal girls is variable and includes endocrine changes, childhood obesity, and juvenile (virginal) hypertrophy of the breast. Benefits of reduction mammaplasty include resolution of pain, improved quality of life, extroversion, and emotional stability. Reduction mammaplasty has been clearly proven to decrease the physical manifestations associated with macromastia, regardless of height and weight, as well as provide psychosocial benefits and improve self-esteem, regardless of age of patient.
2015 A 16-year-old girl comes to the office with her parents because she would like reduction mammaplasty surgery. Height is 5 ft 1 in (155 cm) and weight is 160 lb (72.6 kg). She wears a size 36G brassiere and her breast size has remained the same for the past year. She has significant physical manifestations of macromastia. Her parents report that she does not want to go to work or school because she is embarrassed by the size of her breasts. Which of the following is the most appropriate management of this patient's condition? A ) Defer reduction mammaplasty until the patient has lost at least 20 lb (9.1 kg) B ) Defer reduction mammaplasty until the patient is at least 22 years of age C ) Perform reduction mammaplasty D ) Refuse to perform surgery until the patient is evaluated by a psychiatrist
The correct response is Option D. Gynecomastia can occur because of imbalances of estrogen and testosterone, and it can be associated with obesity, certain drugs, and Klinefelter syndrome. It can also be associated with testicular tumors. Therefore, a testicular examination should be performed. Mammography is not necessary for a routine workup if physical examination shows no abnormal masses. A chest x-ray study is not needed for an otherwise healthy 19-year-old. An x-ray study of the hand can evaluate closure of growth plates when determining if puberty has completed. This is unlikely to be necessary for someone who is 19 years old.
2015 A 19-year-old man with no available medical history presents for initial evaluation of gynecomastia. Which of the following is most appropriate to include in this patient's workup? A ) Chest x-ray study B ) Hand x-ray study C ) Mammography D ) Testicular examination
The correct response is Option A. Texturing of the implant surface has been shown to decrease the rate of capsular contracture when compared with smooth implants when the implants are placed in the subglandular position. The benefit of textured implants may not be present when the implants are placed in a submuscular pocket. There is no difference in hematoma rates for textured versus smooth implants. Both symmastia and implant malposition are related to pocket dissection and not related to the type of implant placed. In the case of symmastia, the pockets have encroached upon the sternum and are close to each other or are touching. Implant malposition can be related to factors such as inadequate dissection of the pocket, or over-dissection of the pocket. Finally, some studies have demonstrated an increase in rippling with textured implant when compared with smooth implants. However, rippling may be more related to cohesiveness of the gel and fill volumes of the shell, because early reports of experience with the form-stable implant (Natrelle 410) seem to show decreased rates of rippling.
2015 A 24-year-old nulliparous woman comes to the office for augmentation mammaplasty. She currently wears a size 34B brassiere and wants her brassiere size to be increased to a D cup. She is a good candidate for subglandular placement of implants. Which of the following risks is decreased by the use of the textured silicone shell compared with the smooth silicone shell? A ) Capsular contracture B ) Hematoma C ) Prosthesis malposition D ) Rippling E ) Symmastia
The correct response is Option C. The most likely cause for a persistent bulbous tip after traditional maneuvers is convexity of the lower lateral cartilages. Lateral crural mattress sutures are effective in improving this convexity. These sutures are placed spanning the convexity and then tightened to straighten the curvature. Another option would be an alar batten graft, which is a graft placed on the medial surface of the lower lateral cartilage. Additional cephalic trim could lead to weakening of the nasal tip support and would not correct the problem. Columellar strut grafting, shield grafting, and spreader grafting will not effectively improve a naturally convex lower lateral cartilage.
2015 A 25-year-old woman undergoes rhinoplasty to correct a bulbous tip. After a cephalic trim leaving 6 mm of the lower lateral cartilage, transdomal sutures, and infracture, the tip continues to look bulbous. Which of the following techniques is most likely to improve this persistent deformity? A ) Additional cephalic trimming B ) Columellar strut grafting C ) Lateral crural mattress suture D ) Shield grafting E ) Spreader grafting
The correct response is Option C. Mondor disease of the breast is a benign, self-limiting thrombophlebitis of the inframammary veins. Clinically, Mondor disease usually occurs 2 to 3 weeks postoperatively as a painful, tender cord within the superficial veins of the thoracoepigastric system. Management is observation and includes the use of warm, moist dressings and anti-inflammatory agents for symptomatic relief. The use of anticoagulation, antibiotics, or steroids is not indicated. Implant removal is not indicated in the absence of infection. Duplex ultrasonography is not required for management.
2015 A 27-year-old woman is evaluated because of pain 2 weeks after undergoing subglandular augmentation mammaplasty. She has no history of fever, chills, or drainage. Physical examination discloses a painful, tender cord in the inframammary region of the left breast. Which of the following is the most appropriate next step in management? A ) Administration of an antibiotic B ) Administration of an anticoagulant C ) Administration of an anti-inflammatory agent D ) Duplex ultrasonography E ) Removal of the implant
The correct response is Option B. The tuberous breast is a developmental deformity characterized by a constricted inframammary fold, short nipple to inframammary crease distance, and both horizontal and vertical deficiencies. The pathophysiology of the tuberous breast predisposes the patient to develop a double-bubble deformity. In this patient, the inframammary crease must be lowered to accommodate the implant and improve the vertical skin deficiency. Radial release of the lower pole breast fascia is done with either a cautery or a knife. Multiple radial incisions are made, thereby allowing the tight crease to expand and decrease the chance for a double-bubble deformity. Lowering the crease is necessary but will increase the chances of a double-bubble deformity. Subparenchymal implant placement and use of highly cohesive gel implants may help but are not the essential procedures required. The use of acellular dermal matrix can help secure the position of the inframammary crease in a patient who develops a double-bubble deformity secondary to an inferior migration of the implant below the inframammary crease. This does not apply in the patient described.
2015 A 28-year-old woman desires augmentation mammaplasty with silicone implants. Physical examination shows tuberous breast deformity with an elevated inframammary crease. Sternal notch to nipple distance is 21 cm bilaterally. Nipple to inframammary crease distance is 3.5 cm bilaterally. Periareolar mastopexy with 350-mL silicone implants is planned. Which of the following operative plans will most effectively minimize the likelihood of a double-bubble deformity? A ) Lower the inframammary crease by 3 cm B ) Perform radial release of the lower pole breast fascia C ) Place implants in subparenchymal pocket D ) Reinforce the inframammary crease with acellular dermal matrix E ) Use highly cohesive gel implants
The correct response is Option A. Blepharoplasty is the most common facial cosmetic procedure performed on people of Asian descent. Unlike blepharoplasty in Caucasian faces, the goal of Asian blepharoplasty is to create a supratarsal fold. Asian eyelids are characterized by several key elements including absent or low lid crease, shorter tarsus, descending pre-aponeurotic fat, and minimal or absent connection between the levator aponeurosis and the upper lid dermis. The mainstay of surgical correction is creation of a permanent fixation point between the levator muscle and the supratarsal dermis and subdermal structures. The most common complication after Asian blepharoplasty is asymmetry. It is important to remember that unlike Caucasian blepharoplasty, the motivation for Asian blepharoplasty is less frequently rejuvenation. Rather, Asian patients are typically younger and desire creation of a supratarsal fold or correction of a preexisting fold asymmetry. Asymmetry is a common preoperative finding and should be well documented and discussed with the patient before surgery. Small differences in positioning of the newly created crease can be very obvious to both patient and surgeon. When recognized immediately after surgery, early revision should be considered. Other complications unique to Asian blepharoplasty include fold loss, suture extrusion, and epicanthal scarring. Lagophthalmos, which most often results from overresection of upper eyelid skin, is less frequent in Asian blepharoplasty because skin excision is typically more limited, particularly in a younger patient undergoing a partial incision technique. Fold loss may result from a technical error in securing the layers of dermis, epidermis, or levator aponeurosis, or from placing too few sutures. Revisional surgery would be required to correct this problem. Suture extrusion is not uncommon, given that permanent sutures are used for fixation. Meticulous placement of clear 7-0 nylon and trimming of suture ends will minimize the risk of this complication. If a suture becomes exposed in the first few months postoperatively, removal should be deferred until scarring is mature and fixation is more tenacious. Epicanthoplasty is commonly used is Asian patients with epicanthal folds undergoing blepharoplasty. Various techniques, including Y, W, and modified Z-plasties, have been reported. The epicanthal region is prone to hypertrophic scarring, particularly in Asians who have thick dermis.
2015 A 32-year-old Korean woman undergoes bilateral upper lid blepharoplasty for creation of a supratarsal crease. A partial incision technique is used. Which of the following is the most likely complication of this procedure? A ) Asymmetry B ) Epicanthal webbing C ) Fold loss D ) Lagophthalmos E ) Suture extrusion
The correct response is Option A. The subcutaneous fat of the abdomen is anatomically arranged in two layers: superficial and deep. The superficial adipose layer is located 1 to 2 mm below the dermis and is dense and compact with numerous septations. The deep adipose layer is loose and areolar with few septa. Conventional or traditional liposuction is performed within the deep adipose layer with larger cannulas. Superficial liposuction or subdermal liposuction involves the removal of fat from the superficial compartment found 1 to 2 mm below the dermis, disrupting the extensive septations. Most complications following liposuction are minor and resolve without further surgical intervention. However, the most common complications following superficial liposuction are contour irregularities. Less common complications associated with superficial liposuction include seroma, hyperpigmentation, infection, hypertrophic scar, chronic induration, skin necrosis, and infection.
2015 A 32-year-old woman is evaluated for lipodystrophy of the central abdomen with skin laxity. She desires volume reduction as well as improvement of the skin laxity. The patient is scheduled for superficial liposuction of the abdomen. Which of the following postoperative complications is most likely in this patient? A ) Contour irregularities B ) Hyperpigmentation C ) Infection D ) Seroma E ) Skin necrosis
The correct response is Option C. Diagnosis of rupture is difficult by physical examination alone, which is why the majority of ruptures are silent. Subsequent MRI screening for silent rupture is recommended initially 3 years postoperatively, then every 2 years thereafter. Highly cohesive, anatomically shaped, silicone-filled breast implants combine the "gummy bear" silicone with an anatomical shape, in which inferior pole projection is higher than the superior pole projection. In studies of Allergan's Natrelle 410 breast implants (the "Pivotal Study," the 410 Swedish MRI study, and the 410 European MRI study) approximately 3 in 100 women had silent ruptures. Cohesive gel is still subject to rupture, because rupture occurs when the shell fails. In cohesive implants, however, as opposed to noncohesive implants, the rupture rarely becomes extracapsular.
2015 A 32-year-old woman is scheduled to undergo augmentation mammaplasty with highly cohesive, anatomically shaped, silicone-filled breast implants. She asks the surgeon about postoperative monitoring for implant rupture. This patient should be counseled that, according to FDA recommendations, postoperative monitoring for rupture most appropriately includes which of the following? A ) Manual examination 3 years postoperatively, then annually thereafter B ) MRI screening 2 years postoperatively, then every 3 years thereafter C ) MRI screening 3 years postoperatively, then every 2 years thereafter D ) Ultrasonography screening 2 years postoperatively, then every 3 years thereafter E ) Ultrasonography screening 3 years postoperatively, then every 2 years thereafter
The correct response is Option E. In this patient, the most appropriate code is one that addresses the vestibular stenosis only: 30465. The more inclusive codes of 30410 and 30420 include surgery involving the lower lateral (alar) cartilages to address tip issues, and, in the case of 30410 and 30420, work on the bony pyramid as well; none of which are a component of the procedure undertaken, which is limited to addressing the internal nasal valve narrowing. CPT code 30520 is most appropriate to use for the septoplasty because the harvesting of cartilage is included in the submucous resection surgical field, and, as a result, cannot be coded separately as a septal cartilage graft harvest. If one were not performing a septoplasty, then the most appropriate code would be 20912 for the septal graft harvest to be used for the spreader grafts.
2015 A 35-year-old man is evaluated for a history of sleep apnea and nasal airway obstruction. Physical examination shows a narrowed middle vault with an internal valve of less than 10 degrees and a septum that deviates along the maxillary crest to the left, narrowing the airway. The operative plan is for spreader grafts from the septum, and submucous resection of the deviated portion of the septum. Which of the following is the most appropriate Current Procedural Terminology (CPT) coding for this procedure? CPT code Description 20912 Septal cartilage graft 30410 Rhinoplasty, primary; bony pyramid, lateral and alar cartilages and/or tip 30420 Rhinoplasty, primary; bony pyramid, lateral and alar cartilages and/or tip, including major septal repair 30465 Repair of nasal vestibular stenosis 30520 Septoplasty or submucous resection with or without cartilage scoring, contouring, or replacement with graft A ) 20912 and 30420 B ) 20912 and 30465 C ) 30410 and 30465 D ) 30420 and 30465 E ) 30465 and 30520
The correct response is Option D. The perfusion of the nipple-areola complex is a major concern during breast procedures involving periareolar and intraparenchymal incisions. The nipple-areola complex has a very rich and overlapping perfusion through multiple sources. This fact allows the design of various pedicles to carry the nipple and areola with different techniques. The blood supply through the internal thoracic vessels reaches the breast, nipple, and areola through the intercostal perforators, which may be divided during both subpectoral and subglandular implant placement. The location of the implant deep or superficial to the pectoralis muscle will not change the perfusion through the superficial epigastric vessels. The same is true for the blood supply through the lateral thoracic vessels. However, the flow through the thoracoacromial vessels to the breast parenchyma will be preserved by placement of the implant deep to the pectoralis muscle. Creation of a subglandular pocket above the muscle will interrupt the collaterals from the thoracoacromial vessels through the muscle to the parenchyma. The thoracodorsal artery is not a major source of blood supply to the breast and the position of the implant will not affect it.
2015 A 35-year-old woman comes to the office for consultation because she is dissatisfied with the appearance of her "deflated" and "saggy" breasts. Augmentation/mastopexy is planned. Compared with placement of the implant in the subglandular position, placement of the implant in the subpectoral space will preserve blood supply to the breast tissue and skin through which of the following arteries? A ) Internal thoracic B ) Lateral thoracic C ) Superficial superior epigastric D ) Thoracoacromial E ) Thoracodorsal
The correct response is Option E. Chemical peeling causes controlled destruction of parts of the epidermis and/or dermis, followed by regeneration of new dermal and epidermal tissues. In a controlled manner, a chemical peel induces injury at a specific depth of the skin. Peels are categorized as superficial, medium depth, or deep, depending on the level of injury. Superficial peels cause necrosis of the epidermis only. Medium-depth peels create a wound through the epidermis into the level of the upper reticular dermis. Deep peels penetrate to the mid reticular dermis. Salicylic acid 20 to 30% would cause injury to the stratum corneum and possibly the stratum granulosum with exfoliation. The depth is less than 100 µm, which is classified as superficial-very light. Both glycolic acid 50 to 70% and the Jessner solution penetrate to a depth of 100 µm, which is considered superficial-light. These agents cause necrosis of the entire epidermis down to the basal layer and stimulate regeneration of new epithelium. A medium-depth peel extends 200 µm, penetrates through the epidermis and papillary dermis to the upper reticular dermis, and results in increased collagen production. Trichloroacetic acid solution 35 to 50% would penetrate to medium-depth. A deep peel penetrates to deeper than 400 µm and causes necrosis to part or all of the mid reticular dermis. Careful preprocedure evaluation is imperative in choosing the appropriate peel for each patient. Fitzpatrick skin type must be assessed to determine the risk of post-peel complications. If a patient has had a recent medium or deep peel within the past 3 months, facial surgery with extensive undermining or isotretinoin therapy within the past 6 months, or a history of keloid scarring, then care must be taken when selecting medium-depth or deep peels because the risk of hyperpigmentation and/or permanent scarring is increased. Patients with mixed and dermal melasma are often difficult to treat because of the deeper pigment. A test spot is helpful in determining the patient's tolerance for the peel when there is concern about the potential adverse effects. Tretinoin 0.01% causes increased turnover of follicular epithelial cells and helps prevent collagen loss. It is not indicated for melasma.
2015 A 36-year-old woman with Fitzpatrick Type II skin is evaluated because of melasma that is refractory to hydroquinone therapy. A biopsy is performed, and examination of the specimen confirms the diagnosis of mixed melasma extending to the upper reticular dermis. Administration of which of the following is most appropriate to treat this area? A ) Glycolic acid 50 to 70% B ) Jessner solution C ) Salicylic acid 20 to 30% D ) Tretinoin E ) Trichloroacetic acid 35 to 50%
The correct response is Option A. The most severe and feared early occurring complication of soft-tissue filler agents is tissue necrosis, caused by interruption of the vascular supply to the area by either direct injury of the vessel, compression of the area around the vessel, or obstruction of the vessel by the filler material. It is a rare event, and although more commonly reported in the glabellar region, it has been reported following injection of the nasolabial fold area with hyaluronic acid gel and calcium hydroxylapatite (Radiesse) filler products, causing alar necrosis. Treatment options for impending necrosis are based on those recommended for the treatment of the glabella and remain anecdotal. Typically, if noted immediately, injections are halted, warm compresses are applied, and nitroglycerin paste is used for local vasodilatation. Immediate use of hyaluronidase to the injection site is also recommended. The use of hyperbaric oxygen is controversial. Recognition of the possible problem is essential, so that early intervention can minimize tissue necrosis and subsequent deformity. Ice would potentially worsen the already compromised blood flow to the area and is not recommended. Benzodiazepines and antihistamines do not treat the underlying problem, which is tissue ischemia.
2015 A 38-year-old woman receives an injection of hyaluronic acid gel fillers to improve the appearance of her nasolabial folds. The evening after she received the injection, the patient calls the answering service and reports to the surgeon that, several hours after the injection, she developed skin "irritation" on the left side of the nose with skin discoloration, swelling, and numbness. Which of the following is the most appropriate next step in management? A ) Evaluate the patient in person B ) Initiate treatment with an oral antihistamine C ) Initiate treatment with an oral benzodiazepine D ) Tell the patient to immediately apply ice E ) Reassurance
The correct response is Option A. A history of two late-term miscarriages stands out as a significant risk factor for thrombophilia (inherited and acquired) and subsequent deep vein thrombosis. It is imperative that these patients be identified and further evaluated. If abdominoplasty is performed, chemoprophylaxis is required. The most common inherited thrombophilia is factor V Leiden, which is present in 3 to 7% of the Caucasian population. Multiple inherited thrombophilic conditions can be present in the same individual. Studies have shown that complication rates in abdominoplasty increase in patients with a BMI greater than or equal to 30 kg/m2. Based upon history and physical examination alone, the other complications of hematoma, infection, recurrent diastasis, or wound dehiscence should not be significantly increased.
2015 A 39-year-old woman is evaluated because she is dissatisfied with the appearance of her abdomen. She has had five pregnancies with two full-term deliveries and three second-trimester miscarriages. She takes no birth control pills. Physical examination shows wide diastasis with excess abdominal skin. BMI is 28 kg/m2. Abdominoplasty with translocation of the umbilicus is planned. This patient is at increased risk for which of the following complications? A ) Deep vein thrombosis B ) Hematoma C ) Infection D ) Recurrent diastasis E ) Wound dehiscence
The correct response is Option A. Studies have shown the fibrocystic changes alone in the breast are not directly linked to an increased risk of breast cancer, so there is no indication for fine-needle aspiration or core biopsy. Cancer risk increases in benign breast disease with increased proliferation and atypical hyperplasia. Even with no family history of breast cancer, it is recommended that the patient continue routine mammograms. Fibrocystic breast disease has not been linked in the literature to an increased risk of mutations of the BRCA genes; therefore, genetic testing is unnecessary.
2015 A 39-year-old woman with a history of fibrocystic breast lesions comes to the office for consultation. She has no family history of breast cancer. Results of routine mammograms have been negative; she has never undergone biopsy. Which of the following is the most appropriate recommendation for this patient regarding managing her risk of breast cancer? A ) Continue to schedule routine mammograms B ) Refer for mastectomy C ) Schedule core needle biopsy D ) Schedule fine-needle aspiration E ) Schedule genetic testing
The correct response is Option E. Innervation of the anterolateral abdominal wall arises from the anterior rami of spinal nerves T7 to L1. These branches include the intercostal nerves (T7-T11), the subcostal nerve (T12), and the iliohypogastric and ilioinguinal nerves (L1). Because these nerves travel in the plane between the transversus abdominis and internal oblique muscles, they can be conveniently blocked in this area with a single transversus abdominis plane (TAP) block on each side. Although the other nerve block techniques are frequently used in combination, each covers a smaller territory or single dermatome. The TAP block may be performed via several different approaches. Most experts agree that there is a reliable block of the T10 to L1 dermatomes when the lateral approach from the triangle of Petit is used. The subcostal approach of the TAP block can give a more cephalad block. The combination of bilateral TAP blocks and rectus sheath injections has been found to decrease the need for postoperative narcotic use after abdominoplasty. It has also been useful for patients receiving transverse rectus abdominis musculocutaneous (TRAM) and deep inferior epigastric artery perforator (DIEP) flaps. The TAP block was also found to be superior to conventional ilioinguinal and iliohypogastric nerve blocks in a comparison study of open inguinal hernia repairs.
2015 A 40-year-old woman, gravida 2, para 2, with abdominal laxity and rectus diastasis is scheduled to undergo abdominoplasty with rectus plication. Which of the following intraoperative nerve blocks is likely to provide postoperative analgesia to the greatest area of lower abdominal skin for this patient? A ) Direct midline injection of plication area B ) Iliohypogastric nerve block C ) Ilioinguinal nerve block D ) Subcostal nerve block E ) Transversus abdominis plane block
The correct response is Option C. A relative contraindication to nipple-sparing mastectomy is a centrally located tumor. Although various authors have employed different distance criteria, it is generally accepted that patients whose tumors are within 2 cm of the nipple are not candidates for nipple-sparing mastectomy. Nipple-sparing mastectomy is an appropriate option for high-risk patients undergoing prophylactic mastectomy and for patients diagnosed with breast cancer who meet certain criteria. Those criteria are: tumor size of 3 cm or less, at least 2 cm from the nipple, not multicentric, and with clinically negative nodes. Comedo carcinoma of the breast is a type of ductal carcinoma in situ. It is considered to be an early stage of breast cancer, is confined to the ducts, and usually does not spread beyond. It is not a contraindication to nipple-sparing mastectomy. Invasive lobular carcinoma originates from the breast lobules, may form a thickening of the breast tissue rather than a discrete mass, and is often bilateral. As long as it meets the above criteria, it is not a contraindication to nipple-sparing mastectomy. Inflammatory breast cancer, Paget disease, and tumors infiltrating the skin are also not candidates for skin-sparing or nipple-sparing mastectomy, according to several authors. In more recent studies, a tumor size of 3 cm or less appears to result in no increase in local or regional recurrence in nipple-sparing mastectomy compared with alternative surgical approaches. A tumor of 2.5 cm is not a contraindication to nipple-sparing mastectomy.
2015 A 42-year-old woman comes to the office for treatment after receiving a diagnosis of cancer of the right breast. She has decided to undergo mastectomy of the right breast. Which of the following is a relative CONTRAINDICATION to nipple-sparing mastectomy? A ) Comedo-type breast tumor B ) Invasive lobular carcinoma C ) Subareolar tumor D ) Tumor location 3 cm from the nipple E ) Tumor size of 2.5 cm
The correct response is Option D. The breast cancer-susceptibility gene types 1 and 2 (BRCA1 and BRCA2) are tumor suppressor genes. Mutations in BRCA1 and BRCA2 are associated with hereditary breast and ovarian cancers. Additionally, they can be associated with increased risks of pancreatic and prostate cancer. Thyroid, lung, esophageal, and colon cancer are not associated with increased risks of BRCA1 and BRCA2 mutations.
2015 A 42-year-old woman with a 3-cm invasive ductal carcinoma of the right breast is evaluated for breast reconstruction. She has not decided how she wants to manage her contralateral breast. Regarding eliciting a family history, which of the following cancers is associated with a mutation in a breast cancer-susceptibility gene? A ) Colon B ) Esophageal C ) Lung D ) Pancreatic E ) Thyroid
The correct response is Option E. It is very common for nipples to be inverted at birth and concerned parents often discuss the condition with pediatricians and practitioners. The mammary ridge begins to develop in the fourth week of development, and produces the primary and secondary buds. The mammary pit is a small depression present at birth into which the lactiferous ducts open. After birth, proliferating mesoderm during the first several weeks of life will cause the inverted nipple to protrude. In some cases, the inverted nipple persists. Although there may not be a functional consequence, it can lead to difficulty with lactation. Han and Hong described a grading system for inverted nipple: grade I, in which the nipple can be made to protrude manually and can maintain position without traction; grade II, in which the nipple fails to maintain projection after manipulation; and grade III, in which the nipple cannot be pulled out manually. Treatment is based on the grade. In some cases, breast-feeding or use of a breast pump is sufficient to correct the variation. Surgical intervention can be approached in several ways. Constricting the base of the inverted nipple can create projection, severing the ducts can cause projection, and myotomy can cause projection. Any division of the ducts will compromise a patient's ability to breast-feed.
2015 A 43-year-old woman brings her 10-day-old daughter for evaluation because she is concerned about the appearance of her daughter's nipples. The newborn was born at full-term after an uncomplicated pregnancy. Physical examination shows that the newborn has inverted nipples bilaterally. Which of the following is the most appropriate next step in management? A ) Continuous elastic outside distraction starting at day of life 14 B ) Suction lengthening of the nipple starting at day of life 14 C ) Suction lengthening of the nipple starting immediately D ) Transection of the fibrotic bands at 6 months of age E ) Observation
The correct response is Option C. Over the past decade, several large retrospective studies have looked at the rate of developing breast cancer after reduction mammaplasty. The breast cancer rates in patients undergoing reduction mammaplasty have consistently been decreased by about 30%. This differs from prophylactic mastectomy, which lowers the rate by as much as 90%.
2015 A 45-year-old woman comes to the office because she is interested in having reduction mammaplasty. She asks if undergoing reduction mammaplasty would decrease her risk of breast cancer. Which of the following is the most appropriate response to this patient? A ) The rate of breast cancer has been shown to be slightly increased in patients who undergo reduction mammaplasty B ) Reduction mammaplasty appears to decrease the rate by 90% C ) Reduction mammaplasty decreases the rate of breast cancer but less than prophylactic mastectomies D ) There is no evidence that reduction mammaplasty decreases cancer risk
The correct response is Option A. In the augmented breast, the tissues of the lower pole become attenuated, compromising its blood supply. Both the weight of a large implant and the skin stretch caused by implant volume causes thinning and stretching of the lower pole tissues. The long distance from her nipple to inframammary crease provides further evidence that this has occurred. The other pedicles obtain their blood supply from perforator vessels preserved with a subpectoral pocket. Furthermore, the medial and lateral skin flap circulations are less affected by the weight of the implant as is the lower pole. The inferior pedicle can be used in these situations; however, added caution needs to be taken.
2015 A 45-year-old woman who underwent augmentation mammaplasty 20 years ago comes to the office for a mastopexy to correct her "sagging breasts." The 450-mL smooth, round saline implants were placed in a subglandular plane through a transaxillary approach. Mastopexy with implant exchange to 350-mL silicone implant is planned. Which of the following pedicles is most likely to present the greatest risk of vascular compromise of the nipple-areola complex in this patient? A ) Inferior B ) Lateral C ) Medial superior D ) Superior E ) Superior lateral
The correct response is Option D. Although it may seem aggressive to have patients sign a financial policy before receiving treatment stating that if there are any matters in dispute after the fact, they waive their right to privacy under the Health Insurance Portability and Accountability Act of 1996 (HIPAA) guidelines, this is the only way in which a case like this may be successfully handled. Without such protection, no information (including the fact that the person was even a patient) may be shared with a credit card company, another patient, the police, or a collection agency because it would be a violation of HIPAA, even if appropriate informed consent were obtained. Calling a patient with any kind of threat about a financial matter should never be done because this would violate ethical standards. Without a patient specifically waiving his/her right to privacy under HIPAA in a signed document, no information about that patient may be shared with the credit card company, and the physician is unable to confirm that the patient is indeed under his care. HIPAA affords complete protection of a patient's privacy, and, in situations such as these, to the great disadvantage of physicians who accept credit cards for payment. Any dispute cannot be properly addressed by the physician without a patient signing away these rights specifically.
2015 A 46-year-old woman receives several thousand dollars' worth of injectable filler and neurotoxin treatments following an initial consultation with the plastic surgeon. Before receiving the injections, she indicated that she understood the planned treatments and was made aware of the costs. She provided informed consent and paid for the services with a credit card. The plastic surgeon calls the patient for follow-up several weeks after the treatment, and the patient reports being happy with the results. Several months later, the plastic surgeon receives a notice from the credit card company that the patient is now disputing the charges, and the credit card company promptly removes the fees from the plastic surgeon's account. Which of the following is the most appropriate action by the plastic surgeon in response to this situation? A ) Call the patient and advise her that her charges will be turned over to the police for fraud B ) Call the patient who referred her and ask if she could encourage this patient to reverse the charges C ) Contact a lawyer to consider legal action against this patient D ) Ensure that all future patients sign a policy in advance of any treatment that waives their privacy protection under HIPAA in matters of dispute of payment E ) Send documentation to the credit card company of the treatments the patient received and proof that she had signed informed consent
The correct response is Option E. Anatomically, the orbicularis oculi muscle is divided into three segments: pretarsal, preseptal, and orbital. However, functionally, the orbicularis oculi muscle is divided into the medial inner canthal orbicularis and the extracanthal orbicularis. The medial inner canthal orbicularis is responsible for blinking, lower lid tone, and the pumping mechanism of the lacrimal system. Innervation to the inner canthal orbicularis is from the buccal branches of the facial nerve. The zygomatic branch of the facial nerve innervates the extracanthal orbicularis, which controls eyelid closure, voluntary squinting, and animation. The temporal, marginal mandibular, and cervical branches do not provide innervation to the orbicularis oculi muscle.
2015 A 49-year-old woman is evaluated because of a traumatic laceration of the right lower eyelid and cheek. Physical examination shows difficulty with eyelid closure, voluntary squinting, and animation. Which of the following branches of the facial nerve is most likely injured? A ) Buccal B ) Cervical C ) Marginal mandibular D ) Temporal E ) Zygomatic
The correct response is Option A. Patients coming to the office for cosmetic blepharoplasty must always be evaluated for eyelid ptosis before the procedure. Although older patients often have dermatochalasis, fat herniation, and laxity of the orbicularis muscles, upper eyelid ptosis is frequently present as well. Failure to diagnose and address the ptosis at the time of blepharoplasty surgery will often result in patient dissatisfaction, because the ptosis is often more apparent after debulking the upper lid. A detailed history and physical examination will show both the presence and often the cause of the blepharoptosis. Although involutional ptosis is the most common age-related form of ptosis, myogenic, neurogenic, and mechanical causes must also be ruled out. The Tensilon test is done when an acquired ptosis caused by myasthenia gravis is suspected. In adults, 1 mg of neostigmine is injected intramuscularly. The ptosis improves in 5 to 15 minutes if myasthenia gravis is the cause. The phenylephrine test is performed to determine if a posterior approach will be adequate for correction of the ptosis, as in patients who have mild ptosis secondary to Horner syndrome. With this test, application of phenylephrine 2.5% drops initially, or after 3 to 5 minutes, stimulates the Müller muscle and results in 2 to 3 mm of lid elevation. Physical examination of the eyelids should include measurements of the palpebral fissure height, the marginal reflex distance, levator function or excursion, scleral show, lid crease height and contour, superior sulcus depth and contour, lagophthalmos, Bell phenomenon, ocular dominance, and eyelid laxity. The marginal reflex distance 1 (MRD1) is best at determining the degree of ptosis present. With the patient in primary gaze, a light is shined on the eyes and a corneal light reflex is seen mid pupil. The distance of the light reflex to the upper lid margin is the MRD1. This measurement is more accurate than vertical palpebral fissure height, because lower lid position can affect the latter measurement. A normal MRD1 is between 3.5 and 4.5 mm. A MRD1 less than 2.5 mm is considered clinically significant ptosis.
2015 A 55-year-old woman comes to the office for evaluation for blepharoplasty. She says that her eyelids look heavy and give her a tired appearance. She has no history of ophthalmologic issues. Which of the following elements on physical examination is most appropriate for grading ptosis in this patient? A ) Measurement of marginal reflex distance 1 B ) Measurement of palpebral fissure height C ) Measurement of superior sulcus depth D ) Phenylephrine test E ) Tensilon test
The correct response is Option B. This patient's symptoms relate to an injury to the spinal accessory nerve (cranial nerve XI). This nerve innervates the sternocleidomastoid and the trapezius muscles. The spinal accessory nerve exits the cranium through the jugular foramen. It then passes deep to the styloid process and under the sternocleidomastoid muscle. The nerve exits the posterior border of the sternocleidomastoid fascia within 2 cm superior to the great auricular nerve. After it exits the muscle, the nerve is vulnerable to injury because it is tightly sandwiched between the skin and the muscle fascia. It then runs obliquely and inferiorly to the anterior edge of the trapezius muscle. The course of the spinal accessory nerve usually follows a path drawn by a line perpendicular to and bisecting a line connecting the angle of the mandible and the tip of the mastoid process. Lore fascia is a dense tissue inferior to the auricle that can be used to anchor the superficial musculoaponeurotic system fascia. The facial nerve is 2.5 cm deep to this fascia. McKinney point is where the great auricular nerve consistently crosses the mid transverse belly of the sternocleidomastoid muscle approximately 6.5 cm below the caudal edge of the bony external auditory canal. The spinal accessory nerve is posterior to the platysma.
2015 A 55-year-old woman has pain and weakness of the shoulder, and inability to lift her shoulder girdle 2 weeks after undergoing rhytidectomy, advancement of an extended superficial musculoaponeurotic system flap, and plication of the platysma with complete transection. A nerve injury is suspected. During which of the following parts of the rhytidectomy was the nerve most likely injured? A ) Anchoring the superficial musculoaponeurotic system flap to Lore fascia B ) Dissection of the lateral neck C ) Flap elevation at McKinney point D ) Subplatysmal fat resection E ) Transection of the platysma
The correct response is Option B. Along with ocular hazards and fire hazards, laser smoke plume is a significant occupational hazard, which is often ignored when lasers are used. There are numerous substances, some carcinogenic and mutagenic, released during laser pyrolysis of tissue. Viable skin bacteria, including coagulase-negative Staphylococcus, Corynebacterium, and Neisseria, have been recovered from the laser plume following laser skin resurfacing. In addition, intact viral DNA, particularly of human papillomavirus, has been isolated from carbon dioxide laser plume. Most surgical masks only filter particles that are 5 microns in diameter or larger; however, 77% of particles in the laser plume are 1 micron or smaller. Therefore, well-fitted high-filtration or laser masks should be used instead of standard surgical masks. A high-efficiency smoke evacuator should also be used, but it needs to be within 1 to 2 cm of the laser smoke plume source. The effectiveness of the smoke evacuator is decreased from 99 to 50% as the distance from the laser-treated site is increased from 1 to 2 cm, so 20 to 25 cm away is ineffective.
2015 A 55-year-old woman with moderate sun damage to facial skin and facial wrinkles comes to the office for laser skin resurfacing. Which of the following is the most appropriate management regarding respiratory protection from the laser smoke plume? A ) High-efficiency smoke evacuator placed 20 to 25 cm away from the site of the laser plume B ) High-efficiency smoke evacuator placed within 1 to 2 cm of the smoke plume C ) Standard surgical mask alone D ) Standard wall suction E ) No specialized equipment or protection is necessary
The correct response is Option C. Recent articles have greatly improved our understanding of the anatomy of the lower eyelid, tear trough, pre-zygomatic space, and the ligaments in the orbital area. The tear trough ligament is a true osteocutaneous ligament between the palpebral and orbital portions of the muscle. It extends inferolaterally from the medial canthus to approximately the mid-pupillary line, where is connects with the bilayered orbicularis retaining ligament. Recent anatomic dissection work has shown that this ligament is the anatomic basis of the tear trough deformity. Fillers should be placed inferior to the tear trough ligament; placing them superiorly will only serve to emphasize lower eyelid fat, and will emphasize the tear trough deformity.
2015 A 60-year-old man comes to the office because he desires improvement in the appearance of his lower eyelid and upper cheek area. Physical examination shows a prominent tear trough. Which of the following is the anatomic basis of the tear trough? A ) Attachment of the orbital septum to the arcus marginalis B ) Cleft between the palpebral and orbital parts of the orbicularis oculi C ) Osteocutaneous ligament arising from the medial portion of the maxilla D ) Prominence of the orbital rim following descent of the malar fat pad E ) Triangular confluence of the origins of the orbicularis oculi, levator labii superioris alaeque nasi, and levator labii superioris
The correct response is Option D. Gynecomastia is defined as the benign proliferation of male glandular breast tissue. The most common symptom of gynecomastia is painless breast enlargement in adolescent and elderly men. Examination of a breast biopsy specimen may be necessary to exclude breast cancer if mammography and/or breast ultrasonography are suggestive of malignancy. Microscopic findings include ductal epithelial hyperplasia with proliferation of stroma and fibroblasts. Pseudogynecomastia is defined as breast enlargement secondary to fat deposition without glandular proliferation. Fibroadenoma are the most common benign tumor of the female breast. They are most often diagnosed in women between the ages of 20 and 35 years. Fibroadenomas are often ovoid or spherical, freely movable, and often well circumscribed. Microscopic findings include ductal tissue with proliferation of stroma surrounded by fibroblasts. Ductal carcinoma accounts for the majority of breast cancers. Ductal carcinoma is characterized microscopically by cords and nests of tumor cells with varying amounts of gland formation, and cytologic features that range from bland to highly malignant. Epidermal cysts are inclusion cysts lined by well-differentiated epidermis filled with keratin. Lymphangiomas of the breast are distended lymphatic channels interspersed with breast lobules.
2015 A 62-year-old man is evaluated for an 8-month history of fullness of the right breast associated with a subareolar mass. He reports no history of pain, nipple discharge, skin changes, or systemic symptoms. There is no family history of breast cancer. Examination of a specimen obtained on biopsy shows ductal epithelial hyperplasia with proliferation of stroma and fibroblasts. Which of the following is the most likely diagnosis? A ) Ductal carcinoma B ) Epidermal cyst C ) Fibroadenoma D ) Gynecomastia E ) Lymphangioma
The correct response is Option B. Percutaneous biopsy methods are commonly accepted for the initial evaluation of clinically occult breast lesions, although certain nonmalignant lesions pose dilemmas with respect to the most appropriate clinical management. Papillary lesions of the breast can either be benign or malignant, although differentiation is radiologically difficult. Moreover, it is difficult for pathologists to reliably distinguish among benign, atypical, and malignant papillary lesions on the limited fragmented tissue specimens they receive after needle sampling. Previous studies have demonstrated high rates of ductal carcinoma in situ (11%) in patients diagnosed with benign papillomas by needle biopsy and who subsequently underwent a surgical excision, although conflicting data suggest an extremely decreased rate of malignancy when histology is benign on needle biopsy. The management of benign papillary lesions is somewhat controversial. Although conservative follow-up with either yearly mammogram or short-interval follow-up may be appropriate for certain patients diagnosed with benign papilloma, certain features of this patient's lesion make conservative follow-up inappropriate. Sonographic follow-up in a 65-year-old woman with mature breast parenchyma and a solid mammographically detected mass would not provide much additional information, and a repeat percutaneous biopsy, whether core needle or vacuum-assisted, would also not be effective. Given the size of the lesion and the age of the patient, surgical excision is warranted despite the lack of atypia on needle biopsy. Benign papillomas tend to be smaller than 1 cm and centrally located, whereas malignant lesions are more often greater than 1.5 cm and are peripherally located.
2015 A 65-year-old woman comes to the office 1 month before a scheduled mastopexy. Annual mammography shows a 1.5-cm mass in the upper outer quadrant. Core needle biopsy is performed. Pathologic examination of excised tissue identifies papilloma without atypia. Which of the following is the most appropriate next step in management? A ) Bilateral breast sonography B ) Excisional biopsy of needle-localized area C ) Repeat annual mammography in 12 months D ) Repeat mammography at 6-month intervals for 1 year E ) Stereotactic vacuum-assisted biopsy
The correct response is Option C. Tears are a trilaminar fluid. The precorneal layer is formed by mucin-secreting goblet cells in the conjunctiva. This inner layer of the tear film covers the cornea and promotes the dispersion of the overlying aqueous layer. The meibomian glands produce the outer lipid layer. This oil layer helps to prevent the evaporation of the tear film. The lacrimal gland secretes the middle layer. This aqueous layer is made of water and proteins. This layer promotes osmotic regulation and the control of infectious agents. As a result, dysfunction can result in dryness and an increase in infections.
2015 A 65-year-old woman has inadequate tear secretion because of a poorly functioning lacrimal gland. Which of the following aspects of this patient's tear film is most likely to be affected? A ) The amount of lipid in the tear film B ) The amount of mucin in the tear film C ) The antimicrobial property of the tear film D ) The degree of evaporation of the tear film E ) The dispersion of the tear film
The correct response is Option B. The popularity of brachioplasty has significantly increased in America over the past decade, in large part because of the number of patients undergoing bariatric surgery for morbid obesity. In this population of patients who have undergone massive weight loss, the severity of excess upper extremity skin mandates a long, often hypertrophic scar in the bicipital groove, which is generally accepted by patients. By contrast, older patients with skin laxity but little lipodystrophy and no history of significant weight change are hard-pressed to accept this visible and often unpredictable scar. For this population, minimal-incision brachioplasty has emerged as an excellent solution and has itself been increasing in popularity over the past few years. In minimal-incision brachioplasty, incisions are limited to the axilla. The procedure is usually combined with suction lipoplasty to remove some excess upper extremity fat, to treat dog ears at the proximal and distal extent of the scar, or to facilitate undermining. Several key maneuvers improve the appearance of the scar and are tantamount to achieving high patient satisfaction after this procedure. The benefit of anchoring the superficial fascial system was first recognized by surgeons performing traditional brachioplasty procedures. A similar concept applies to minimal-incision brachioplasty, where anchoring of the arm and axillary dermis to the superficial fascia is seen to have several advantages. As part of a layered closure, this technique distributes tension in a more even and controlled manner, releasing the high tension on the final skin closure and decreasing the risk of a widened scar. Moreover, these sutures close the dead space within the axilla and recreate the axillary hollow. Suturing the arm dermis to the axillary dermis without fascial reinforcement results in a high-tension closure and often a widened, hypertrophic scar. Suturing the skin to the deeper fascial layers would be difficult and deforming.
2015 A 66-year-old woman undergoes minimal-incision brachioplasty to treat bilateral upper extremity skin laxity. Height is 5 ft 6 in (167.6 cm) and weight is 140 lb (63.5 kg). BMI is 22.6 kg/m2. Which of the following suturing techniques is most appropriate to minimize widening of the scar? A ) Arm dermis to axillary dermis B ) Arm dermis to axillary dermis to axillary fascia C ) Arm dermis to axillary dermis to pectoralis major fascia D ) Axillary dermis to lateral pectoralis major tendon E ) Axillary dermis to superficial pectoralis minor fascia
The correct response is Option D. The final report, in conjunction with the Department of Health in Australia, has shown a 2 to 6 times increased rupture rate in Poly Implant Prothèse (PIP) implants, which is detectable within 5 years of implantation. Increased levels of siloxane have been detected, but are not considered a health risk. No organic impurities have been detected and platinum levels are decreased in PIP gel compared with medical grade silicone. There is no increased breast cancer risk and no evidence of cytotoxicity. In the light of the increased rupture rate and the nonmedical grade nature of PIP silicone gel, the following recommendations were made: · all providers of breast implant surgery should contact any women who have or may have PIP implants, if they have not already done so, and offer them a specialist consultation and any appropriate investigation to determine if the implants are still intact; · if the original provider is unable or unwilling to do this, a woman should seek referral through her general practitioner to an appropriate specialist; · if there is any sign of rupture, she should be offered an explantation; · if the implants still appear to be intact, she should be offered the opportunity to discuss with her specialist the best way forward; · if, in the light of this advice a woman decides with her specialist that, in her individual circumstances, she wishes to have her implants removed, her health care provider should support her in carrying out this surgery. Where her original provider is unable or unwilling to help, the NHS will remove, but not normally replace, the implants; · if a woman decides not to seek early explantation, she should be offered annual follow up in line with the advice issued by the specialty surgical associations in January 2012. Women who make this choice should be encouraged to consult their doctor if they notice any signs of tenderness or pain, or swollen lymph glands in or around their breasts or armpits, which may indicate a rupture. At the first signs of rupture, they should be offered removal of the implants.
2015 A French woman, who underwent placement of Poly Implant Prothèse (PIP) gel implants in 2009, comes to the office for consultation because she had heard that the implants were filled with a nonmedical grade silicone. She reports that she has not had any problems with the implants, but would like to know the implications of retaining the implants and whether she should have them removed. This patient should be told that she is at increased risk for which of the following complications if she retains the implants? A ) Breast cancer B ) Cytotoxicity C ) Heavy metal poisoning D ) Implant rupture E ) Siloxane poisoning
The correct response is Option E. The three most common causes for adolescent unilateral breast enlargement are giant fibroadenoma, phyllodes tumor (previously called cystosarcoma phyllodes), and juvenile breast hyprterophy. Differentiation between phyllodes tumor and giant fibroadenoma on core needle biopsy is difficult. Phyllodes tumors are fibroepithelial tumors and stromal derived. The stromal component can appear similar to a fibroadenoma, and this similarity can make the two difficult to distinguish; in some cases, the stromal component resembles a soft-tissue sarcoma. Core needle biopsy is performed for diagnosis, and phyllodes tumors typically have increased cellularity, mitosis, and stromal overgrowth when compared with fibroadenomas. Phyllodes tumors of the breast represent approximately 3% of breast neoplasms. They are classified as benign, borderline, or malignant. Wide excision with 1-cm margins is recommended for all classifications. The extent of resection is determined by the grade, and the grade is associated with the risk of local recurrence. Margin-negative, breast-conserving therapy is appropriate for benign phyllodes tumors. Malignant phyllodes tumors behave more similarly to sarcomas than to other types of breast cancer. Although there is controversy about the role of radiation therapy, this modality is less effective than surgery, and is reserved for margin-positive, malignant tumors. Chemotherapy is not indicated in this patient. The greatest risk with benign phyllodes tumors is local recurrence, and overall, the prognosis of these tumors is considered excellent. The only known condition associated with the development of phyllodes tumors is Li-Fraumeni syndrome.
2015 An otherwise healthy 17-year-old nulliparous girl is evaluated because of significant breast asymmetry. She has noted an increase in the size of her left breast over the past 2 months. Current medication includes a combined oral contraceptive. The patient undergoes mammography and biopsy. Pathologic examination of excised tissue shows a benign phyllodes tumor. Which of the following is the most appropriate next step in management? A ) Enucleation B ) Mastectomy with sentinel lymph node biopsy C ) Progestin-only oral contraceptive D ) Tamoxifen followed by lumpectomy E ) Wide local excision
The correct response is Option E. Thiamine deficiency is most often identified shortly after bariatric surgery but can be diagnosed later. Some patients can develop Wernicke-Korsakoff encephalopathy (WKE). Body stores of thiamine can last from 3 to 6 weeks, and thiamine deficiency is more associated with decreased dietary intake. Although clinical manifestations are very uncommon, and WKE is considered a rare complication, approximately 11% of patients who have undergone Roux-en-Y gastric bypass surgery and take vitamin supplementation show evidence of thiamine deficiency 2 years postoperatively. The hallmark of thiamine deficiency is neurologic symptoms, but in contrast to WKE, patients rarely exhibit confusion, ataxia, and oculomotor abnormalities. If thiamine deficiency is not recognized and treated, it can have devastating results, including irreversible brain damage and death. Full nutritional workup of patients is critical. Confusion is a symptom of diabetic ketosis, but diabetic ketosis is notable for signs of dehydration and excessive thirst or urination, and is associated with Kussmaul respirations. Acute thyroiditis has symptoms of pain and swelling of the anterior neck. Pulmonary embolism can have symptoms of respiratory distress and right-sided heart strain. Early pregnancy often results in nausea, but not lethargy and confusion
2015 An otherwise healthy 41-year-old woman who underwent Roux-en-Y gastric bypass surgery 24 months ago, followed by a 120-lb (54-kg) weight loss that she maintained for 6 months, undergoes plastic surgery evaluation for a panniculectomy. Medical history includes hypothyroidism that is controlled with levothyroxine. Preoperative cardiovascular examination shows no abnormalities, and results of a pregnancy test on the day of surgery are negative. She undergoes panniculectomy and thigh lift, and on extubation, the patient is lethargic and confused. ECG shows sinus tachycardia, and she remains somnolent and confused. Analysis of thyroid-stimulating hormone and cardiac enzymes, chest x-ray study, and ventilation-perfusion scan show no abnormalities. Which of the following is the most likely diagnosis? A ) Acute thyroiditis B ) Diabetic ketosis C ) Pulmonary embolism D ) Undiagnosed pregnancy E ) Vitamin B1 (thiamine) deficiency
The correct response is Option B. Because lidocaine absorbs slowly from fat, infiltrate solutions that contain up to 35 mg/kg of lidocaine are generally considered safe. Nonetheless, lidocaine toxicity is still a risk of the procedure. In tumescent solution with epinephrine, peak plasma lidocaine levels occur approximately 10 to 14 hours after infiltration, and thus, the presentation 6 hours after discharge is consistent with peak plasma concentration. Lidocaine toxicity has symptoms of neurologic or cardiac toxicity. In the early stages, the complications are primarily neurologic and can include slurred speech, restlessness, tinnitus, and a metallic taste, as well as numbness of the mouth. As the concentrations increase, the neurologic concentrations become more severe, and can progress to muscle twitching, seizures, and cardiac arrest. Treatment of lidocaine toxicity is supportive. Fat embolism presents as a petechial rash, respiratory dysfunction, and cerebral dysfunction, and the symptoms usually appear 24 to 48 hours after surgery. Pulmonary embolism presents as leg pain and edema, tachycardia, and low-grade fevers. Parietal strokes usually cause sensory symptoms, self-perception anomalies, and left-right agnosia. Third spacing refers to fluid shifts into interstitial spaces and can cause edema, hypotension, and decreased cardiac output.
2015 An otherwise healthy 52-year-old woman with a family history of cardiac disease undergoes suction-assisted lipectomy of the flanks, thighs, and abdomen using a tumescent technique. She returns to the emergency department 6 hours after discharge because of slurred speech and restlessness. Which of the following is the most likely diagnosis? A ) Fat embolism B ) Lidocaine toxicity C ) Parietal stroke D ) Pulmonary embolism E ) Third spacing
The correct response is Option D. Reduction mammaplasty with a superior pedicle that involves resection of the tissue at the base of the breast is associated with a higher risk of injury to the nerve branches that innervate the nipple-areolar complex. Innervation from the lateral cutaneous branches runs deep within the pectoral fascia before sharply turning in an anterior direction to innervate the nipple from its deep aspect. The lateral pedicle, inferior pedicle, and inferocentral pedicle save the tissue containing the lateral cutaneous branches, decreasing the chance for injury to this nerve and reduced nipple sensation. The superomedial pedicle preserves the anterior cutaneous branches which run superficially from the medial aspect of the breast, and also provide sensation to the nipple. The superior pedicle resects both the medial and lateral innervations to the nipple.
2015 Reduction mammaplasty using which of the following pedicles has the greatest risk of altered nipple sensation? A ) Inferior B ) Inferocentral C ) Lateral D ) Superior E ) Superomedial
The correct response is Option B. Component separation for closure of large abdominal wall defects was first described by Ramirez in 1990. The purpose of the surgery is to achieve abdominal wall closure with well-vascularized, innervated muscle flaps. The primary vascular supply to the rectus muscles are the deep inferior epigastric artery and vein, which arise from the external iliac vessels. The internal mammary vessels give rise to the superior epigastric arteries and veins, which is a secondary, nondominant vascular supply of the rectus muscles. The femoral vessels give rise to the superficial inferior epigastric artery and vein, which perfuse the skin and subcutaneous fat of the inferior lateral abdomen.
2015 The dominant vascular supply of the rectus abdominis muscle originates from which of the following vessels? A ) Common femoral B ) External iliac C ) Internal iliac D ) Internal mammary E ) Superficial femoral
The correct response is Option E. The intercartilaginous incision follows the caudal border of the upper lateral cartilage and is located between it and the cephalad border of the alar lateral crus. This incision may connect, and frequently does, with a transfixion incision at the caudal border of the septum at the septal angle.
2015 The intercartilaginous incision in rhinoplasty follows the caudal border of which of the following? A ) Alar lateral crus B ) Caudal septum C ) Lower lateral cartilage D ) Middle crus E ) Upper lateral cartilage
The correct response is Option D. Nipple-sparing mastectomies (NSMs) are becoming more common for both therapeutic and prophylactic mastectomies. Nipple-areola complex (NAC) necrosis can imperil reconstructive efforts, as well as negatively affect patients emotionally. It is important to maximize perfusion to the mastectomy skin flaps and NAC while still performing an oncologically sound procedure. There are multiple different incisions for performing NSM. Periareolar, inframammary-fold, radial, and vertical incisions are the most common. Periareolar incisions are associated with an increased risk of NAC necrosis in NSMs. Type of reconstruction, small breast size, and patient age have not been shown to be linked to increased rates of NAC necrosis.
2015 Which of the following characteristics is correlated with increased risk of nipple-areola complex necrosis in nipple-sparing mastectomies with immediate reconstruction? A ) Autologous tissue reconstruction B ) Direct to implant reconstruction C ) Patient age D ) Periareolar incision E ) Small breast size
The correct response is Option D. Carbon dioxide laser treatments can cause operating room fires. Several papers have shown that a nasopharyngeal oxygen delivery can decrease oxygen levels in the operative field when it is required, but the best way to decrease the level of oxygen on the field is not to use it. Foot pedals can be accidentally activated and should be avoided. Clamping the laser cord can damage the fibers and ignite the laser fiber sheath. Laser skin surfacing can be accomplished without intubation.
2015 Which of the following is most effective in decreasing the risk of fire when using a carbon dioxide laser for facial resurfacing? A ) Clamp the laser cord to surgical drapes B ) Intubate laser patients to prevent oxygen accumulation on the field C ) Provide supplemental oxygen with a nasopharyngeal cannula D ) Use conscious sedation, nerve blocks, and no supplemental oxygen E ) Use foot pedals only for activating the laser
The correct response is Option E. A common complication of the "donut" (circumareolar) mastopexy is widening of the areola. This can be minimized by using a Gore-Tex suture placed using the "wagon-wheel" technique and limiting the amount of skin resected to a 2:1 ratio of outside diameter to areolar diameter. Boxy breast shape is associated with Wise pattern mastopexy. Nipple necrosis is associated with combined augmentation and mastopexy. Increased distance from the nipple to the inframammary fold is associated with vertical mastopexies in which the height of the medial and lateral pillars is too tall. Loss of nipple sensitivity is unusual because there is no parenchymal resection.
2015 Which of the following is the most common complication associated with "donut" mastopexy? A ) Boxy breast shape B ) Increased distance from nipple to inframammary fold C ) Loss of nipple sensation D ) Nipple necrosis E ) Widening of the areola
The correct response is Option B. In general, patients are satisfied with the results of brachioplasty. Complications are not uncommon and some studies report complication rates as high as 50%. Most of these complications are minor in nature. Wound dehiscence is managed with dressing changes. Hematoma and seromas are typically managed with observation or aspiration. In the case of infections, the majority are managed with oral antibiotics. The most common reason for additional surgery after brachioplasty is to revise the scar. Many patients will have an unfavorable scar that either hypertrophies or widens. This can be addressed by a scar revision, laser treatment, or triamcinolone acetonide (Kenalog) injection.
2015 Which of the following is the most common reason for reoperation after brachioplasty in patients who have undergone significant weight loss? A ) Hematoma B ) Hypertrophic scar C ) Infection D ) Seroma E ) Wound dehiscence
The correct response is Option B. There are a number of uncommon aplastic deformities of the breast. These include: total absence of the breast and nipple (amastia), absence of the nipple (athelia), and absence of the mammary gland (amazia), as described in this case. These anomalies may occur in isolation, or may be associated with various syndromes, such as Poland syndrome, where the absence of the breast is associated with absence of the pectoralis major muscle, rib cage and ipsilateral upper limb deformities. Ectodermal dysplasias can affect the breast, but two or more abnormalities of ectodermal structures - hair, teeth, nails, sweat glands, craniofacial structures - would be required to consider the diagnosis.
2016 A 13-year-old girl is evaluated for breast asymmetry. Examination shows total absence of the left mammary gland tissue, with normal areola and nipple. Pectoral muscles are normal. No hand, facial, or other body abnormalities are noted. Which of the following is the most likely diagnosis? A ) Amastia B ) Amazia C ) Athelia D ) Ectodermal dysplasia E ) Poland sequence
The correct response is Option C. Common hallmarks of tuberous breast deformity include varying degrees of hypoplastic breast parenchyma, deficiencies of the inferior pole, herniation of the parenchyma in the areola, enlarged areolae, superior placement of the inframammary fold, and asymmetry. Surgical goals are to achieve symmetry, sufficient volume (especially in the hypoplastic areas), lowering of the inframammary fold, reduction of areolar tissue herniation, and correction of any ptosis. A double-bubble deformity can occur when the inframammary fold is not sufficiently obliterated. The risk for this is increased with superiorly displaced inframammary folds, as in tuberous breasts. Parenchymal scoring would both release any constricting bands to allow the lower pole tissue to spread over the implant as well as release the superiorly displaced inframammary fold. While decreasing the areolar diameter and lowering of the inframammary fold are goals for breast improvement, neither will treat a doublebubble deformity. A periareolar incision is often advocated in repair of tuberous breasts because of the ability to reduce the areola; it alone, however, will not prevent a double-bubble deformity. Subpectoral placement of implants increases the risk for double-bubble deformity while subglandular placement of implants decreases the risk. Many advocate a dual-plane approach to capitalize on increased upper pole coverage combined with the benefits of a subglandular relationship in the inferior pole.
2016 A 22-year-old nulliparous woman is evaluated for improvement of breast shape and size. Examination shows bilateral hypoplastic breasts with constricted bases and herniation of breast parenchyma in the areolae. Tuberous breast deformity is diagnosed. Bilateral breast augmentation with smooth, round gel implants via periareolar incisions is planned. Which of the following maneuvers is most likely to decrease the risk for a "double-bubble" deformity? A ) Decreasing the areolar diameter B ) Lowering of the inframammary fold C ) Parenchymal scoring D ) Periareolar incision E ) Subpectoral placement of the implant
The correct response is Option E. On anterior/posterior view, the alar margin and columella have been described as ideally having the appearance of a gentle gull wing in flight. The columella represents the body of the gull and in this patient it is noted to be elongated with the wings (alar margin) appearing to be normal. On lateral view, the nostril should have an oval shape. A line drawn along the long axis should bisect it into equal halves with the alar rim being 1 to 2 mm above this line and the columella 1 to 2 mm below. The patient described exhibits findings consistent with a hanging columella. The etiology of this problem is due to either a long caudal septum, long medial crura, or combination of the two. The transfixion incision is in the membranous septum at the border of the caudal septum. It is the only incision listed above that allows access to the caudal septum for excision and can also resect any redundant membranous septum that may develop as a result of the setback. An intracartilaginous incision is made within the substance of the lateral crus of the lower lateral cartilage. It can be used in a closed approach to combine the incision for the access to the nose with the removal of the cartilage superior to the incision to accomplish a cephalic trim of the lower lateral cartilage. A Killian incision is used for access to the septum. It is placed 1 to 2 cm posterior to the caudal border of the septal cartilage. An alar rim incision is made in the vestibular skin just inside the border of the nostril. This incision can be used as an approach to create a pocket for a nonanatomical rim graft. The intercartilaginous incision is made between the upper and lower lateral cartilages and would not improve columellar show.
2016 A 22-year-old woman is evaluated for revision rhinoplasty. A closed approach is planned. The alar margin is in an appropriate position. On lateral view, excessive columellar show is noted. Which of the following incisions is most appropriate for management of this deformity? A ) Alar rim B ) Intercartilaginous C ) Intracartilaginous D ) Killian E ) Transfixion
The correct response is Option C. This patient is presenting with symptoms of salicylism or salicylic acid toxicity, a rare side effect of salicylic acid peels. Symptoms can include: rapid breathing, tinnitus, hearing loss, dizziness, abdominal cramps and central nervous system reactions. It is more likely to occur when large surface areas are peeled. It has been reported with 20% salicylic acid applied to 50% of the body surface, and lesser areas when stronger concentrations are used. Therefore, care should be taken when treating skin conditions that cover large surface areas, such as acne or psoriasis, with this peel. In general, however, salicylic acid peels are safe when used in more modestly sized areas (less than 20% TBSA) and have minimal complications. They can be used in darker skin types (IV-VI) successfully. Salicylism has also been reported when large areas are peeled with Jessner's solution. The other listed peels do not exhibit this type of toxic reaction.
2016 A 23-year-old man is evaluated 1 day after undergoing a chemical peel to the face, entire back, arms, forearms, and hands, in a nonmedical setting. The patient reports nausea, disorientation, and ringing of the ears. Which of the following chemical peels was most likely used on this patient? A ) Glycolic acid B ) Resorcinol C ) Salicylic acid D ) Solid carbon dioxide slush E ) Trichloroacetic acid
The correct response is Option B. The depressor septi nasi muscle is a small, paired muscle located on both sides of the nasal septum, originating from the medial crural footplates. Its action pulls the nasal tip downward and shortens the upper lip. The overactive muscle can cause the "smiling deformity" in the patient described. It is important to evaluate the smile of the rhinoplasty patient to determine whether the depressor septi nasi muscle needs to be addressed during the procedure in order to optimize results. The depressor septi nasi muscle can be released from the medial crura through the trans-nasal approach. If the patient has tethering of the frenulum, a depressor septi nasi muscle dissection and transposition can be performed through the trans-oral approach. The depressor anguli oris originates from the mandible and inserts into the angle of the mouth. It depresses the corner of the mouth and is associated with frowning, and does not affect the nasal tip or upper lip. The nasalis consists of two parts: transverse and alar. The transverse part arises from the maxilla, above and lateral to the incisive fossa; its fibers proceed upward and medial, expanding into a thin aponeurosis which is continuous on the bridge of the nose with that of the muscle of the opposite side, and with the aponeurosis of the procerus. The alar part is attached by one end to the greater alar cartilage, and by the other to the integument at the point of the nose. The transverse part compresses the nostrils, drawing them toward the septum. The alar part dilates, or flares, the nostrils. While the nasalis is involved in nasal movement, it does not cause the smile deformity described. The levator anguli oris arises from the canine fossa, located under the infraorbital foramen. The muscle's fibers insert at the mouth's angle, and it intermingles with the zygomaticus, triangularis, and orbicularis oris muscles. Although the muscle is involved in and helps form the smile, it elevates the angle of the mouth at the corner and is not involved in nasal tip drooping and shortened upper lip. The levator labii superioris is a broad, flat, quadrangle muscle, and may be considered as three parts extending between the lateral side of the nose and the zygoma in the infraorbital area: angular (medial), infraorbital (intermediate), and zygomatic (lateral) head. The medial part of the angular head inserts into the greater part of the lower lateral cartilages and nasal skin, and helps dilate the nostrils. The lateral part of the angular head, infraorbital, and zygomatic head all insert by merging with fibers of the orbicularis oris, and serve to elevate and evert the upper lip, the primary function of the muscle. Although it may contribute to shortening the upper lip upon activation, it does not cause nasal tip droop.
2016 A 23-year-old woman comes to the office for consultation regarding rhinoplasty because she is dissatisfied with her smile. Physical examination shows a drooping nasal tip, shortened upper lip, and transverse upper lip crease when the patient smiles. Which of the following muscles is the most likely cause of these findings? A ) Depressor anguli oris B ) Depressor septi nasi C ) Levator anguli oris D ) Levator labii superioris E ) Nasalis
The correct response is Option D. Adult gynecomastia occurs because of drugs, unresolved pubertal gynecomastia, and unknown causes (idiopathic gynecomastia). Tamoxifen and raloxifene, both selective estrogen receptor modulators (SERMs), can be used for the treatment of male gynecomastia. Indeed, the use of SERMs is recommended to prevent gynecomastia as a result of antiandrogen monotherapy for treatment of prostate cancer. They are most effective, however, when used to treat gynecomastia that has been present for less than 1 year. Anastrozole, an aromatase inhibitor, is recommended for the treatment of breast cancer and as a preventative measure in high-risk women. Surgical resection is the appropriate treatment for painful gynecomastia that has been present for greater than 1 year. Both open resection and liposuction are appropriate techniques. Radiation therapy is indicated to prevent gynecomastia as a result of prostate cancer treatment. Spironolactone is a cause of—not a treatment for—gynecomastia.
2016 A 24-year-old man is evaluated because of a 15-month history of painful idiopathic gynecomastia. Which of the following is the most appropriate treatment? A ) Anastrozole B ) Radiation therapy C ) Spironolactone D ) Surgical resection E ) Tamoxifen
The correct response is Option C. A double-bubble breast deformity following breast augmentation mammaplasty is represented by the development of two parallel, curvilinear transverse lines in the lower pole of the breast. The native inframammary fold is disrupted and represented by the superior transverse line. The lower transverse line represents the lower limit of implant pocket dissection or the final position of implant descent. Predisposing anatomic factors for the development of a double-bubble deformity include tuberous breasts, constricted inframammary folds, or a short inframammary fold-to-nipple distance. Other factors that can increase the risk for the development of a double-bubble deformity include glandular ptosis, postpartum involution of the breasts, excessive implant size, and overdissection of the implant pocket. Correction of the double-bubble deformity may require conversion of the implant to a subglandular position, capsulorrhaphies, use of form-stable implants, or dermal rafts.
2016 A 25-year-old woman comes to the office because she is dissatisfied after undergoing breast augmentation mammaplasty for correction of tuberous breast deformities. Physical examination shows two parallel creases running transversely across the lower pole of each breast with inferior displacement of the implant. Which of the following best describes the position of the original inframammary fold in this patient? A ) Above the superior and inferior transverse creases B ) At the inferior transverse crease C ) At the superior transverse crease D ) Below the superior and inferior transverse creases
The correct response is Option E. Correction of aesthetic and functional deformities of the nasal tip requires an understanding of the underlying anatomy and the use of cartilage grafting. A pinched nasal tip deformity results from decreased interdomal distance or narrow domal arches of the lower lateral cartilages. Subdomal grafts can be used to correct a pinched nasal tip deformity as well as asymmetry of the domes. The subdomal graft is bar shaped, spanning beneath both domes, controlling the horizontal and vertical orientation of the domes. The columellar strut graft corrects an underprojecting tip by increasing tip projection. The lateral crural graft is utilized to correct alar contour deformities secondary to deformed lateral crus of the lower lateral cartilages. Septal extension grafts control projection, shape, and rotation of the nasal tip. The spreader graft will expand the internal nasal valve and middle one-third of the nose but not correct a pinched tip.
2016 A 27-year-old woman comes to the office for evaluation of her nasal tip. She asks for rhinoplasty for improvement of her aesthetic appearance. Physical examination shows asymmetric projection of the nasal tip with a pinched nasal deformity. Which of the following cartilage grafts would best correct this deformity? A ) Columellar strut B ) Lateral crural strut C ) Septal extension D ) Spreader E ) Subdomal
The correct response is Option E. Capsular contracture occurs when there is fibrosis of the peri-implant capsule. The severity is typically described by the Baker Grade classification. Grade 1: the breast is soft and appears normal in size and shape Grade 2: the breast is a little firm and appears normal Grade 3: the breast is firm and appears abnormal Grade 4: the breast is firm, appears abnormal, and is painful Studies have shown a decreased relative risk for Baker grade 3-4 capsular contracture in primary breast augmentation associated with inframammary fold incision, textured mplants, and subpectoral placement. The relative risk for capsular contracture was increased with periareolar or axillary incision, smooth implants, and subglandular placement. There is no evidence that wearing a support bra or implant massage will decrease the risk for capsular contracture. While hematoma is linked to capsular contracture, the presence of a drain does not prevent hematoma.
2016 A 28-year-old woman is evaluated for micromastia. During consultation, she reports that her best friend underwent breast augmentation that was complicated by painful capsular contracture. Which of the following measures is most likely to prevent this complication in this patient? A ) Initiation of implant massage on postoperative day 5 B ) Placement of a closed suction drain for prevention of postoperative hematoma C ) Use of a surgical support bra postoperatively for 2 weeks D ) Use of a subglandular, smooth, round implant via periareolar incision E ) Use of a subpectoral, textured implant via inframammary incision
The correct response is Option A. There are incidents of surgical procedures of the breast associated with galactorrhea leading to skin breakdown, nipple necrosis, and cellulitis. A dopamine agonist such as bromocriptine will cause decreased lactation in cases of galactorrhea/galactocele, thereby improving wound healing. Antibiotics such as sulfamethoxazole and trimethoprim (Bactrim) are generally not required, because the exudate is sterile. There is no need for debridement of the wound edges. Negative pressure wound therapy may increase lactation and galactorrhea, further impairing wound healing. Metoclopramide is a dopamine antagonist used for nausea and vomiting.
2016 A 28-year-old woman, gravida 2, para 2, undergoes augmentation mammaplasty 1 year post partum. On postoperative day 3, the patient comes to the office because of impaired wound healing at the incision site. Physical examination shows white viscous discharge leaking from the edge of the wound consistent with galactorrhea. Which of the following is the most appropriate management? A ) Administration of bromocriptine B ) Administration of metoclopramide C ) Administration of trimethoprim-sulfamethoxazole D ) Application of negative pressure wound therapy E ) Debridement of the wound edges with wet-to-dry dressings
The correct response is Option E. After bariatric surgery, patients can continue to lose weight as a result of the surgical procedure for approximately 2 years. Thus, most recommendations call for waiting until patients are 12 to 18 months out from their bariatric surgery and at a stable weight for 3 to 6 months. Ideally, patients should be within 10 to 15% of their goal weight. In this case, the patient is still within the time frame of active weight loss, and notes that she is actively losing weight. Thus, the appropriate answer is to wait until weight loss has stabilized. Because this patient is actively losing weight and there are no clinical findings of any issues such as malabsorption, there is no indication currently to evaluate her with an upper GI series or revise her bypass. In addition, as noted above, the risks for surgery are increased at this patient's BMI. Thus, elective liposuction or panniculectomy is not appropriate at this time. Furthermore, because the patient is actively losing weight, the risk for revision surgery to address additional skin laxity that may develop with further weight loss makes undertaking these procedures not appropriate at this point in time.
2016 A 34-year-old woman is evaluated for body contouring after Roux-en-y gastric bypass surgery 6 months ago. There is no evidence of malabsorption. BMI is 36.3 kg/m2. She had a 75-lb (34-kg) weight loss and is actively losing weight. The patient reports low back pain. Which of the following is the most appropriate next step in management? A ) Liposuction B ) Panniculectomy C ) Revision of the gastric bypass surgery D ) Upper GI series E ) Observation
The correct response is Option B. Nipple-sparing mastectomies are becoming more prevalent with the rise of prophylactic mastectomies and increased comfort with performing it in presence of in situ and invasive carcinoma. While universal standards for patient selection do not exist, there is consensus that general recommendations include no inflammatory breast cancers, no pathologic nipple discharge, no Paget's disease, and for many, a 2-cm distance from the tumor to the nipple. Nipple-sparing mastectomy incisions have been described as radial lateral, lateral inframammary fold, and peri-areolar. Because the entire skin envelope is preserved, direct-to-implant reconstruction is possible with inferolateral support from acellular dermal matrices or other scaffolds. Areolar involvement in mastectomy incisions is associated with increased rates of nipple necrosis. The weight of mastectomy specimen, volume of implants, use of acellular dermal matrices or laterality of surgery have not been shown to be associated with nipple necrosis.
2016 A 35-year-old BRCA-positive woman undergoes bilateral prophylactic nipplesparing mastectomy via lateral radial incisions with periareolar extensions. She undergoes direct-to-implant reconstruction with an acellular dermal matrix inferior lateral sling. Mastectomy specimens weigh 435 g each, and placement of 450 mL smooth, round, high-profile implants is performed. Postoperatively, 25% partial nipple areolar complex necrosis in both breasts is noted. Which of the following most likely contributed to the necrosis? A ) Bilateral surgery B ) Mastectomy incision C ) Use of acellular dermal matrix D ) Volume of the implants E ) Weight of the mastectomy specimen
The correct response is Option E. Progressive tension sutures are placed from Scarpa's fascia to the abdominal wall fascia. This helps close the dead space, minimize flap movement, and minimize seroma rate. When placing these sutures with progressive tension, final tension on the abdominal suture line can be lessened. In so doing, healing complications can be reduced. The more common method for reducing tension on flap closure is to close the native umbilical skin opening in a vertical direction. This technique leaves a vertical incision in the midline of the abdominal flap. The need for revision of this scar is not infrequent. Further, most patients want to avoid this scar. Creating a 3-cm transverse incision for the umbilicus would decrease the tension on the flap; however, the appearance of the umbilicus would be aesthetically unacceptable. Relaxing incision of the external oblique fascia is used for closure of ventral herniorrhaphy and would not lessen skin flap tension. Scoring Scarpa's fascia to the dermis would injure the subdermal vascular plexus, on which the vascularity of the abdominal flap depends. Upper abdominal liposuction can be performed at the same time as abdominoplasty, provided care is taken to maximize preservation of the lateral rectus perforators. It would not be a solution to minimize skin tension.
2016 A 35-year-old woman comes to the office for consultation regarding a tummy tuck. She wants the scar as low as possible, but she does not want a lower vertical midline scar. Physical examination shows mild upper and lower abdominal skin excess and rectus abdominis diastasis. BMI is 27 kg/m2. Abdominoplasty and repair of diastasis are planned. Intraoperatively, there is marked tension on the lower central abdominal flap closure. Which of the following is the most appropriate maneuver to decrease the tension on the repair? A ) Creation of a 3-cm transverse umbilical opening B ) Liposuction of the upper abdomen C ) Relaxing incision of the external oblique fascia D ) Scoring of Scarpa fascia to the dermis E ) Use of progressive tension sutures
The correct response is Option D. Skin laxity is the single greatest determinant of whether liposuction is an appropriate modality in an algorithmic approach to upper arm lipodystrophy. The determination of excessive fat can be made by the pinch test, and patients with greater than 1.5 cm of fat on a pinch test may be candidates. The classification of lipodystrophy, described by Rohrich et al., includes skin excess, fat excess, and the location of skin excess. Where there is skin excess, the skin must be excised for a favorable result. Liposuction alone can exacerbate the appearance and presence of excess skin. Similarly, skin laxity is a predictor of liposuction success. With marked laxity, the skin is unlikely to have enough elastic properties to retract. Although there are some papers that show increased retraction of skin with laser liposuction, this has not been shown to be a consistent result in large-scale studies.
2016 A 35-year-old woman comes to the office for lipodystrophy of the upper arms. Physical examination shows negligible skin laxity; pinch test shows a thickness of 3 cm of the entire upper arm and the chest wall. Which of the following is the most appropriate surgical intervention? A ) Extended brachioplasty B ) Limited medial brachioplasty C ) Mini brachioplasty D ) Suction-assisted lipectomy E ) Traditional brachioplasty
The correct response is Option C. This patient presents with a parenthesis deformity with vertically oriented lower lateral cartilages. To correct this deformity, the lower lateral cartilages must be rotated inferiorly. A lateral crural strut graft is a strip of cartilage 3 to 4 mm in width sutured to the deep surface of the lateral crura and then either buried or sutured to the soft tissue of the pyriform aperture. In this way, the native lower lateral cartilage can be rotated inferiorly and held in place. A caudal septal extension graft is sutured to the caudal septum and is used to control nasal tip projection as well as lengthen an overly shortened nose. A crural turnover graft is created by folding the cephalic portion of the upper lateral cartilage inferiorly onto itself. It thereby strengthens itself and is used to support weakened or collapsed lower lateral cartilages. Spreader grafts are placed at the dorsal edge of the septum to correct internal nasal valve collapse and support the upper lateral cartilages. A spring graft widens the middle vault by spanning between both upper lateral cartilages.
2016 A 35-year-old woman is dissatisfied with the appearance of her nose. Physical examination shows parenthesis tip deformity with vertically oriented lower lateral cartilages. Which of the following grafts is most appropriate for correction of this deformity? A ) Caudal septal extension B ) Crural turnover C ) Lateral crural strut D ) Spreader E ) Spring
The correct response is Option C. Among the most common complications following body contouring for post-massive weight loss-induced skin laxity is seroma, occurring in up to 35 to 50% of patients. To minimize the risk for seroma, preoperative nutritional repletion, especially for protein, and intraoperative use of closed suction drains, aggressive minimization of dead space, limited degree of skin flap undermining, and use of well-fitted elastic compression garments are among the techniques that are commonly recommended. None of these, even in combination, can completely guarantee the elimination of this complication. After several ineffective aspirations, compression garments alone are not likely to eliminate a seroma for this patient. The most appropriate next management measure for this patient's seroma is ultrasound-directed percutaneous closed suction drain placement. Operative incision and drainage is not indicated unless the closed drain fails, following failed sclerosant therapy, or if the seroma is shown to be infected. A muscle flap is not indicated in this setting.
2016 A 37-year-old woman reports nontender swelling of the lumbar area after undergoing lower body lift following massive weight loss. Examination shows tense swelling and a positive fluid wave test. Percutaneous needle aspiration is performed on a weekly basis, and fluid is still present after three aspirations of 150 mL each of a clear, yellowish serum. Which of the following is the most appropriate next step in management? A ) Compression B ) Operative incision and drainage C ) Placement of a closed suction drain tube D ) Use of an ipsilateral gluteus maximus muscle advancement flap
The correct response is Option E. Transconjunctival blepharoplasty preserves the middle lamella, which includes the orbicularis oculi muscle. Preserving this layer significantly decreases the incidence of ectropion and lower eyelid malposition. Disadvantages of this technique include more difficulty with visualization and access. Many authors believe that a cutaneous skin muscle approach is more effective in blending the lid-cheek junction and transposing fat. Some studies have shown minor reductions in hematoma and infection rates with the transconjunctival approach; however, the differences are small and not the most significant advantage. The potential for corneal injury is greater with the transconjunctival approach. Most surgeons use corneal shields to prevent this complication. The capsule palpebral fascia is routinely cut to access the fat compartments during a transconjunctival blepharoplasty.
2016 A 45-year-old woman comes to the office to discuss aesthetic improvement of the lower eyelids. Physical examination shows lower eyelid pseudoherniation of fat and fine skin wrinkles. A postseptal transconjunctival approach with skin pinch excision is considered. The most significant advantage of this approach compared with a transcutaneous skin-muscle flap is a lower risk of which of the following complications? A ) Capsulopalpebral fascia injury B ) Corneal injury C ) Hematoma D ) Infection E ) Lid malposition
The correct response is Option A. The borders of the breast and, therefore, the limits of the mastectomy are the sternum, clavicle, inframammary fold, and anterior border of the latissimus dorsi muscle. The other options are not designated as borders of the breast tissue.
2016 A 45-year-old woman undergoes mastectomy. Which of the following anatomical landmarks denotes a limit of the breast and, therefore, a limit to the extent of the mastectomy? A ) Anterior border of latissimus dorsi muscle B ) Inferior origin of pectoralis major muscle C ) Lateral pectoralis minor muscle D ) Superficial fascia of the serratus anterior muscle E ) Supraclavicular lymph node basin
The correct response is Option A. Atypical ductal hyperplasia is a risk factor for breast cancer in both the ipsilateral and contralateral breast, although the risk for the ipsilateral breast is higher. There is an approximately three-fold to five-fold increase in the risk for breast cancer in patients who have had biopsy-proven atypia. In an excisional biopsy, no additional surgery is needed for a finding of atypia; this is in distinction to atypia found on core biopsy, in which excision is recommended. In neither case is mastectomy indicated. Radiation therapy is indicated for positive surgical margins in breast cancer excision, tumor size greater than 5 cm, more than four positive axillary nodes, and T4 disease. The finding in this scenario should prompt the practitioner to perform a full risk assessment. Although the Gail model has been criticized for underestimating the risk for cancer in the context of atypical hyperplasia, it is still the most appropriate choice of the options listed. If the risk for breast cancer is sufficiently high after all factors are considered, hormonal therapy as a preventive measure may be indicated.
2016 A 47-year-old woman undergoes bilateral reduction mammaplasty surgery. Pathologic analysis of the resected breast tissue shows atypical ductal hyperplasia in the left breast. Which of the following is the most appropriate next step in management? A ) Gail model risk assessment B ) Hormonal therapy with aromatase inhibitors C ) Hormonal therapy with selective estrogen receptor modulators D ) Left-sided mastectomy E ) Postoperative radiation therapy
The correct response is Option B. Goals of improvement would be upper pole fullness and a coned, rounded breast, with raising the nipple. Because the superior pole thickness is less than 2 cm, a subglandular implant is not recommended. A dual-plane implant would not address the ptosis and would likely leave persistent ptosis. Vertical mastopexy alone would require some modification to address the excess vertical skin with some element of horizontal inferior excision. This would not address the lack of upper pole volume in the long term. The striae indicate poor tissue strength. Staged implant placement would have the fewest risks.
2016 A 47-year-old woman, gravida 3, para 3, is evaluated for improvement of breast appearance. She breast-fed all three of her children for 1 year each. Examination shows the distance from nipple to sternal notch is 27 cm bilaterally; decreased superior pole volume, and striae are also noted. There is Grade 3 ptosis bilaterally. The pinch of the superior pole soft tissue is 1 cm. Which of the following procedures is most likely to improve superior pole volume and breast shape in this patient? A ) Dual-plane implant augmentation B ) Mastopexy with dual-plane implant augmentation C ) Mastopexy with subglandular implant augmentation D ) Subglandular implant augmentation E ) Vertical mastopexy
The correct response is Option E. HIV-positive patients with nondetectable viral loads and normal T-cell counts are at no higher risk for infection with transdermal procedures than HIV-negative patients. While finding adequate subdermal fat for fat harvesting can be challenging in some HIV patients because of "wasting," or loss of fat, in patients with a normal or high BMI, fat for transfer is generally not an issue. No studies have shown higher rates of transferred fat reabsorption in HIV, while studies have shown long-term (12-month) retention using CT scans as an objective measure. Wasting is a hallmark of late-stage AIDS. Antiretroviral therapy is a mainstay of treatment for HIV-infected patients, generally improves wasting, and has dramatically improved the health and longevity of HIV-positive patients. Nonetheless, antiretrovirals are a major contributing factor to facial lipoatrophy even in stable HIV-positive patients with low or nondetectable viral loads and normal and near normal CD4 cell levels. While facial fat grafting in any patient is somewhat unpredictable, for healthy HIV-positive patients with adequate body fat, it is a reasonable approach to facial rejuvenation in patients with lipoatrophy. Patient satisfaction and bruising should show the same variability as the HIV negative patient.
2016 A 48-year-old man with HIV infection comes to the office requesting facial rejuvenation because of the accelerated changes he has seen since his HIVpositive status was identified. He is receiving antiretroviral therapy, has a nondetectable viral load, and normal T-cell count. Physical examination shows significant facial lipoatrophy despite the patient appearing somewhat overweight with a BMI of 27 kg/m2. The patient desires fat transfer to correct this deformity. Which of the following best describes this patient's risk evaluation when compared with HIV-negative patients? A ) Decreased satisfaction rate B ) Higher risk of infection C ) Increased bruising D ) Increased fat graft resorption E ) Similar risk profile
The correct response is Option D. A sternal notch to nipple distance of 16 cm represents a high-riding nipple. Revisional surgery for correction of a high-riding nipple is complex, and it is difficult to achieve a favorable result because of the surgeon's and patient's desire to avoid a scar extending superior to the nipple areola. Further, the paucity of excess skin between the nipple and clavicle limits the reconstructive options. Suggested strategies include direct reposition of the nipple-areola complex, expansion of the skin between the nipple and clavicle, and repositioning of the breast parenchyma and inframammary crease. Breast size asymmetry can be improved with either liposuction or revision mastopexy/reduction. The operation is usually performed using the previous incisions. A dog ear of the inferior vertical scar is easily revised with a small transverse scar within the inframammary crease. The majority of these early postoperative deformities will resolve without surgery. Recurrence of ptosis or an elongation of the nipple to inframammary crease distance occurs with all mastopexy operations. When performing secondary mastopexy, this can be improved with shortening the vertical scar with wedge resection at the inframammary crease. Knowledge of the location of the previous nipple areola pedicle is helpful in minimizing vascular complications. Widened circumareolar scars can be revised with excellent results. Utilizing a permanent suture around the areola helps control size of the areola and tension on the suture line.
2016 A 48-year-old woman comes to the office because she is very unhappy with the appearance of her breasts following a bilateral mastopexy performed 1 year ago. Height is 5 ft 7 in (170 cm). BMI is 26 kg/m2. Which of the following findings on physical examination would be most difficult to correct? A ) Asymmetrical breast size B ) Dog ear of the inferior vertical scar C ) Nipple to inframammary crease distance of 16 cm D ) Nipple to sternal notch distance of 16 cm E ) Widened circumareolar scar
The correct response is Option C. In this patient, the highest Caprini risk factor is the history of a VTE, which carries a score of 3. The length of surgery is greater than 45 minutes so it would be considered major and would carry a score of 2, as would her history of malignancy. Age, obesity, and contraceptive use all carry a score of 1. This gives the patient a Caprini score of 10.
2016 A 51-year-old woman comes to the office for consultation for abdominal and lower extremity liposuction. The procedure is expected to last approximately 2.5 hours. BMI is 30 kg/m2. The patient takes an oral contraceptive. The patient reports having had a small venous thromboembolism (VTE) during lumpectomy for breast cancer that took place in her early 40s. Which of the following factors increases the risk of VTE and Caprini risk assessment score most significantly? A ) Age B ) History of malignancy C ) History of VTE D ) Length of surgery E ) Use of an oral contraceptive
The correct response is Option A. Minimal lagophthalmos in the postoperative period following blepharoplasty is not uncommon and generally self-correcting. Minimal lagophthalmos may persist but is often asymptomatic owing to compensatory blinking and increased tear production, both of which are the result of the mild exposure and resulting corneal stimulation. Laser-assisted in situ keratomileusis (LASIK) procedure involves creation of a corneal flap that interrupts the long ciliary nerves of the ophthalmic division of the trigeminal nerve. The interruption of these nerves results in decreased sensation to the cornea and a decreased corneal reflex arc. Patients with compensated exposure from blepharoplasty may lose the compensatory blink in reaction to corneal irritation following LASIK. This may cause dry-eye symptoms. With time, the corneal reflex arc improves, and the transient neurotrophic keratopathy and dry-eye symptoms improve. The other options are unlikely to cause her symptoms of dry eyes.
2016 A 53-year-old woman is evaluated 6 weeks after undergoing blepharoplasty. Physical examination shows 1 mm of lagophthalmos with no dry-eye symptoms. The patient returns 8 months later for evaluation of dry eye, which began one month after undergoing laser-assisted in situ keratomileusis (LASIK) surgery. Which of the following is the most likely reason for this patient's dry-eye symptoms? A ) Blunted blink reflex secondary to decreased corneal reflex arc B ) Chronic use of vasoconstrictive eyedrops C ) Transient decrease in functioning of the orbicular muscle of the eye secondary to stretching from lid traction during surgery D ) Transient decrease in tear production caused by lacrimal gland pressure injury E ) Transient hypersensitivity of the cornea
The correct response is Option C. Studies have shown that contralateral symmetry procedures performed synchronously with unilateral autologous tissue reconstruction after mastectomy (including reconstructions with a free perforator flap) are acceptable. Situations requiring reduction rather than augmentation or mastopexy seem to be the most suited to this timing. A patient with a lower BMI and gigantomastia is unlikely to achieve symmetry without a contralateral procedure. Autologous tissue reconstruction likely would not need a contralateral implant for upper pole symmetry. An attempt at improvement in symmetry using contralateral surgery would be acceptable should the patient so choose, but recommending a prophylactic mastectomy on the contralateral side solely for symmetry and not for risk reduction may be overly aggressive when other methods such as reduction mammaplasty exist with a likelihood of acceptable postoperative symmetry.
2016 A 53-year-old woman with a BMI of 27 kg/m2, gigantomastia, and grade III ptosis is considering unilateral mastectomy and autologous reconstruction with abdominal tissue. Which of the following is the most appropriate advice regarding the contralateral breast? A ) A contralateral prophylactic mastectomy should be performed to maximize symmetry B ) Contralateral reduction should be accompanied by placement of an implant on that side to address upper pole symmetry with the autologous reconstruction C ) Simultaneous contralateral reduction may be performed with an acceptable risk profile D ) Symmetry is unlikely to be achieved, so a contralateral matching procedure would not be recommended E ) Symmetry with autologous tissue is likely to be achieved without the contralateral matching procedure
The correct response is Option A. Estrogen is the primary hormone in promoting the development of the breast epithelium and ductal tissue. Progesterone acts in combination with estrogen to regulate breast development. With the onset of menopause, there is a decrease in the secretion of estrogen and progesterone. As a result of the decrease in the circulating levels of these hormones, the breast undergoes regression and atrophy of the glandular elements. Oxytocin and prolactin are hormones involved in the physiology of lactation. Growth hormone and testosterone may have an effect on breast tissue, but they are not primary factors in the physiology of the female breast.
2016 A 55-year-old postmenopausal woman desires improvement in the appearance of her breasts. Change in which of the following levels of hormones is most likely responsible for postmenopausal involution of breast tissue? A ) Estrogen B ) Growth hormone C ) Oxytocin D ) Prolactin E ) Testosterone
The correct response is Option E. When superficial musculoaponeurotic system (SMAS) tightening procedures are performed, the tension of the facelift is secured at the SMAS level rather than the skin. With a skin-only facelift, the lift must depend solely on the skin sutures for support. The more tension on the skin, the more likely a spread or hypertrophic scar will occur. SMAS procedures do not decrease the longevity of results. Many surgeons believe that a SMAS facelift will produce better results and longer lasting results. Despite these opinions, there are no conclusive evidence-based studies to prove an increase in longevity of results. Hematoma formation is related to hypertension and extent of dissection, not whether a SMASectomy was performed. SMAS procedures put the facial nerve at greater danger than skin-only facelifts. The incidence of facial nerve injury is low with both techniques. Infection rates are low with both procedures.
2016 A 55-year-old woman comes to the office to discuss a facelift. A rhytidectomy with SMASectomy (superficial musculoaponeurotic system) is planned. Compared with a skin-only facelift, a SMAS tightening procedure is associated with a decrease in which of the following? A ) Facial nerve injury B ) Hematoma formation C ) Infection rate D ) Longevity of result E ) Tension on the skin closure
he correct response is Option A. Breast reconstruction requires reconstruction of both the soft-tissue envelope and volume. In the setting of radiated chest wall tissues, it would be difficult to recreate a breast envelope of sufficient size to match the contralateral breast with tissue expansion, even with acellular dermal matrix or total submuscular placement of a tissue expander. The history of radiation would likely cause an element of capsular contracture and decrease symmetry to the contralateral breast with grade 2 ptosis. Both the latissimus dorsi flap and the thoracodorsal artery perforator flap would recruit non-radiated skin to create an adequate envelope, but would not provide adequate volume alone. The deep inferior epigastric perforator (DIEP) flap would provide both envelope and volume.
2016 A 57-year-old woman with a history of modified left radical mastectomy followed by adjuvant chemotherapy and radiation therapy is evaluated six months after her last radiation treatment for breast reconstruction. Physical examination shows slight skin redundancy of the chest wall and grade 2 ptosis of the contralateral breast. She wears a size 36D brassiere and does not want to undergo symmetry procedures on the contralateral breast. Which of the following reconstructive options is most likely to achieve breast symmetry in this patient? A ) Deep inferior epigastric perforator flap B ) Latissimus dorsi flap C ) Thoracodorsal artery perforator flap D ) Tissue expander with acellular dermal matrix and staged implant placement E ) Total submuscular tissue expander and staged implant placement
The correct response is Option A. Pedicled transverse rectus abdominis muscle (TRAM) flaps are based on the superior epigastric system, which is often less robust than the deep inferior epigastric system. Therefore, surgical delay by ligation of the deep inferior epigastric system may facilitate overall viability of the transferred tissue. Ligation of the superior epigastric system would make a pedicled TRAM flap unlikely to survive. The internal mammary ligation may also interrupt blood supply to the superior epigastric system, and even if the tissue is fed through collaterals, it would not strengthen the flap. Division of the superficial inferior epigastric system might also help, but it is not as critical as ligation of the deep inferior epigastric system. The hypogastric system does not have a direct impact on the pedicled TRAM tissues.
2016 A 62-year-old woman undergoes breast reconstruction using autologous tissue from the abdomen. Intraoperatively, use of a perforator flap is found to be impossible because of multiple small nondominant perforators. Conversion to a delayed pedicled transverse rectus abdominis muscle flap is planned. Ligating which of the following vessels in this stage will best facilitate future viability of the tissue transferred in the next stage? A ) Deep inferior epigastric B ) Hypogastric C ) Internal mammary D ) Superficial inferior epigastric E ) Superior epigastric
The correct response is Option E. A spreader graft is placed between the septum and the upper lateral cartilages. Poiseuille law states that resistance = (viscosity × length)/radius4. About half of nasal airway resistance occurs at the internal nasal valve. The internal nasal valve, formed at the junction of the septum (medially), the nasal floor (inferiorly), the inferior turbinate (laterally), and the caudal border of the upper lateral cartilages (superiorly), accounts for a significant amount of airway resistance. Maneuvers that increase the radius at the internal nasal valve will decrease resistance exponentially.
2016 A healthy 26-year-old woman undergoes rhinoplasty using a spreader graft. Which of the following is the most likely cause of decreased airway resistance after placement of the spreader graft? A ) Decreased angle at the external nasal valve B ) Decreased area of airway C ) Decreased radius at the internal nasal valve D ) Increased angle at the external nasal valve E ) Increased radius at the internal nasal valve
The correct response is Option E. Postsurgical changes in the breast after reduction mammaplasty encompass a variety of physical and radiographic manifestations. On presentation of a breast mass after reduction mammaplasty, a diagnostic protocol is used to determine whether operative intervention is appropriate, to avert unnecessary biopsy and to avoid overlooking breast malignancy. Fat necrosis, oil cysts, fibrosis, organizing hematoma, calcifications and, rarely, concurrent malignancy should be considered in this patient. In the absence of acute phenomena characteristic of a wound infection, work-up should consist of mammography and ultrasonography of the affected breast. Combining the radiographic findings with echographic appearance of the mass will help to differentiate fat necrosis and other benign conditions from the more ominous malignant etiology. The mammographic appearance of fat necrosis ranges from completely undetectable to a spiculated density and clustered microcalcifications. Many authors believe that the calcifications of fat necrosis can be distinguished from those seen with breast malignancies. Ultrasonographic findings include a solitary cyst, heterogenous echogenicity, and microcalcifications. Any remaining doubt as to the biologic nature of the mass should then be pursued with needle or open biopsy of the mass. Injection of any agent into the mass before it is definitively diagnosed is contraindicated, as is surgical removal or observation without obtaining a confident exclusion of malignancy. Referral to an oncologist would be premature in this instance and would provoke an unnecessary level of patient anxiety.
2016 A healthy 27-year-old woman is evaluated 16 weeks after bilateral reduction mammaplasty with an inferior pedicle technique. Histologic examination of the resected tissue shows no malignancy. She reports a tender mass in the right breast that she noted 8 weeks after operation. She is now apprehensive because her mother had breast cancer at age 34. Physical examination of the affected breast shows a palpable, slightly tender, discrete, firm 2-cm subcutaneous mass beneath the upper areolar border. Examination shows no skin dimpling, nipple retraction, erythema, or edema. Which of the following is the most appropriate next step in management? A ) Conduct needle aspiration of the lesion B ) Inject triamcinolone acetonide suspension 20 mg into the mass C ) Proceed to surgery for removal of the mass D ) Refer the patient for an oncology consultation E ) Schedule ultrasonography and mammography of the affected breast
The correct response is Option E. The most common cause of postoperative facial nerve weakness following rhytidectomy is residual effect from local anesthesia. This effect can take several hours to wear off and the most reasonable course of management is to observe and reexamine the patient to ensure return of function. Corticosteroid injection is not indicated in this situation. Aspirating under the flap would not prove beneficial and is typically reserved for a small seroma that can develop within days following a rhytidectomy, not immediately following surgery. If there is cause for concern that a hematoma is present, then the patient should be returned to the operating room for evacuation of this and hemostasis. A hematoma would not, however, cause weakness of the facial nerve immediately postoperatively. Although nerve entrapment from sutures is a possible explanation for facial nerve weakness, it is much less likely a cause than a residual effect from the tumescent anesthesia.
2016 A healthy 64-year-old woman undergoes rhytidectomy with superficial musculoaponeurotic system (SMAS) plication and platysmaplasty. Preoperatively, 150 mL of tumescent solution is infiltrated into the face and neck. In the recovery room, the patient has buccal branch weakness of the right side. Overall facial swelling is noted, but the right side is slightly more swollen than the left side; the swelling and bruising are symmetric. Which of the following is the most appropriate next step in management of the right side of the face? A ) Injection of corticosteroid B ) Percutaneous aspiration C ) Reexploration D ) Release of potential nerve entrapment from sutures E ) Observation only
The correct response is Option D. The patient described has moderate acquired blepharoptosis characterized by a high eyelid crease and 3 mm of ptosis. Her levator palpebrae muscle excursion is excellent (10 mm). The most likely cause is disinsertion of the levator aponeurosis from the tarsal plate due to chronic stretching of the levator muscle or involutional changes associated with aging. Surgical treatment most commonly involves advancing and reattaching the levator muscle to the tarsal plate. An upper eyelid blepharoplasty alone or with a browlift would not correct the underlying cause of the ptosis. The Fasanella-Servat (tarsoconjunctival mullerectomy) procedure classically excises a portion of mucosa, Müller's muscle, and superior tarsus, thereby shortening the posterior lamella. This can address mild ptosis (1 to 2 mm) in a patient with otherwise excellent levator function. The frontalis sling procedure uses autologous or alloplastic material tunneled from the brow to upper tarsus to accomplish upper eyelid elevation when levator function is poor. The cause of ptosis in these cases is likely congenital.
2016 A healthy 65-year-old woman is evaluated because of drooping of the upper eyelids. Examination shows high eyelid creases and 3 mm of ptosis. Levator excursion measures 10 mm. Which of the following is most likely to correct the blepharoptosis in this patient? A ) Brow lift surgery B ) Fasanella-Servat procedure C ) Frontalis sling D ) Levator advancement E ) Upper eyelid blepharoplasty
The correct response is Option D. While the overall incidence of nerve injury during rhytidectomy is low, consequences, depending on which nerve is involved, can range from minor annoyance to devastating aesthetic and functional sequelae. Identification of the location of nerves tha t are likely to be subject to sharp or blunt injury during rhytidectomy is key to prevention of injury. An intimate knowledge of the anatomy is imperative, particularly for the trunk and branches of cranial nerve VII, the auriculotemporal nerve, and the great auricular nerve (GAN). Cranial nerve VII branch laceration can result in deficits of brow elevation (frontal branch), paralysis of the orbicularis oculi (zygomatic branch), buccinator incompetence (buccal branch), asymmetry of the lip depressors (marginal mandibular branch), or loss of platysma tone (cervical branch). The auriculotemporal nerve innervates the external auditory meatus, upper helix, and temporal scalp. Gustatory sweating (Frey's syndrome) occurs due to aberrant reinnervation of cutaneous sweat glands after disruption of auriculotemporal nerve branches, more likely after parotidectomy. Motor function of the posterior auricular muscle is provided by the temporal branch of cranial nerve VII. Transection of the GAN would result in a sensory disturbance to the lobule of the ear and may elicit dysesthesia, cold intolerance, or focally painful neuroma. It is the most frequently injured nerve during rhytidectomy, with an incidence estimated at up to 2.6%. It may be repaired with epineural suture to help prevent neuroma. A recently described method to avoid injury to the GAN locates it within a triangle constructed using the anterior limb perpendicular to the Frankfort horizontal and the posterior limb angled 30 degrees behind the first limb and passing through the midpoint of the earlobe.
2016 Accidental division of the great auricular nerve during rhytidectomy most commonly results in which of the following outcomes? A ) Gustatory sweating B ) Inability to elevate the brow C ) Loss of sensation to the temporal scalp D ) Numbness of the earlobe E ) Paralysis of the posterior auricular muscle
The correct response is Option C. The tuberous breast deformity results in a protruding, oblong shape that resembles a tuberous root plant (Latin derivation tuber = to swell). The features noted in the tuberous breast deformity include a constricted breast base, decreased breast parenchyma, abnormal elevation of the inframammary fold, a decreased skin envelope, and herniation of the breast parenchyma through the central breast and into the areola. The areola is large and lacks firm underlying structure, thus allowing the breast tissue to protrude through this path of least resistance. The deformity is also often referred to as a tubular breast, constricted breast, doughnut breast, nipple breast, breast with narrow base, dome nipple, and snoopy dog breast. The overall etiologic factors leading to the full expression of the constricted breast deformity are still largely unknown and likely involve a delicate balance of anatomic and endocrinologic forces. A constricting fibrous ring at the level of the areola periphery, representing probably a thickening of the superficial fascia coupled with the normally absent fascial layer in the NAC, has been proposed as a likely cause. The ring is composed of dense fibrous tissue made of large concentrations of collagen and elastic fibers arranged longitudinally. It is usually denser at the lower part of the breast and does not allow the developing breast parenchyma to expand during puberty. It has been suggested that a thickening of the superficial fascia combined with the absence of a superficial fascial layer under the NAC is the underlying natomic/histopathologic cause of the deformity. The cause of the thickened fascia is unknown, although at least one study by Klinger, Caviggioli, et al. demonstrated altered collagen in both disposition and quantity. The same study excluded amyloid deposition as a component of the fibrosis. The areola in the tuberous breast still contains the normal muscular structures that result in areolar changes with stimulation and temperature changes, although the tissue beneath the areola may be thinned.
2016 An 18-year-old woman comes to the office for evaluation of her breasts. Photographs of the patient are shown. Which of the following statements most accurately describes the anatomy of this patient's breasts? A ) The areola is normal size although the breast is small B ) The breast tissue is uniformly distributed throughout the breast pocket C ) The inframammary fold is elevated D ) The skin envelope has greater laxity than in a normal breast E ) The underlying musculature is underdeveloped
The correct response is Option E. The patient described is likely to have gynecomastia, which can occur in up to 30% of men. In the younger, postpubertal patients, testicular cancer may be a cause. Examination of the scrotum and testes to rule out any testicular masses has to be included in the physical examination and documented. Measurement of beta-human chorionic gonadotropin concentration can also be effective. The other examinations listed do not directly contribute to the assessment of gynecomastia.
2016 An otherwise healthy 25-year-old man comes to the office with concerns about recent enlargement of his breasts. BMI is 28 kg/m2 and has not changed during the past year. He does not take any prescription or illicit drugs. He has no nipple discharge. Examination of which of the following is most appropriate? A ) Cervical lymph nodes B ) Cranial nerves C ) Peripheral pulses D ) Prostate E ) Scrotum and testes
The correct response is Option B. Acellular dermal matrix (ADM) use has been increasing over time with tissue expander or implant-based breast reconstruction following mastectomies. Many potential advantages and disadvantages have been studied, and some of the data are contradictory. However, the consensus of the literature indicates that ADMs are associated with decreased capsular contracture rates. There is literature to suggest that seroma rates are increased or remain stable, not decreased, with ADMs. ADMs have not been shown to decrease independent patient risk factors for complications such as tobacco use or to decrease cancer recurrence rates. ADMs also do not appear to improve vascularity of the tissue overlying them when initially placed.
2016 An otherwise healthy 38-year-old woman who is a smoker is considering implantbased breast reconstruction following mastectomy. She has been counseled about likely use of acellular dermal matrix when the intermediate tissue expander is placed and wants to further understand why the matrix will be used in her body. The patient should be advised that use of acellular dermal matrix is associated with a decreased risk for which of the following? A ) Cancer recurrence B ) Capsular contracture C ) Seroma D ) Skin flap necrosis E ) Smoking-related complications
The correct response is Option B. Ptosis following botulinum toxin type A (Botox) treatment to the glabellar area is most commonly associated with inadvertent exposure of the levator palpebrae superioris muscle to Botox. This muscle is the primary upper eyelid elevator and is innervated by the third cranial nerve. The muscle portion arises from the greater wing of the sphenoid and is typically 40 mm long. The tendinous distal portion is 14 to 20 mm long and is termed the levator aponeurosis. The transition from the muscular to the tendinous portion occurs in the region of Whitnall's ligament, a condensation of the superior sheath of the levator muscle. Ptosis from Botox treatments most commonly occurs when the Botox spreads outside the intended target muscle. This is most commonly a technical error on the part of the injector by not staying high enough within the corrugator muscle and above the orbital rim. Ptosis from Botox injections cannot be reversed; however, the condition does completely resolve when the Botox effect wears off after several (3 to 4) months. Interval treatment to help improve, but not definitively treat, the ptosis consists of alpha-adrenergic eyedrops such as Iodipine or phenylephrine ophthalmic preparations, which cause stimulation of Müller's muscle to help improve the condition somewhat, but do not adequately resolve the ptosis. Müller's muscle is an accessory eyelid elevator and lies deep to the levator. It is innervated by the sympathetic nervous system. Contraction of this muscle (such as with pharmacologic stimulation) contributes about 2 mm to lid retraction.
2016 An otherwise healthy 46-year-old woman undergoes botulinum toxin type A (Botox) treatment for severe glabellar lines. Twelve units of Botox is administered into each corrugator muscle. Eight days later, the patient comes to the office because of ptosis of the right eyelid. Which of the following muscles is most likely inadvertently affected in this patient? A ) Frontalis B ) Levator palpebrae superioris C ) Müller muscle D ) Orbicular muscle of the eye E ) Procerus
The correct response is Option E. Resurfacing causes complete ablation of the epidermis and superficial papillary dermis with thermal injury and coagulation through the papillary dermis. Wound healing occurs by re-epithelialization from the dermal appendages (hair follicles and sebaceous glands) and is complete within 7 to 10 days. The thermally damaged dermal layers are repaired by the stimulation of fibroblasts. This coagulated tissue is replaced by new bundles of tight collagen, a process that continues for up to 6 months. Thus, the result tends to improve over time. During the initial period of healing and during the re-epithelialization, there is considerable edema and exudation of proteinaceous material, resulting in redness and crusting. Erythema is most intense during the first month after treatment but may persist for 6 months or more. In addition to intense erythema, other expected adverse effects occurring in virtually all patients during the first postoperative week include marked edema, pain, and pruritus. Allergic reactions, or contact dermatitis, occur most often in open postoperative dressing regimens where the patient has used topical products that contain irritants. Signs and symptoms suggestive of an allergic contact dermatitis include diffuse and intense facial erythema and/or pruritus. Bacterial and fungal infections are often the result of prolonged (greater than 48 hours) wound occlusion in the postoperative period. Although the risk of bacterial infection is increased in the closed technique compared with the open, in this case, the dressing was changed at day 1 and it is therefore unlikely. Additional findings on examination suggestive of infection include lesions with skin invasion, focal areas of increased erythema, discoloration, purulent rather than serous discharge, and ulceration. The patient described did not exhibit any of these findings. The most common bacterial pathogen is Staphylococcus aureus, and the most common fungal pathogen is Candida. Despite adequate antiviral prophylaxis, 2 to 7% of laser-treated patients have been shown to develop herpes simplex virus reactivation. An outbreak on laser-treated skin may have symptoms such as superficial erosions and irregular redness. These findings are not present in this case.
2016 An otherwise healthy 54-year-old woman with Fitzpatrick Type II skin undergoes full-face carbon dioxide laser resurfacing. She received acyclovir for 3 days before the procedure. She is treated with a closed dressing regimen. On postoperative day 2, the patient has onset of facial pain and pruritus. Physical examination shows marked diffuse erythema and edema. Which of the following is the most likely diagnosis? A ) Allergic reaction B ) Bacterial infection C ) Fungal infection D ) Herpes simplex virus E ) Normal healing
The correct response is Option D. Nonsurgical fat freezing treatment (CoolSculpting) is a method of noninvasive fat reduction that occurs via the use of cryolipolysis. It is a noninvasive technique that takes place in an office setting and does not involve the use of surgery. Following CoolSculpting treatment, patients have some aspect of bruising and swelling, which can last for up to several weeks. Results of CoolSculpting treatments typically take 3 to 4 months to develop. Results are additive, however, with multiple treatments. CoolSculpting involves the utilization of different applicators that have been developed to treat various parts of the body effectively. Each actual treatment is for 1 hour; however, most areas of the body may require multiple treatments to be effectively managed. Treatment times for an abdomen can range from 2 to 6 hours alone, whereas outer thighs are treated for at least 2 hours per side and inner thighs at least 1 hour per side. Although not all of these treatments need to be performed in one sitting, the total time for CoolSculpting treatments for most areas of the body are much longer than it would take to surgically perform liposuction. In general, however, CoolSculpting is a less expensive treatment option compared with liposuction as there is no fee necessary for anesthesia or operating-room costs.
2016 Compared with liposuction, which of the following is the greatest advantage of cryolipolysis? A ) No procedural discomfort B ) No risk of bruising C ) No risk of posttreatment swelling D ) No surgical intervention E ) Shorter duration of treatment time
The correct response is Option B. The medial antebrachial nerve is most at risk for injury during routine brachioplasty surgery secondary to its superficial location within the subcutaneous tissue within the area of typical skin and soft-tissue excision. This nerve arises from the medial cord of the brachial plexus 78% of the time and from the lower trunk in 22%. After emerging from the axilla, the medial antebrachial cutaneous nerve travels medial to the brachial artery and lies adjacent to the basilic vein at the distal upper arm. In the distal or mid brachium, this nerve pierces the deep fascia to become very superficial running above the deep fascia at an average of 14 cm proximal to the medial epicondyle. Despite some minor anatomical variability, this nerve has been found to be consistently present in the deep plane of dissection for the standard brachioplasty technique.
2016 During routine brachioplasty, which of the following nerves is/are most likely at risk during typical dissection? A ) Lateral antebrachial cutaneous nerve B ) Medial antebrachial cutaneous nerve C ) Sensory branches of the axillary nerve D ) Sensory branches of the radial nerve
The correct response is Option A. As a result of the rapid increase in the number of bariatric surgical procedures performed each year, the frequency of body contouring procedures has risen concomitantly over the past decade. Among the most popular of these is abdominoplasty for resection of redundant skin on the torso. When designing the outline of skin to be resected, among the preeminent concerns is maximizing the aesthetic result of the procedure. Removal of the redundant panniculus may involve a low transverse incision only or a more extensive resection such as the fleur-de-lis or corset pattern incisions. The length of the navel stalk may limit any transposition of this structure. Accordingly, one must plan for sitting the navel in an aesthetically pleasing location to complement the finished result. The umbilicus is typically inset along a horizontal line that spans the iliac crests. This will result in the most natural appearance for most individuals. A location 10 cm above the anterior vulvar commissure would result in a placement that is unnaturally low. The other options would yield a position of the umbilicus that is too high.
2016 For women undergoing abdominoplasty after massive weight loss, which of the following is the best position of the navel? A ) Along the line drawn between the iliac crests B ) At the horizontal level of the tenth ribs C ) Between the first and second tendinous inscriptions D ) In the midline 10 cm above the vulvar commissure E ) One-third of the distance from the xiphoid to the pubis
The correct response is Option B. The breasts, or mammary glands, are modified sweat glands. They are ingrowths from the ectoderm that form the lactiferous ducts and alveoli. They begin as linear mammary ridges with 15 to 20 buds. During the seventh week in utero, these buds undergo apoptosis, leaving a single pair of solid buds—the primary mammary buds—at the fourth or fifth intercostal space. Proliferating masses of mesenchyme are at the center of each limb bud. The mesoderm gives rise to organs, musculature, vasculature, and connective tissues. The endoderm becomes the epithelial lining of the alimentary tract. Bilateral mesenchymatous condensations develop into the sternum.
2016 The mammary glands develop from which of the following embryologic structures? A ) Bilateral mesenchymal condensations B ) Ingrowths from the ectoderm C ) Ingrowths from the mesoderm D ) Proliferating masses of endoderm E ) Proliferating masses of mesenchyme
The correct response is Option E. Extending the skin incisions too far medially can create a webbing of the nasal skin. Carrying the incision medially does not promote inadequate skin excision, lagophthalmos, or retrobulbar hematoma. Injury to the lacrimal system is unlikely to occur with a more medial incision.
2016 When skin excisions are designed for upper blepharoplasty, carrying the medial extent past the punctum is most likely to result in which of the following adverse effects? A ) Inadequate vertical skin excision B ) Injury to the lacrimal system C ) Lagophthalmos D ) Retrobulbar hematoma E ) Webbing of the nasal skin
The correct response is Option E. According to the literature, the skin island of the profunda artery perforator (PAP) flap is inferior to the gluteal fold.1,2 An ellipse of the inferior buttock with the inferior border within the gluteal fold describes the skin island of the inferior gluteal artery perforator (IGAP) free flap.3 An ellipse of the anteromedial thigh with the superior border within the gluteal fold describes the transverse upper gracilis (TUG) flap. An ellipse of the middle buttock, from the posterior superior iliac spine to the apex of the greater trochanter, describes the superior gluteal artery perforator (SGAP) flap.3 An ellipse of the lateral hip superior to the iliac crest describes the Rubens or lateral hip flap.4 The only option that correctly identifies the skin island for the PAP flap is an ellipse of the posteromedial thigh with the superior border within the gluteal fold.The superior marking is within or just below the gluteal fold and the inferior marking is roughly 7 cm below the superior marking. The flap is an ellipse so the scar does not extend outside of the gluteal fold.
2016 Which of the following is the most accurate location of the elliptical skin island of a profunda artery perforator (PAP) flap? A ) Anteromedial thigh with the superior border within the groin crease B ) Inferior buttock with the inferior border within the gluteal fold C ) Lateral hip superior to the iliac crest D ) Middle buttock, from the posterior superior iliac spine to the apex of the greater trochanter E ) Posteromedial thigh with the superior border within the gluteal fold
The correct response is Option E. Medically necessary criteria for Medicare approval of abdominal lipectomy/panniculectomy include: 1. Inability to walk normally 2. Chronic pain and ulceration created by the abdominal skin fold 3. When the panniculus hangs below the level of the pubis 4. Intertrigo of the pannus that is persistent or recurrent over a 3-month period while receiving appropriate medical therapy 5. Stable weight for at least 6 months and 18 months after gastric bypass surgery According to Medicare guidelines, the other four options listed are not considered reasons that make this procedure medically necessary: 1. Treatment of neck and back pain 2. Repairing abdominal wall laxity or diastasis recti 3. Improving appearance 4. Treating psychological symptomatology
2016 Which of the following symptoms meet the current Medicare guidelines for approval of abdominal lipectomy/panniculectomy following massive weight loss? A ) Neck and back pain B ) Psychological distress C ) Unsatisfactory appearance D ) Diastasis recti E ) Recurrent intertrigo